Download as pdf or txt
Download as pdf or txt
You are on page 1of 212

PRELIMINARY CONCEPTS

Remark. The authors have made an appendix of prerequisite knowledge; it contains,


notably, the following.

1. Riemann integral
Definition 1.1. A bounded real-valued function f : [a, b] → R is called Riemann
integrable if for every  > 0 there exists a partition P such that U (P, f )−L(P, f ) < .
Rb
Definition 1.2. For f integrable on [a, b] we define a f (x) dx to be the common
value
Z b
(1) f (x) dx = inf U (P, f ) = sup L(P, f ).
a P P

Remark. The following lemma will prove useful: that any integrable function can
be approximated (in some sense) by continuous functions. The proof also shows
that one can alternatively use step functions.

Lemma 1.3. Suppose f is integrable on the circle (i.e., a 2π periodic function on


R), bounded by B. Then there exists a sequence {fk } of continuous functions on
the circle, bounded by B, such that
Z π
(2) |f (x) − fk (x)| dx → 0 as k → ∞
−π

2. Measure Zero
Remark. A notion of “smallness” for sets.

Definition 2.1. E ⊂ R is said to be of measure 0 if, given any  > 0, there exists
a countable family of open intervals {Ik } such that
E ⊂ ∪k Ik
(1) P

(2) k=1 |Ik | < 

Lemma 2.2. The union of countably many sets of measure 0 is of measure 0.

Theorem 2.3. A bounded function f : [a, b] → R is integrable if and only if its set
of discontinuities is of measure 0.

Remark. The authors did not include the following knowledge in the appendix, but
it is also assumed. You will need to use this knowledge at various points, but not
prove it.

Elementary concepts: convergence of sequences, Cauchy sequences, Cauchy cri-


terion, completeness, density.
2 PRELIMINARY CONCEPTS
3. Pointwise convergence
Remark. Just as one can consider the convergence of a sequence of points, one can
consider the convergence of a sequence {fn } of functions. The simplest type of
convergence is “pointwise convergence”, implying that for each fixed point, the se-
quence of points {fn (x)} converges. (Reference: Rudin, Principles of Mathematical
Analysis, chapter 7)
Definition 3.1. E ⊂ X a matrix space, {fn : E → C} a sequence of functions.
If limn→∞ fn (x) exists for each x ∈ E (call that limit f (x)), then we say {fn }
converges pointwise to f .
Definition 3.2 ( − N rephrasing). If, for each x ∈ E, we have that given any
 > 0, there exists an N ∈ N such that n > N implies |fn (x) − f (x)| < , then we
say that {fn } converges pointwise to f on E.
Remark. There are certain important qualities that are not preserved by pointwise
convergence; that is, though all the {fn } might possess a certain property, their
pointwise limit f may not necessarily have that property.
Remarks.
(1) The pointwise limit of continuous functions need not be continu-
ous.
(2) A pointwise convergent sum of continuous functions need not be continuous.
(3) Limit and derivative need not commute.
(4) Limit of a convergent sequence of integrable functions need not
be integrable.
(5) Limit and integration do not commute

4. Uniform Convergence
Remark. There is a stronger form of convergence, however: so-called uniform con-
vergence, which turns out to preserve at least some of the above properties. (Ref-
erence: Rudin, Principles of Mathematical Analysis, chapter 7)
Definition 4.1 (uniform convergence). Let fn : E → C; n = 1, 2, 3, . . . . and f :
E → C. If, for all  > 0, there exists an N such that n > N implies |fn (x)−f (x)| < 
for all x ∈ E, then we say {fn } converges to f uniformly.
Remark. Intuition: the tube.
Theorem 4.2 (Cauchy criterion). {fn } converges uniformly to f on E ⇐⇒ for all
 > 0, there exists N such that m, n > N implies |fn (x) − fm (x)| <  for all x ∈ E.
Proof. ⇒) Fix  > 0. Since convergence is uniform on E, we know there exists
N > 0 such that if n > N then |fn (x) − f (x)| < /2 for all x ∈ E.
⇐) Note that for each fixed x ∈ E, {fn (x)} is a Cauchy sequence, and so
converges to some value, f (x). By hypothesis, given  > 0, there exists N such
that n > N implies |fn (x) − fm (x)| < 2 for all n, m ≥ N . Let m → ∞; we get
|fn (x) − f (x)| ≤ 2 . 

Theorem 4.3 (recharacterization). Say that limn→∞ fn (x) = f (x), and let Mn :=
supx∈E |fn (x)−f (x)|. Then fn → f uniformly on E if and only if limn→∞ Mn = 0.
PRELIMINARY CONCEPTS 3

Theorem 4.4 (Weierstrass M-test). Let {fn : E → C} be a set of functions.P


If there exist constants
P {Mn } such that |fn (x)| ≤ Mn for all x ∈ E and Mn
converges, then fn converges uniformly.
Proof. Use the Cauchy Criterion for sums. 

5. Consequences of uniform convergence


Theorem 5.1. If {fn } are continuous functions that converge uniformly to some
f , then f must be continuous.
Proof. Exercise. 
Theorem 5.2. Suppose fn : [a, b] → C are Riemann integrable functions. If fn →
f unifomly on [a, b], then f is itself Riemann integrable, and
Z b Z b
(3) f dx = lim fn dx.
a n→∞ a

Proof. Let n = sup[a,b] |fn (x) − f (x)|. Then fn − n ≤ f ≤ fn + n . Thus


Z b Z Z Z b
(4) (fn − n ) dx ≤ f ≤ f ≤ (fn + n ) dx
a a
Z Z Z b
(5) ⇒0≤ f≤ f≤ 2n dx = 2n (b − a)
a
Let n → ∞; then n → 0, so the upper and lower integrals are equal. 
Remark. It is not true that uniform convergence of fn → f implies that fn0 → f 0 .
However, one does have the following.
Theorem 5.3. {fn : [a, b] → C} be differentiable functions. If fn0 converge uni-
formly on [a, b], and the {fn } converge pointwise at some point x0 ∈ [a, b] then
(1) {fn } converge uniformly on [a, b] to some differentiable function f , and
(2) f 0 (x) = limn→∞ fn0 (x); x ∈ [a, b]

6. Fubini’s theorem
Remark. One theorem that will come in handy is the following. (Reference: Rudin,
Real and Complex Analysis, pp. 165ff)
Theorem 6.1. If f is a (measurable) function such that
Z Z
(6) |f (x, y)| dx dy < ∞,
R R
R R R R
then R RR fR (x, y) dx dy = R R f (x, y) dy dxthe order of integration in the double
integral R R f (x, y) dx dy may be reversed without changing the value of the inte-
gral.
Remark. Note that this is not true in general; see Rudin, p. 166 for examples.
THE WAVE EQUATION: D’ALEMBERT’S FORMULA

Initial Setting; Reduction of Problem

Suppose one has a string of length L > 0, fixed at both


ends. We let u(x, t) denote the displacement of the string
at position x at time t; e.g., u(x, 0) describes the string at
time t = 0, etc. Physical considerations imply that if u
is twice-differentiable, it satisfies the following (the “wave
equation”): The Wave
Equation

1 ∂ 2u ∂ 2u
(1) = 2,
c2 ∂t2 ∂x
for some constant c > 0.
Changing our units (let X = a1 x, T = 1b t), and letting
U (X, T ) = u(x, t), we get with appropriate choice (a =
L L
π , b = cπ ) of constants that the wave equation (1) above is
∂ ∂
∂T = b ∂t
∂ ∂
equivalent to the following: ∂X = a ∂x
c2 π 2 ∂ 2 u π2 ∂ 2 u
2 2 c2 L2 ∂T 2 = L2 ∂X 2
∂ U ∂ U
(2) 2
=
∂T ∂X 2
where 0 ≤ X ≤ π. That is, without loss of generality, we
may assume that the string is of length π, and that the
constant c = 1, i.e., that
∂ 2u ∂ 2u
(3) = 2 on 0 ≤ x ≤ π, t ≥ 0.
∂t2 ∂x
Goal: find
u(x, t)
Solutions on R are all combinations of satisfying
the above.
traveling waves.
Book gives
two methods
both relevant
one more so
2 THE WAVE EQUATION: D’ALEMBERT’S FORMULA
First observation. Suppose we ignore the initial condi-
tions (that u(0, t) = u(π, t) = 0 for all t > 0). Then, for
any twice differentiable function F , if we define either
(4) u(x, t) = F (x + t) or u(x, t) = F (x − t),
it is an easy calculation that either solves the equation.
Such solutions are called traveling waves, for obvious rea-
sons.
In fact, any twice-differentiable solution u to the
wave equation on R must be a combination of (op-
posing) traveling waves.
For let ξ = x + t, η = x − t, and notate
(5) ν(ξ, η) = u(x, t).
Then, again by the Chain Rule,
∂ ∂ξ ∂ ∂η ∂
(6) = +
∂x ∂x ∂ξ ∂x ∂η
∂ ∂
(7) = +
∂ξ ∂η
and, similarly,
∂ ∂ ∂
(8) = − ,
∂t ∂ξ ∂η
so the wave equation is equivalent to
∂ 2v ∂ 2v ∂ 2v ∂ 2v ∂ 2v ∂ 2v
(9) +2 + = 2 −2 + ,
∂ξ 2 ∂ξ∂η ∂η 2 ∂ξ ∂ξ∂η ∂η 2
i.e.,
∂ 2v
(10) = 0.
∂ξ∂η
Thus
(11) v(ξ, η) = F (ξ) + G(η),
or, switching back to x, t notation,
(12) u(x, t) = F (x + t) + G(x − t),
a sum of two waves in opposite directions, as claimed.
LECTURE 2: D’ALEMBERT’S FORMULA (CONT’D);
STANDING WAVES; HEAT EQUATION

Remark. Finishing up Chapter I (motivating the problems).


From Chapter II onwards the presentation will be more rig-
orous; however this first chapter introduces some important
themes.
Method I: Traveling Waves

Recall: last time we showed that every solution to the wave


equation can be expressed as a sum of two travelling waves,
in opposing directions. Q. What about fixed-
endpoint string mo-
Returning to our string of length π, let u(x, 0) = f (x); tion?
0 ≤ x ≤ π denote the initial configuration of the string; we
extend f first as an odd function to [−π, π], and then as a
2π-periodic function to all of R; do likewise for u(x, t) (also
set u(x, t) = u(x, −t) for t < 0). Then u is a solution for
the wave equation on all of R, so must be a sum of traveling
waves as above. What are F and G? Q. What are those
Well, we know waves?

(1) u(x, t) = F (x + t) + G(x − t),


so initial conditions

(2) f (x) = u(x, 0) = F (x) + G(x).


Not uniquely deter-
In fact, we haven’t imposed enough constraints to de- mined; we impose
termine the functions uniquely; we impose an additional
(“initial velocity”) condition, that Initial velocity con-
dition
∂u
(3) (x, 0) = g(x),
∂t
where g is an odd, 2π-periodic function satisfying g(0) =
g(x) = 0 like f .
2 D’ALEMBERT, STANDING WAVES, HEAT EQUATION

Thus our system is

(4) F (x) + G(x) = f (x)


(5) F 0 (x) − G0 (x) = g(x),
That’s enough; now which implies
we can solve it
(6) 2F 0 (x) = f 0 (x) + g(x)
(7) 2G0 (x) = f 0 (x) − g(x),
forcing
 Z x 
1
(8) F (x) = f (x) + g(y) dy + C1
2 0
 Z x 
1
(9) G(x) = f (x) − g(y) dy + C2
2 0
d’Alembert’s and thus (note C1 + C2 = 0)
formula
1 x+t
Z
1
(10) u(x, t) = [f (x + t) + f (x − t)] + g(y) dy.
2 2 x−t
That’s one way; an-
other which leads di-
rectly to the main Method II: superposition of standing waves
question of Fourier
analysis is Remark. Motivation for the notion of Fourier series - the
simplest version of Fourier analysis.
Same problem
Again we examine the wave equation
∂ 2u ∂ 2u
(11) = 2
∂t2 ∂t
with initial condition u(x, 0) = f (x) and initial velocity
Standing waves condition ∂u
∂t (x, 0) = g(x) as before.
Idea: consider solutions called “standing waves,” i.e., of
the simple form u(x, t) = φ(x)ψ(t). In that case, the
functions must satisfy the system
(12) φ00 (x) − λφ(x) = 0
(13) ψ 00 (t) − λψ(t) = 0,
D’ALEMBERT, STANDING WAVES, HEAT EQUATION 3

and thus (tossing out the non-oscillating solutions)


(14) ψ(t) = A cos(mt) + B sin(mt)
(15) φ(x) = Ã cos(mx) + B̃ sin(mx)
blah blah blah ....solutions are of the form φ(0) = 0; so à = 0;
φ(π) = 0; so m ∈ Z
(16) um (x, t) = (Am cos mt + Bm sin mt) sin mx if B̃ 6= 0

for m = 1, 2, 3, . . . (we combine negative indices with posi-


tive). infinite number of
So we have an infinite number of solutions. Recall/observe solutions

that any linear combination of solutions is still a solution,


and suppose u(x, t) is some linear combination of such so-
lutions, i.e,.

X
(17) u(x, t) = (Am cos mt + Bm sin mt) sin mx;
m=1
What are the Am and Bm ? Recalling that u(x, 0) =
f (x), we see
X∞
(18) Am sin mx = f (x).
m=1
The first question
This motivates the question: given a sufficiently good
function f on [0, π] with f (0) =Pf (π) = 0, can we find (Note we also see
coefficients Am such that f (x) = ∞m=1 Am sin mx?
from the initial vel.
cond’n that g(x) =
P∞
More generally, the question we will really be con- m=1 mB m sin mx.)
The more general
cerned with is the following: given an arbitrary function question
F on [−π, π], can we find coefficients am such that

X
(19) F (x) = am eimx
m=−∞

(note that eimx = cos(mx) + i sin(mx)).


Well, it is an easy calculation that
Z π 
1 0 if n 6= m
(20) eimx e−inx dx =
2π −π 1 if n = m
4 D’ALEMBERT, STANDING WAVES, HEAT EQUATION

“formally”: i.e., if so, at least formally, one would expect that


one ignores problems Z π
1
of convergence (21) an = F (x)e−inx dx.
2π −π
These coefficients are in fact called the Fourier coefficients
of F , and our job will be to determine in what sense, and
under what conditions, the above equality is true.
Remark. Recall from linear algebra: one may recognize this
infinite dimensional as an inner product space; then we have an orthonormal
orthonormal basis basis (infinite dimensional space). Fourier coefficients are
the coefficients...
The heat equation
Main points: 1. in-
troduce the terminol- Suppose one has an infinite plate (R2 ) with an initial
ogy: heat equation,
Laplacian; Dirichlet heat distribution. Let u(x, y, t) denote the temperature of
problem; polar coor- the place at time t, position (x, y). Physical considerations
dinates, 2. observe
that the same prob-
yield an equation governing the evolution of that heat:
lem arises.
Time-dependent σ ∂u ∂ 2 u ∂ 2 u
heat equation (22) = 2+ 2
∂u
κ ∂t ∂x ∂y
∂t =0
We’ll mainly be concerned with the steady-state heat
equation
∂ 2u ∂ 2u
(23) ∆u := 2 + 2 = 0
∂x ∂y
The Dirichlet problem (D some open domain (set), and
Dirichlet Problem ∂D its boundary):

∆u = 0 on D
(24)
u = f on ∂D
E.g., one can consider the Dirichlet problem on
D = {(x, y) ∈ R2 : x2 + y 2 < 1} = {(r, θ) ∈ R × S 1 : r ∈ [0, 1)}
the unit disk.
In polar coordinates, the condition ∆u = 0 becomes (ex-
Laplacian in polar ercise)
coordinates
D’ALEMBERT, STANDING WAVES, HEAT EQUATION 5

∂ 2 u 1 ∂u 1 ∂ 2u
(25) ∆u = 2 + + = 0,
∂r r ∂r r2 ∂θ2
2
2∂ u ∂u ∂ 2u
(26) i.e., r +r =− 2
Same approach as ∂r2 ∂r ∂θ
before: separation of We take the same “separation” approach as above: let
variables u(r, θ) = F (r)G(θ); then by a similar argument, we get
 00
G (θ) + λG(θ) = 0
(27)
r2 F 00 (r) + rF 0 (r) − λF (r) = 0
As before, we obtain a family of solutions,
(28) um (r, θ) = r|m| eimθ ; m ∈ Z
and, supposing that u be some linear combination
X ∞
(29) u(r, θ) = am r|m| eimθ
−∞
Different setting, of those solutions, we see that, in this different setting, we
same question arrive at an identical question. For the boundary value
condition requires that
X∞
(30) u(1, θ) = am eimθ = f (θ),
−∞
so, our question is, again: “Given any reasonable function
f on [0, 2π] with f (0) = f (2π), can we find coefficients am
such that
X∞
(31) f (θ) = am eimθ ?”
−∞
LECTURE 3: INTRODUCTION TO FOURIER SERIES

Basic definitions: basic function classes (read pp. 31-33


yourself), Fourier series; Examples: calculations of Fourier
series, Dirichlet and Poisson kernels (pp. 34-39)

1. Basic knowledge some definitions


(1) Continuous functions on [0, L].
(2) Piecewise continuous functions (only finitely many
discontinuities)
(3) Riemann integrable functions (note bounded)
(4) Functions on the circle (correspondence with 2π pe-
riodic functions on R such that f (0) = f (2π). Warning: only inte-
grable functions
Remark. We will assume that all of our functions are Rie-
mann integrable
2. Definitions and examples Important defn.:
Definition 2.1. Given an integrable function f : [a, b] → Fourier series

C, we define the Fourier series of f as



2πinx
X
fˆ(n)e L ,
n=−∞
where
Z b
1
fˆ(n) := f (x)e−2πinx/L dx
L a
denotes the n-th Fourier coefficient of f for n ∈ N. Fourier coefficient
Example of Fourier
Example: the Fourier series of the 2π-periodic odd func- series
tion defined on [0, π] by f (θ) = θ(π − θ).
Well, our function is f (θ) = θ(π − θ) for θ > 0 (with
derivative f 0 (θ) = π −2θ there), and f (θ) = θ(π +θ) for θ <
2 LECTURE 3: INTRODUCTION TO FOURIER SERIES

0 (with derivative π + 2θ there). Then, using integration


by parts, we get Only sketch it! Let
Z π them fill in the de-
ˆ 1
f (n) := f (θ)e−inθ dθ tails - it’s just calcu-
2π −π lus.

1 π 0
 
−f (θ) inθ
Z
1
= e + f (θ)e−inθ dθ
2π in −π in −π
 Z 0 Z π 
1 2
= θe−inθ dθ − θe−inθ dθ
2π in
Z π −π 0
1
=− θ(e−inθ + einθ ) dθ (C.O.V.: γ = −θ)
inπ 0

1 π
 Z 
2 θ
=− sin(nθ) − sin(nθ) dθ
inπ n 0 n 0
2
= 3 [(−1)n+1 + 1]
n πi
4
= 3
n πi
Note we don’t know for n odd, and 0 otherwise. Thus the Fourier series is
if there’s any rela-
tion - think of Taylor X 4
series
f (θ) ∼ 3
eikx .
k π
k odd

Remark. There is, a priori, no guarantee that the Fourier


series will converge at all; even if it does converge, it may
Simple observation not converge to f . In fact, if f and g agree everywhere
excluding a finite set of points, their Fourier series will be
identical. So asking for pointwise convergence to the origi-
nal function is in fact a priori futile.
partial sum

Definition 2.2. We define the N th partial sum of the Fourier


series of f , N ∈ N by
N
X
SN (f )(x) := fˆ(n)e2πinx/L .
n=−N
Define trigonometric
polynomials!
LECTURE 3: INTRODUCTION TO FOURIER SERIES 3

Remark. In this text, “convergence of the Fourier series to


f ” will always mean convergence of the above partial sums
to f .
3. Some important constructions Dirichlet kernel: for
now, opaque
Definition 3.1. We define the Dirichlet kernel DN by
N
X
DN (x) = einx
n=−N

where x ∈ [−π, π].


sin((N + 21 )x)
Lemma 3.2. DN (x) = sin(x/2)
Leave to them.

Proof. Consider the geometric sums


N
X −1
X
ix n
(e ) and (eix )n .
n=0 n=−N


Question. (dumb) what are the Fourier coefficients of DN ?
Poisson kernel:
arose in heat
Definition 3.3. We define the Poisson kernel Pr (θ) by equation

X
Pr (θ) = r|n| einθ
n=−∞

where θ ∈ [−π, π] and r ∈ [0, 1)


Remark. Note that the sum is both absolutely and uni-
formly convergent. (Obviously: notice the |n|.)
Question. (non-dumb?) what are the Fourier coefficients of
Pr (θ)? (Point: uniform convergence is necessary.)
1−r2
Lemma 3.4. Pr (θ) = 1−2r cos θ+r2
4 LECTURE 3: INTRODUCTION TO FOURIER SERIES

Proof. Letting ω = reiθ , we have a sum of geometric series:



X ∞
X
n
Pr (θ) = ω + ω̄ n
n=0 n=1
1 ω̄ 1 − |ω|2
= + =
1 − ω 1 − ω̄ |1 − ω|2
1 − r2
=
1 − 2r cos θ + r2

4. Uniqueness of Fourier Series
Remark. The question: if the Fourier series did recover the
original function uniquely, then functions with the same
Fourier coefficients would have to be equal; in particular, if
fˆ(n) = 0 for all n ∈ Z, then f ≡ 0. This is of course false.
However one has the following.
Finally: first real
theorem!
i.e., on [−π, π], with Theorem 4.1 (Uniqueness). Let f be an integrable func-
f (−π) = f (π) tion on the circle. Suppose fˆ(n) = 0 for all n ∈ Z. If f is
continuous at θ0 , then f (θ0 ) = 0.
Remark. Thus f ≡ 0 a.e., since integrable functions are
continuous except on a set of measure zero.
Remark. Observe that if fˆ(n) = 0 for all n ∈ Z, then f
must be orthogonal to all finite linear combinations of the
blah, i.e., orthogonal to all trigonometric polynomials.
Proof. Proof by contradiction. Suppose f (0) > 0 (WLOG
assume θ0 = 0). By continuity, we know f (x) > 21 f (0) in
some neighborhood (−δ, δ) of 0.
We now create a sequence {pk (θ)} of trigonometric poly-
nomials (i.e., finite linear combinations of the {einx : n ∈
Z}) as follows: Let
p(θ) = cos θ + ,
LECTURE 3: INTRODUCTION TO FOURIER SERIES 5

Draw a picture where  > 0 is chosen so small that outside of (−δ, δ), we
still have

|p(θ)| < 1 − .
2
Notice that by continuity of cos θ, we can choose a small
η > 0 such that inside of (−η, η), we have

p(θ) > 1 + .
2
k
Let pk (θ) = [p(θ)] (note that these are all trigonometric
polynomials). Point: powers of
Since the Fourier
P inx coefficients fˆ(n) := hf, e2πinx/L i are all p will (uniformly)
shrink outside of the
0, we have hf, cn e i = 0 for all trigonometric polynomi- δ neighborhood, but
als. However, we have just created a sequence of trigono- grow to infinity uni-
formly inside the η
metric polynomials for which that does not happen....(to neighborhood
be continued)  (The Idea)
LECTURE 4: UNIQUENESS, PART II

1. Uniqueness of Fourier Series, continued


Recall: we were in the midst of the following theorem:
i.e., f integrable
Theorem 1.1 (Uniqueness). Let f be an integrable func- on [−π, π], with
f (−π) = f (π)
tion on the circle. Suppose fˆ(n) = 0 for all n ∈ Z. If f is
continuous at θ0 , then f (θ0 ) = 0.
Proof. Proof by contradiction. Suppose f (0) > 0 (WLOG assume θ0 = 0). By conti-
nuity, we know f (x) > 21 f (0) in some neighborhood (−δ, δ) of 0.
We now create a sequence {pk (θ)} of trigonometric polynomials (i.e., finite linear com-
binations of the {einx : n ∈ Z}) as follows: Let
p(θ) = cos θ + ,
where  > 0 is chosen so small that outside of (−δ, δ), we still have Draw a picture

|p(θ)| < 1 − .
2
Notice that by continuity of cos θ, we can choose a small η > 0 such that inside of (−η, η),
we have

p(θ) > 1 + .
2
Let pk (θ) = [p(θ)]k (note that these are all trigonometric polynomials). Point: powers of
Since the Fourier coefficients fˆ(n) := hf, e2πinx/L i are all 0, we have hf, cn einx i = 0
P
p will (uniformly)
for all trigonometric polynomials. However, we have just created a sequence of trigono- shrink outside of the
metric polynomials for which that does not happen....(to be continued) δ neighborhood, but

The details: We estimate −π f (θ)pk (θ) dθ in three parts: grow to infinity uni-
formly inside the η
(1) In the η-neighborhood, we have the crude estimate neighborhood
Z (The Idea)
f (0) 
f (θ)pk (θ) dθ ≥ 2η (1 + )k The details
(−η,η) 2 2
which goes to infinity as k does.
(2) Outside of the δ neighborhood, we have the (again
crude) estimate
Z
≤ 2πB(1 −  )k

f (θ)p k (θ) dθ

(−δ,δ)c
2
where B is the bound on (integrable) f ; this goes to
0 as k → ∞.
2 LECTURE 4: UNIQUENESS, PART II

(3) Between the two, p and f are non-negative, so the


integral there is positive.
Together, the above prove that
Z
lim f (θ)pk (θ) dθ = ∞,
k→∞ (−π,π)
a contradiction. 

2. Consequences of Uniqueness theorem


Some consequences
Corollary 2.1. Suppose f is continuous on the circle. If
fˆ(n) = 0 for all n ∈ Z, then f ≡ 0.
Thus if two contin-
uous functions have
the same Fourier co- Proof. Obvious. 
efficients, they must
be identical. Question. At this point, do we know that if a function is
continuous, then its Fourier series converges back to the
function? (No. In fact, that is false.)
Recovering the func-
tion from the Fourier
series. Corollary P 2.2. Suppose f is a continuous function on the
circle, and ∞ ˆ
n=−∞ |f (n)| < ∞ (i.e., the Fourier series of
f is absolutely convergent). Then
lim SN (f )(θ) = f (θ)
N →∞
uniformly on the circle.
Note we do not have
this for cts. fns. in
Proof. (Trivial.) Since ∞ ˆ
P
general. n=−∞ |f (n)| < ∞, we know that
the Fourier series converges uniformly to some continuous
function, which has Fourier coefficients fˆ(n); n ∈ Z. Thus
we have two continuous functions with the same Fourier
coefficients, so they must, by the previous lemma, be iden-
tical. 
Natural question

Question. When do we have absolute convergence of the


sum of the Fourier coefficients?
Introduce a useful
notation
LECTURE 4: UNIQUENESS, PART II 3

Definition 2.3 (Big O notation). We say f (x) = O(g(x))


as x → a if there exists a C > 0 such that
f (x)
lim ≤ C.
x→a g(x)
ˆ
lim|n|→∞ |f|n|
(n)|
2 ≤ C
for some C
Corollary 2.4. Let f be a function on the circle. If f is
twice continuously differentiable (i.e., is of class C 2 ), then
fˆ(n) = O(1/|n|2 ) as |n| → ∞.
And thus we can re-
cover the function.
Proof. Proof is trivial: integration by parts.
Z 2π
2π fˆ(n) = f (θ)e−inθ dθ
0
2π
−e−inθ 1 2π 0
 Z
= f (θ) + f (θ)e−inθ dθ
in 0 in 0
Z 2π
1
= f 0 (θ)e−inθ dθ
in 0
Continuing with another integration by parts, Stop there for a sec-
ond! fb0 (n) = infˆ(n)
−inθ 2π Z 2π
 
1 −e 1 and so on and so
= f 0 (θ) + 2
f 00 (θ)e−inθ dθ forth
in in 0 (in) 0
Z 2π
1
=− 2 f 00 (θ)e−inθ dθ
n 0
So
Z 2π Z 2π
ˆ 1 00 −inθ 1
2
|f 00 | ≤ C,

|f (n)||n | ≤ f (θ)e dθ ≤
2π 0 2π 0
as desired. 
This is a good place to introduce the following important
notion:
Definition 2.5. Let f be a function for which there exists
a constant A such that for all x, h
|f (x + h) − f (x)| ≤ A|h|α .
4 LECTURE 4: UNIQUENESS, PART II

Then we say that f satisfies a Holder condition (or is Holder


continuous) of order α.
Remark. In fact (as you will show) if a function is Holder
continuous order α > 1/2, then the Fourier series converges
absolutely (and thus uniformly to f ).
LECTURE 5: CONVOLUTIONS AND GOOD KERNELS

1. Convolutions
Remark. Now for a seemingly simple, but important no-
tion....
Definition 1.1. Let f, g : R → C be 2π-periodic functions.
The convolution f ∗ g of f and g is the function defined on
[−π, π] by
Z π
1
(f ∗ g)(x) := f (y)g(x − y) dy
2π −π
Remarks.
(1) It is an easy exercise (C.O.V.) to see that f ∗g = g ∗f .
(2) Convolution as weighted average.
(3) Turns out that many important constructs can be
expressed in terms of convolutions. E.g., The Hilbert
Transform
For example, consider f ∗ DN , the Dirichlet kernel : “kernel” = “that
which one convolves
against”
Z π N
1 X
(f ∗ DN )(x) := f (y) ein(x−y) dy
2π −π −N
N Z π
X 1
= f (y)ein(x−y) dy
2π −π
−N
N Z π
X 1
= e inx
f (y)e−iny dy
2π −π
−N
XN
= einx fˆ(n) =: SN (f )(x),
−N
the N th partial sum of the Fourier series. The question about
convergence of
Fourier series can
be thought of as
the convergence
of a sequence of
particular weighted
averages.
2 LECTURE 5: CONVOLUTIONS AND GOOD KERNELS
2. Properties of Convolution
Theorem 2.1 (Basic properties). Let f, g, h be 2π-periodic
1
L (T) a Banach al- integrable functions, and c ∈ C. Then
gebra.
i. (Linearity I) f ∗ (g + h) = (f ∗ g) + (f ∗ h)
ii. (Linearity II) (cf ) ∗ g = c(f ∗ g) = f ∗ (cg)
iii. (Commutative) f ∗ g = g ∗ f
iv. (Associative) (f ∗ g) ∗ h = f ∗ (g ∗ h)
v. (Continuity!) f ∗ g is continuous.
vi. (Interaction with Fourier transform) f[ ∗ g(n) = fˆ(n)ĝ(n)
What is f\ ∗ DN (n)?
ˆ
f (n)χ[−N,N ] (n).
Remark. If one assumes that f, g, h are continuous func-
Obvious if continu- tions, then all of the properties, excluding the fifth, are
ous immediate calculations. E.g., (using Fubini’s theorem) one
can prove (vi) for continuous functions as follows.
Z π
1
f[∗ g(n) := (f ∗ g)(x)e−inx dx
2π −π
Z π Z π 
1 1
= f (y)g(x − y) dy e−inx dx
2π −π 2π −π
Z π  Z π 
1 1
= f (y)e−iny g(x − y)e−in(x−y) dx dy
2π −π 2π
Z π  Z−ππ 
1 1
= f (y)e−iny g(x)e−inx dx dy
2π −π 2π −π
= fˆ(n)ĝ(n)
Proof of (v), first for
cts. fns. (fairly stan- Proof. We will prove the second-to-last property, initially
dard argument) in the case that f, g are continuous. Then we will show
how one obtains the result for integrable functions by ap-
proximating them with continuous ones.
Step I. Suppose f, g are continuous; we want to show
f ∗ g is also. I.e., we want to show that given any  > 0,
there exists a δ such that |x1 − x2 | < δ implies |(f ∗ g)(x1 ) −
(f ∗ g)(x2 )| < . This is actually
uniform continuity;
note [−π, π] is
compact.
LECTURE 5: CONVOLUTIONS AND GOOD KERNELS 3

Well, we estimate crudely with the triangle inequality:


1 π
Z

|(f ∗ g)(x1 ) − (f ∗ g)(x2 )| ≤ f (y)[g(x 1 − y) − g(x 2 − y)] dy
2π −π
Z π
1
≤ |f (y)| |g(x1 − y) − g(x2 − y)| dy.
2π −π
“standard esti- f , being integrable, is bounded by some B on [−π, π], so
mates”
Key: uniform
we use the uniform continuity of g to choose δ > 0 such that
continuity of g |a − b| < δ implies |g(a) − g(b)| < B . Then, if |x1 − x2 | < δ,
1
we see the above is smaller than 2π 2πB B = . In other
words, the convolution is (uniformly) continuous.
What if we don’t
have continuity?
Question. Notice that we really needed the continuity of g
(but only boundedness of f ). How are we going to do this
without continuity?
Approximate (in
L1 ) with continuous Step II. Suppose f, g are integrable. By the lemma in
functions.
the appendix, any integrable function can be approximated
in the L1 sense by a sequence of continuous functions. So
let {fk } and {gk } be such sequences for f, g, respectively.
It suffices to show the following.
Why does this suf-
fice? (Dumb ques- Claim: fk ∗ gk → f ∗ g uniformly on [−π, π].
tion.)
Proof. First note that
f ∗ g − fk ∗ gk = (f − fk ) ∗ g + fk ∗ (g − gk ), so
|f ∗ g − fk ∗ gk | ≤ |(f − fk ) ∗ g| + |fk ∗ (g − gk )|
Now Z π
1
|(f − fk ) ∗ g(x)| ≤ |f (x − y) − fk (x − y)| |g(y)| dy
2π −π
Z π
1
≤ sup |g(y)| |f (y) − fk (y)| dy → 0
2π y −π
as k → ∞. Similarly for |fk ∗(g −gk )|; thus the convergence
depends on k (and not on x); i.e., we have the desired
uniform convergence. (End of proof of claim.) 
4 LECTURE 5: CONVOLUTIONS AND GOOD KERNELS
Then, by Step I, since fk ∗ gk are continuous, their uniform
limit f ∗ g is also. 
LECTURE 6: CONVOLUTIONS AND GOOD KERNELS

1. Convolutions, continued
Recall: we were proving certain properties about the con-
volution of two (integrable) functions; more importantly,
we were showing examples of how one extends results on
continuous functions to integrable functions, using the “L1
approximation lemma.”
Second example
Claim: (vi) For integrable f, g, f[∗ g = fˆĝ. of extending a
result from contin-
uous functions to
Proof. Let {fk }, {gk } be sequences of continuous functions integrable ones.
converging to f, g in L1 . In the previous example, we
showed that fk ∗ gk converges to f ∗ g uniformly, so, in- Key: uniform con-
terchanging integral and limit, vergence of fk ∗ gk

f\k ∗ gk (n) → f ∗ g(n)


[
for each fixed n ∈ Z. We have the result
Now, since fk and gk are continuous, we know already already for continu-
ous functions.
k ∗ gk = fk gbk . That is, we actually have
that f\ b
fbk (n)gbk (n) → f[
∗ g(n)
as k → ∞. So STS that
fbk (n)gbk (n) → fˆ(n)ĝ(n).
That follows from the L1 statement: fbk (n) → fˆ(n), since Where L1 conver-
Z π gence comes into
1 play.
|fˆ(n) − fbk (n)| = −inx

[f (x) − f k (x)]e dx
2π −π
Z π
1
≤ |f (x) − fk (x)| dx
2π −π
which goes to 0 as k → ∞ (similarly for gk (n)) so we are
done. 
2 LECTURE 6: CONVOLUTIONS AND GOOD KERNELS

2. Good kernels (Approximations of the


Identity)
Convolution as mul-
tiplication: What is Remark. One thinks of convolution as a kind of multiplica-
1?
tion; in fact, L1 (T) with convolution as multiplication is a
so-called Banach algebra.
Question. What’s the identity element (for this multiplica-
tion)?
Well, there isn’t one (it would be the Dirac delta function,
which is not a function). But although there isn’t an iden-
tity element, we can create a sequence of elements that
approximate the identity: a so-called good kernel.
An extremely useful
notion: an approxi-
mation of the iden- Definition 2.1. Let {Kn : T → R}∞ n=1 be a sequence of
tity functions on the circle. If
1

i. (“sliding in” condition) 2π −π Kn (x) dx = 1 for all n ∈
N, Rπ
1
ii. there exists M > 0 such that 2π −π |Kn (x)| dx ≤ M
for all n ∈ N, and
iii. given any δ > 0,
Z
|Kn (x)| dx → 0
δ≤|x|≤π
as n → ∞,
then we say {Kn }∞
n=1 is a family of good kernels or (more
commonly) an approximation to the identity.
What’s the point?
They approximate
the identity, i.e., the Theorem 2.2. Let f be an integrable function on the circle,
Dirac delta function. and {Kn } a family of good kernels. Then
lim (f ∗ Kn )(x0 ) = f (x0 )
n→∞
whenever f is continuous at x0 . If f is continuous every-
where, then the above limit is uniform.
Proof. Let  > 0 be given. We want to control |f ∗Kn (x0 )−
Sliding in the func- f (x0 )|. Well,
tion to take advan-
tage of smoothness
(here, continuity)
LECTURE 6: CONVOLUTIONS AND GOOD KERNELS 3
Z π
1
|f ∗ Kn (x0 ) − f (x0 )| = Kn (y)f (x0 − y) dy − f (x0 )
2π −π
Z π
1
= |Kn (y)[f (x0 − y) − f (x0 )]| dy
2π −π
We know f is continuous at x0 , so choose δ > 0 such that
|y| < δ implies |f (x0 − y) − f (x0 )| < . Then, obviously, we
break the integral into two parts, the first over |y| < δ, the
Using ||Kn ||1 ≤ M second its complement. For the first integral we get:
and continuity on
the first part Z Z π
1 
|Kn (y)[f (x0 − y) − f (x0 )]| dy ≤ |Kn (y)| dy
2π |y|<δ 2π −π

≤ M.

Using boundedness For the second integral,
of f and the shrink- Z Z
1 2B
ing of Kn |Kn (y)[f (x0 − y) − f (x0 )]| dy ≤ |Kn (y)| dy
2π δ≤|y|≤π 2π δ≤|y|≤π

By the third property of good kernels, the second integral


Q. What do the can be made arbitrarily small; thus we are done.
bounds depend on? Notice that the first bound is independent of x0 , but the
second depends on δ (which depends on x0 ). However, if
f were uniformly continuous, the second bound would not
depend on x0 ; in that case the convergence would be uni-
form. 
Remark. Notice that if the Dirichlet kernel DN were an
approximation of the identity, then we’d immediately have The Dirichlet kernel
is not good.
pointwise convergence of the Fourier series of an integrable
at every point of continuity (recall that we have no such Later: a continuous
function whose
results for integrable functions). Unfortunately, the kernel Fourier series
that governs convergence of the Fourier series is not an diverges
approximation of the identity.
LECTURE 7: CESARO AND ABEL SUMMABILITY

A different way of
recovering the func-
tion.
Remark. As we are starting to see, even if the function is
continuous, the Fourier series may not recover the function.
On the other hand, there should be enough information in
the Fourier coefficients to recover the function, especially There should be
for continuous functions: after all, we have the uniqueness enough informa-
tion!
theorem. How can we recover the function from the Fourier
series, if not in the direct way?

We shall present two different ways of averaging the Fourier


series to recover the function.

1. Cesaro summability
Definition 1.1. Given a sequence {cn }, let sn := nk=0 ck
P
be the sequence of partial sums. We define the N th Cesaro Cesaro sum of the se-
th ries
P∞sequence {sk } (a.k.a. the N Cesaro sum
mean σN of the
of the series k=0 ck ) by
s0 + s1 + · · · + sN −1
σN := .
N
Remark. Convergence implies Cesaro summability.

2. Fejer’s theorem The Fejer kernel


Definition 2.1. We define the Fejer kernel, FN , by let-
ting FN (x) denote the N th Cesaro mean of the sequence
{Dk (x)}; i.e.,
N −1
1 X
FN (x) := Dk (x).
N
k=0
2 LECTURE 7: CESARO AND ABEL SUMMABILITY
Convolving with FN Remark. Then consider:
gives the Cesaro sum N −1
of the Fourier series 1 X
f ∗ FN (x) := f ∗ Dk (x)
N
k=0
N −1
1 X
= Sk (f )(x),
N
k=0
the average of the first N partial sums of the Fourier series
of f .
The Fejer kernel is
an approximation of
the identity Lemma 2.2. One has
1 sin2 (N x/2)
FN (x) = ;
N sin2 (x/2)
further, the Fejer kernel is a good kernel.
Proof. Exercise: prove the closed form. One can use the
closed form to prove theRgood kernel criteria. FN being
1 π
positive, the fact that 2π −π FN = 1 gives us the first two
properties. The last follows from sin2 (x/2) ≥ cδ > 0 for
δ ≤ |x| ≤ π. For then the |FN (x)| ≤ N1cδ in that region, so
the integral converges to 0 as N → ∞. 
Since FN is a good
kernel, we immedi-
ately get Theorem 2.3. Let f be an integrable function on the circle.
If f is continuous at θ0 , then the Fourier series of f is
Cesaro summable to f at θ0 . Further, if f is continuous on
the entire circle, then the convergence of the Cesaro sums
is uniform.
We can recover the
function!
Corollary 2.4. Let f be integrable on the circle. If fˆ(n) =
0 for all n ∈ Z, then f = 0 at all points of continuity of f .
We knew this al-
ready.
Proof. Obvious: at points of continuity, the Fourier series
(namely, 0) is Cesaro summable to f . 
Corollary 2.5. Any continuous function on the circle can
be uniformly approximated by trigonometric polynomials.
LECTURE 7: CESARO AND ABEL SUMMABILITY 3

Proof. The Cesaro sums are averages of the partial sums


(which are trigonometric polynomials) and so are polyno-
mials themselves. 
3. Abel means and summation Abel means of a
Definition 3.1. Let ∞ series
P
k=0 be a series of complexP
numbers.
i. We define the Abel means A(r) of the series ck by Taking the terms
∞ as coefficients of a
X power series
A(r) := ck r k .
k=0
ii. If, for every 0 ≤ r < 1, the Abel means A(r) converges,
and Abel summable to s

lim A(r) = s
r→1
P
then we say the series ck is Abel summable to s.
Examples
Example. Consider ∞ k
P
k=0 (−1) = 1 − 1 + 1 − 1 + 1 . . . . It
diverges, but is Abel summable to s = 12 :

X 1
A(r) := (−1)k rk = .
1+r
k=0
Example. (Done in text.) Consider ∞ k
P
k=0 (−1) (k + 1) =
1 − 2 + 3 − 4 + 5 . . . . It diverges, but is Abel summable to
s = 14 :

X 1
A(r) := (−1)k (k + 1)rk = .
(1 + r)2
k=0

4. The Abel means of a Fourier series


and the Poisson kernel Abel means of a
Definition 4.1. Suppose we know the Fourier series of a Fourier series

function f :

X
f (θ) ∼ an einθ .
n=−∞
4 LECTURE 7: CESARO AND ABEL SUMMABILITY

We define the Abel means Ar (f )(θ) of the Fourier series of


the function f by

X
Ar (f )(θ) = r|n| an einθ
n=−∞

Remarks.
i. If we let c0 = a0 , cn = an einθ + a−n e−inθ , then the Abel
means of the PFourier series above equals the Abel means
of the series ∞ k=1 ck .
ii. For f integrable, |an | is uniformly bounded in n, so
Ar (f ) converges absolutely, and for each fixed 0 ≤ r <
1, uniformly.
This means can also
be recognized as a
convolution. Lemma 4.2 (Abel means as a convolution).
Ar (f )(θ) = (f ∗ Pr )(θ)
Proof. Recall the Poisson kernel

X
Pr (θ) := r|n| einθ
n=−∞
(We also saw that
2
Then
Pr (θ) = 1−2r1−r ∞
cos θ+r 2 X
for 0 ≤ r < 1.) Ar (f )(θ) := r|n| an einθ
n=−∞
∞  Z π 
X 1
= r|n| f (φ)e−inφ dφ einθ
n=−∞
2π −π

!
Z π
1 X
= f (φ) r|n| e−in(φ−θ) dφ
2π −π n=−∞
= (f ∗ Pr )(θ).
(Note we needed 
uniform conver-
gence.)
Pr is an approxima-
Lemma 4.3. The Poisson kernel is an approximation of
tion of the identity. the identity (as r ↑ 1).
LECTURE 7: CESARO AND ABEL SUMMABILITY 5

Proof. Recall we observed that



X
Pr (θ) := r|n| einθ
n=−∞

converges absolutely. We also saw that


1 − |ω|2 1 − r2
Pr (θ) = =
|1 − ω|2 1 − 2r cos θ + r2
where ω = reiθ . Pr is nonnegative.
From the above, we note that Pr (θ) ≥ 0, and that
Z π Z π X∞
1 1
Pr (θ) dθ = r|n| einθ dθ
2π −π 2π −π n=−∞
∞ Z π
X 1
= r|n| einθ dθ = 1,
n=−∞
2π −π
so the first two conditions for approximations of the identity
are satisfied. In the region out-
Now, notice that side of |θ| < δ,
the denominator is
1 − 2r cos θ + r2 = (1 − r)2 + 2r(1 − cos θ). bounded below.

1
For ≤ r ≤ 1 and δ ≤ |θ| ≤ π we see that
2
 2
1
1 − 2r cos θ + r2 = 1 − + (1 − cos θ) ≥ cδ > 0;
2
and thus
1 − r2
Pr (θ) ≤

in δ ≤ |θ| ≤ π; thus
Z
lim Pr (θ) dθ = 0,
r↑1 δ≤|θ|≤π

the third condition is also satisfied. 


Immediate conse-
quence
6 LECTURE 7: CESARO AND ABEL SUMMABILITY
Corollary 4.4. Let f be an integrable function on the cir-
cle. Then the Abel means of the (Fourier series of ) f con-
verges pointwise to f at every point of continuity. If, fur-
ther, f is continuous on the circle, then the convergence is
uniform.
Again we can re-
cover the function.
LECTURE 8: DIRICHLET PROBLEM ON THE UNIT
DISC

1. The Poisson kernel and


the Dirichlet Problem on the unit disc Recall the Dirich-
Recall the Dirichlet problem: let problem

∆u = 0 inside the unit disc D
u = f on ∂D
Guessed form of the
In Chapter I, we guessed that all solutions would be ex- solution
pressible as linear combinations of certain special functions;
precisely, that they would be of the form which we now recog-
∞ nize as Poisson inte-
X grals
u(r, θ) = fˆ(m)r|m| eimθ
m=−∞
=: Ar (f )(θ)
Z π
1
= f (φ)Pr (φ − θ)dθ
2π −π
Now we shall see that this is true.
Theorem 1.1. Let f be an integrable function defined on
the unit circle. Then the Poisson integral
u(r, θ) := (f ∗ Pr )(θ)
solves the heat equation on the disk. That is, u ∈ C 2 (D) The Poisson integral
and satisfies: does solve the heat
equation on the disc.
i. ∆u = 0.
ii. If f is continuous at θ, then
lim u(r, θ) = f (θ)
r↑1
and if f is continuous everywhere then the convergence
is uniform.
2 LECTURE 8: DIRICHLET PROBLEM ON THE UNIT DISC

iii. If f is continuous, then u(r, θ) is the unique solution to


the steady-state heat equation on the disc which satisfies
In fact, it is the conditions (i) and (ii).
unique solution.
Proof. i. First off, notice that since f is integrable, again,
the Fourier series expansion
X∞
u(r, θ) = fˆ(m)r|m| eimθ
m=−∞

of the Poisson integral converges absolutely and uniformly


on any disc 0 ≤ r < ρ, where ρ < 1. Because of (absolute
and) uniform convergence (of the derivatives of the partial
infinitely differen- sums see Rudin, Thm 7.17), we know that u is differentiable
tiable
term-by-term, infinitely many times.
Using the polar form of the Laplacian,
∂ 2 u 1 ∂u 1 ∂ 2u
∆u = 2 + +
∂r r ∂r r2 ∂θ2
Term-by-term we get, by differentiating term-by-term,
differentiation shows ∞ h
Poisson integral is X
harmonic. ∆u(r, θ) = |m|(|m| − 1)fˆ(m)r|m|−2 eimθ +
m=−∞
i
|m|fˆ(m)r|m|−2 eimθ + −m2 fˆ(m)r|m|−2 eimθ
= 0,
We do recover the so the Poisson integral is indeed harmonic.
boundary function.
(ii) is a restatement of the previous result.
To show (iii), suppose there is another such solution,
v(r, θ). It being twice-continuously differentiable, for each
fixed r ∈ (0, 1) we know v(r, ·) has a (uniformly convergent)
Uniqueness Fourier series
X∞
an (r)einθ ,
n=−∞

where an (r) = 1
2π −π v(r, θ)e−inθ dθ.
LECTURE 8: DIRICHLET PROBLEM ON THE UNIT DISC 3

Now, that series must satisfy the heat equation, ∆v = 0.


Plugging the above into the polar Laplacian, and assuming
that we can differentiate term-wise, we get Fake reasoning


X 1 0 n2
[a00n (r) + an (r) − 2 an (r)]einθ = 0
n=−∞
r r

and thus
1 0 n2
a00n (r) + an (r) − 2 an (r) = 0
r r
for all n ∈ Z.
The equation above implies (Exercise 11, Chapter 1) that
The coefficient must
be of a certain form
an (r) = An rn + Bn r−n
for some An and Bn . Now, for n ≥ 1, Bn = 0 since
Z π
1
an (r) := v(r, θ)e−inθ dθ
2π −π
is bounded; to find An , take
Z π
1
An = lim an (r) := lim v(r, θ)e−inθ dθ.
r↑1 r↑1 2π −π

Since v(r, θ) converges uniformly to f , we get An = fˆ(n). The coefficient is ac-


Thus for n ≥ 1, tually the Fourier co-
efficient!

an (r) = fˆ(n)rn ;
similarly for n < 0 (and for n = 0). All together, we have

X
v(r, θ) = fˆ(n)r|n| einθ
n=−∞

and so the solution is indeed unique. 


4 LECTURE 8: DIRICHLET PROBLEM ON THE UNIT DISC

2. Chapter III: L2 convergence of Fourier


Series
Definition 2.1. We define the L2 norm of a function f on
the circle by
Z π
1
||f ||2 := |f (θ)|2 dθ.
2π −π
L2 (T)
Review of basic lin- Some review:
ear algebra
i. Vector space V over R, over C.
Definitions: vector ii. Examples: Rd , Cd
space, inner prod- iii. Inner product (X, Y ) on a vector space V over R. Sym-
uct, positive defi-
nite, norm, Hermit- metric (X, Y ) = (Y, X), (strictly) positive definite (X, X) ≥
ian inner product.... 0, linear in both variables.
iv. Given an inner product (·, ·), we can define a norm on
V by
||X|| := (X, X)1/2 .
E.g., the dot product on Rd gives rise to the standard
Euclidean norm.
v. Inner product (X, Y ) on a vector space V over C. Her-
mitian: (X, Y ) = (Y, X); linear in the first variable,
conjugate linear in the second; etc.
Definition 2.2. Let V be a vector space over R or C with
inner product (·, ·) and associated norm ||·||. If (X, Y ) = 0,
orthogonal vectors we say that X and Y are orthogonal and write X ⊥ Y .
It turns out (see any good linear algebra book, e.g., Leon,
S. Linear Algebra) that any vector space with inner prod-
uct then has the following properties (read the proofs your-
selves).
Properties of (finite- Theorem 2.3. Properties of inner product spaces
dimensional?) inner
product spaces i. The Pythagorean Theorem: If X ⊥ Y , then ||X +
Y ||2 = ||X||2 + ||Y ||2 .
LECTURE 8: DIRICHLET PROBLEM ON THE UNIT DISC 5

ii. The Cauchy-Schwarz Inequality: Given any X, Y ∈ V ,


|(X, Y )| ≤ ||X|| ||Y ||
iii. The Triangle Inequality: Given any X, Y ∈ V , ||X +
Y || ≤ ||X|| + ||Y ||
Proof. The more complicated one is the Cauchy-Schwarz
inequality. First, if ||Y || = 0, then Showing that
(X, Y ) = 0 for all
0 ≤ ||X + tY ||2 for all t ∈ R X; note the inner
product is positive,
= (X + tY, X + tY ) = ||X||2 + 2t<(X, Y ) + 0. but not necessarily
If <(X, Y ) > 0, then taking t << 0 gives a contradiction; strictly.

similarly for <(X, Y ) < 0; thus <(X, Y ) = 0. Similarly for


=(X, Y ).
6 0, let c = (X,Y
Now, if ||Y || = )
(Y,Y ) ; then (X − cY ) ⊥ Y . By
the Pythagorean theorem,
||X||2 = ||X − cY + cY ||2
= ||X − cY ||2 + ||cY ||2 ≥ |c|2 ||Y ||2 ,
h i2
(X,Y )
i.e., ||X|| ≥ (Y,Y ) ||Y ||2 : the C-S inequality (squared).
2


LECTURE 9: THE FOURIER SERIES
RECOVER THE FUNCTION IN THE L2 SENSE

1. Two important infinite-dimensional


vector spaces
1.1. `2 (Z).
Definition 1.1. Let {an }n∈Z denote a (two-sided) sequence of complex
numbers. We define the “little `2 norm” of {an }n∈Z by By the two-sided
X sum we mean
||{an }n∈Z ||`2 = |an |2 the limit of the
n∈Z symmetric partial
and let `2 (Z) denote the vector space of all sequences whose little `2 norm sums.
is finite.
Definition 1.2. For A = {an } and B = {bn } in `2 (Z), we define the inner
product Inner product for
X `2 (Z)
(A, B) := an bn ;
n∈Z

and let ||A|| := (A, A)1/2 denote the related norm.


Question
Question. Is `2 (Z) even a vector space?
Proof. We need to show that if A, B ∈ `2 (Z), then so is A + B. That is, we
need to show that
X
lim |an + bn |2 < ∞.
N →∞
|n|≤N

However, this is obvious (why?): well, by the finite-dimensional triangle


inequality,
 1/2  1/2  1/2
X X X
 |an + bn |2  ≤  |an |2  +  |bn |2 
|n|≤N |n|≤N |n|≤N
≤ ||A|| + ||B||.
Taking the limit as N → ∞ shows that Just use the finite
dimensional C-S and
||A + B|| ≤ ||A|| + ||B|| < ∞
take the limit.
so we’ve demonstrated that A + B ∈ `2 (Z) (and simultaneously that the
infinite-dimensional triangle inequality holds). 
Example 2: a pre-
Hilbert space
2 L2 RECOVERY OF THE FUNCTION
Definition 1.3. An inner-product space with strictly positive-definite inner
product, which is complete with respect to the induced metric, is called a Hilbert space
Hilbert space.
Let R denote the set of complex-valued Riemann integrable functions on
[0, 2π], with addition, scalar multiplication, inner product and norm defined
R is not a Hilbert as usual. It turns out that R fails to be a Hilbert space in two senses:
space. first, the inner product is not strictly positive definite (||f || = 0 only implies
f = 0 a.e.), and second, the space is not complete.
1.2. Checking Cauchy-Schwarz in R.
Proof. Using the fact that 2AB ≤ (A2 + B 2 ), we see that for any λ > 0,
1
|f (x)g(x)| ≤ [λ|f (x)|2 + λ−1 |g(x)|2 ]
2
Cute proof of Then
Cauchy-Schwarz for 1
Z 2π
R |(f, g)| ≤ |f (x)g(x)| dx
2π 0
1
≤ [λ||f ||2 + λ−1 ||g||2 ];
2
||g||
taking λ = ||f || yields the Cauchy-Schwartz inequality. 

2. Fourier series and orthogonality


Remark. We’ve already basically mentioned this before, but we can express
the notion of Fourier series more simply using the language of inner products
and orthogonality.
Fourier series in
the language of i. Let R denote integrable functions on the circle.
infinite-dimensional ii. We define an inner product
inner product spaces Z 2π
1
(f, g) := f (θ)g(θ) dθ.
2π 0
with induced norm ||f ||22 = (f, f ) and (not-quite) metric d(f, g) : ||f −
g||2 .
iii. Let en (θ) := einθ . Easy to see that {en }n∈Z is an orthonormal set.
iv. Remark that the Fourier coefficients are precisely the coefficients of f
in terms of {en }n∈Z :
fˆ(n) = (f, en ).
v. For example, we can express the partial sum of the Fourier series as
X
SN (f ) = (f, en )en .
|n|≤N
Consequences of
having an orthonor- A few observations from linear algebra:
P
mal set. i. f − |n|≤N (f, en )en is orthogonal to en for all |n| ≤ N .
L2 RECOVERY OF THE FUNCTION 3

ii. Using the above, the Pythagorean theorem implies


||f ||2 = ||f − SN (f )||2 + ||SN (f )||2
X
= ||f − SN (f )||2 + |an |2 .
|n|≤N
P The partial sum of
iii. Given any other approximation |n|≤N cn en , we have the Fourier series is
X X the “best approxi-
||f − cn en ||2 = ||f − SN (f )||2 + || (an − cn )en ||2 , mation”
|n|≤N |n|≤N

and thus SN (f ) is the best approximation of f in the L2 sense.


3. Main theorem: Recovery in the L2 sense
Theorem 3.1. Let f be an integrable function on the circle. Then
Z 2π
1
|f (θ) − SN (f )(θ)|2 dθ → 0 as N → ∞,
2π 0
i.e., the Fourier series converges to f “in the L2 sense.”
Proof. Suppose g is continuous. By the Weierstrass approximation theorem,
there exists a sequence of trigonometric polynomials pn which converge to g
uniformly on the circle, i.e., such that given any  > 0, there exists N such Proof is trivial
that n > N implies for continuous
functions, using
|g(x) − pn (x)| <  for all x ∈ [0, 2π]. Weierstrass approxi-
Thus mation.
 Z 2π 1/2
1 2
||g − pn ||2 := |g(x) − pn (x)| dx < .
2π 0
By the “best approximation lemma”, we see that then for some sufficiently
large N (not the same N as above), ||g − SN || ≤  as well.
Suppose now that f is an integrable function. By the L1 approximation
lemma, we can find a continuous function g such that g has the same bound For integrable func-
(B, say) as f and tions, use the L1 ap-
Z 2π proximation lemma.
1 2
|f (x) − g(x)| dx < ,
2π 0 2B
The L2 difference is
 Z 2π 1/2  1/2
2B 2π
Z
1 2
|f (x) − g(x)| dx ≤ |f (x) − g(x)| dx
2π 0 2π 0
≤ .
As before, we can find a trigonometric polynomial p approximating g s.t.
||g − p||2 < ; thus ||f − p||2 < 2 and, by the best approximation lemma, a
partial sum SN (f ) of the Fourier series of f , for N sufficiently large, must
approximate f with at most that error.  (Main work is in the
Weierstrass approxi-
mation theorem)
LECTURE 10:
L2 RECOVERY OF INTEGRABLE FUNCTIONS
AND CONSEQUENCES

1. Main theorem: Recovery in the L2 sense (Have them do the


Theorem 1.1. Let f be an integrable function on the circle. Then second part as exer-
cise.)
Z 2π
1
|f (θ) − SN (f )(θ)|2 dθ → 0 as N → ∞,
2π 0
i.e., the Fourier series converges to f “in the L2 sense.”
Proof. Suppose g is continuous. By the Weierstrass approximation theorem, there exists
a sequence of trigonometric polynomials pn which converge to g uniformly on the circle,
i.e., such that given any  > 0, there exists N such that n > N implies Proof is trivial
|g(x) − pn (x)| <  for all x ∈ [0, 2π]. for continuous
functions, using
Thus
 Z 2π 1/2 Weierstrass approxi-
1 mation.
||g − pn ||2 := |g(x) − pn (x)|2 dx < .
2π 0
By the “best approximation lemma”, we see that then for some sufficiently large N (not
the same N as above), ||g − SN || ≤  as well.
Suppose now that f is an integrable function. By the L1 approximation lemma, we
can find a continuous function g such that g has the same bound (B, say) as f and For integrable func-
1
Z 2π
2 tions, use the L1 ap-
|f (x) − g(x)| dx < , proximation lemma.
2π 0 2B
The L2 difference is
Z 2π 1/2  1/2
2B 2π
 Z
1
|f (x) − g(x)|2 dx ≤ |f (x) − g(x)| dx
2π 0 2π 0
≤ .
As before, we can find a trigonometric polynomial p approximating g s.t. ||g − p||2 < ;
thus ||f − p||2 < 2 and, by the best approximation lemma, a partial sum SN (f ) of the
Fourier series of f , for N sufficiently large, must approximate f with at most that error.
 (Main work is in the
Weierstrass approxi-
Corollary 1.2 (Parseval’s Identity). Let f be an integrable mation theorem)
function, and an = fˆ(n). Then limN →∞ N 2
P
n=−N |an | con-
verges to ||f ||2 .
I.e., the map is an
isometry.
Proof. Recall that, by the Pythagorean theorem,
X
2 2
||f || = ||f − SN (f )|| + |an |2 .
|n|≤N
2 L2 RECOVERY OF THE FUNCTION

Remark. In particular, notice that ||an ||`2 = ||f ||L2 ; and one
has a correspondence between `2 and R. However, there
exist sequences in `2 that do not arise as Fourier series of
(R is incomplete.) functions in R
Comment about
physical meaning -
sounds Corollary 1.3 (Riemann-Lebesgue Lemma). f integrable
on the circle. Then fˆ(n) → 0 as |n| → ∞.
Useful variant of
Parseval:
Lemma 1.4 (Polarized Parseval’s identity). Let f and g be
integrable on the circle, with Fourier coefficients {an } and
{bn }, respectively. Then
Z 2π ∞
1 X
f (θ)g(θ) dθ = an bn .
2π 0 n=−∞

Proof. In any Hermitian inner product space, one has the


Polarization identity polarization identity
1
(f, g) = [||f + g||2 − ||f − g||2 + i(||f + ig||2 − ||f − ig||2 ].
4
Using this in R and Parseval’s identity, we get
1 hX 2
X
2
X
2
X
2
i
(f, g)2 = |an + bn | − |an − bn | + i( |an + ibn | − |an − ibn |
4
1
= [||an + bn ||2 − ||an − bn ||2 + i(||an + ibn ||2 − ||an − ibn ||2 ]
4
= (an , bn )`2 ,
by the polarization identity again...in `2 ! 
2. Return to pointwise convergence
Remark. Convergence in L2 does not guarantee that the
Fourier series converges for any θ. HUH? (blimps)
What about point-
wise convergence?
We do get some Theorem 2.1. Let f be an integrable function on the circle.
small results. Suppose f is differentiable at θ0 . Then limN →∞ SN (f )(θ0 ) =
f (θ0 ).
L2 RECOVERY OF THE FUNCTION 3

Proof. Consider the “slope of the secant line” function


 f (θ −t)−f (θ )
0 0
if t 6= 0 and |t| < π
F (t) := 0
t
−f (θ0 ) if t = 0
F , being differentiable at 0, is bounded near 0. It is, fur-
ther, integrable on δ < |t| for all δ > 0; thus F is integrable
on [−π, π] (see appendix).
Then
Z π
1
SN (f )(θ0 ) − f (θ0 ) = f (θ0 − t)DN (t) dt − f (θ0 )
2π −π
Z π
1
= [f (θ0 − t) − f (θ0 )]DN (t) dt
2π −π
Z π
1
= F (t)tDN (t) dt.
2π −π

Now,
t
tDN (t) = sin((N + 1/2)t)
sin(t/2)
t
= [sin(N t) cos(t/2) + cos(N t) sin(t/2)]
sin(t/2)
so we get
Z π
1
F (t)tDN (t) dt
2π −π
Z π
1 t
= F (t) [sin(N t) cos(t/2) + cos(N t) sin(t/2)] dt
2π −π sin(t/2)
which goes to 0 by the Riemann-Lebesgue lemma. 
Remark. In fact, the conclusion is true even for f only Lip-
schitz at θ0 .
Alternate proof. (Chernoff, P. The American Mathematical
Monthly, vol. 87, No. 5, May 1980, pp. 399-400.) Even cuter proof
4 L2 RECOVERY OF THE FUNCTION
WLOG assume that x0 = 0 and f (x0 ) = 0. Since f (0) =
0 and f 0 (0) exists, the function g(x) = f (x)/[eix − 1] is
(limit exists by bounded near 0 , and thus is integrable since f is.
L’Hôpital’s rule) Then
fˆ(n) = ĝ(n − 1) − ĝ(n),
a telescoping series, and
XN
SN f (0) = fˆ(n) = ĝ(−N − 1) − ĝ(N );
−N
which tends to 0 by the Riemann-Lebesgue lemma. 
Corollary 2.2 (Localization principle of Riemann). Let f
and g be integrable on the circle. Suppose f ≡ g in some
Convergence of neighborhood of a point θ0 . Then
SN (f )(θ0 depends
only on the behavior lim SN (f )(θ0 ) − SN (g)(θ0 ) = 0.
N →∞
of f near θ0 .
(Shocking!?) Remark. Note that neither f nor g need to be differentiable
at θ0 , and that this does not imply that the Fourier series
of either converges at θ0 , only that their convergence or di-
vergence is connected (and, if they converge, they converge
to the same limit).
LECTURE 11: FOURIER SERIES NEED NOT
CONVERGE AT POINTS OF CONTINUITY

1. Continuous function with


Fourier series diverging at a point
1.1. A function which is not a Fourier series of a
function in R.
Consider the series
−1
X einθ
.
n=−∞
n
Suppose the above is the Fourier series of some Riemann
integrable function, f . In that case, if we consider the Abel
means at 0, we get

X rn
|Ar (f )(0)| =
n=1
n
which diverges as r → 1. However, we should also have
Z π
1
|Ar (f )(0)| ≤ |f (θ)| Pr (θ) dθ ≤ sup |f (θ)|
2π −π θ
a contradiction, so the above is not the Fourier series of a
Riemann integrable function.
1.2. A continuous function whose Fourier series di-
verges at a point.
Let
X einθ X einθ
fN (θ) = ˜
and fN (θ) =
n n
1≤|n|≤N −N ≤n≤−1

Claim:
(i) |f˜N (0)| ≥ c log N
(ii) fN (θ) is uniformly bounded in N and θ
2 L2 RECOVERY OF THE FUNCTION
Recall Tauber’s
P the- (i) is evident. To prove (ii) will require a little machinery:
orem: if cn is
Lemma 1.1. Let ∞
P
Abel summable to
s, then
P
cn actu- n=1 cn be an infinite series. If
(i) the Abel means Ar = ∞ n
P
ally converges to s if n=1 r cn are bounded as r ↑ 1,
cn = o(1/n). and
(ii) cn = O(1/n)
then the partial sum sequence SN = N
P
n=1 is bounded.

Proof. We will control the difference between SN and Ar .


For
XN X∞
n
SN − Ar = (cn − r cn ) − r n cn
n=1 n=N +1
so
N
X ∞
X
n
|SN − Ar | ≤ |cn ||1 − r | + |rn ||cn |
n=1 n=N +1
Now, let us make three observations:
Coarse bound on (i) (1 − rn ) = (1 − r)(1 + r + · · · + rn−1 ) ≤ n(1 − r).
(1 − rn ) (ii) Since cn = O(1/n), we have n|cn | ≤ M for some M .
and the O(1/n) con-
trol (iii) We also have that for n ≥ N + 1, |cn | ≤ M N.
Using those two facts to bound the two sums, we con-
tinue:
N ∞
X M X n
≤M (1 − r) + r
n=1
N
n=N +1
M 1
≤ M N (1 − r) + .
N 1−r
Clever choice of r If we take r = 1 − 1/N , then we get
|SN − Ar | ≤ 2M.
Since the Ar are bounded for large enough N , we see that
SN are also bounded. 
Recall that we wanted to prove claim (ii), i.e.:
L2 RECOVERY OF THE FUNCTION 3

einθ
P
Claim: fN (θ) = 1≤|n|≤N n is uniformly bounded in N
follows immediately and θ.
from the lemma
Proof. (of claim (ii)) fN (θ) is the partial sum of the Fourier
inθ
series n6=0 e n , the Fourier series of the sawtooth function
P
f . Since Use the convolution
form of the Abel
(i) The Abel means are expressible as f ∗ Pr (θ), and since means
f is bounded (and ||Pr ||1 ≤ M ) the Abel means are
bounded.
(ii) cn = einθ /n + e−inθ /n, which is certainly O(1/n).
we see SN (f )(θ) is uniformly bounded in N and θ. 
1.3. Creating the example: the Heart of the Matter.

Recall Notice fN has no


X einθ n = 0 term.
X einθ
fN (θ) = and f˜N (θ) = ,
n n
1≤|n|≤N −N ≤n≤−1

trigonometric polynomials of degree N . We define frequency-


shifted versions of fN and f˜N , frequency-shifted
versions of fN , f˜N
PN (θ) = ei2N θ fN (θ) and P̃N (θ) = ei2N θ f˜N (θ),
which are trigonometric polynomials of degree 3N and 2N −
1, respectively.
Then if we consider the partial sums of PN , we see
Lemma 1.2.

 PN if M ≥ 3N
SM (PN ) = P̃ if M = 2N
 N
0 if M < N
(obvious)
P
Now choose any convergent positive series αk and se-
quence of integers {Nk } such that
(i) Nk+1 > 3Nk
(ii) limk→∞ αk log Nk = ∞
4 L2 RECOVERY OF THE FUNCTION
and let Creating the

X function!
f (θ) = αk PNk (θ).
k=1
Since |PN (θ) = |fN (θ)|, which were uniformly bounded, the
above series converges uniformly to a continuous periodic
It’s continuous. function.
However.... On the other hand,
Key claim:
|S2Nm (f )(0)| ≥ cαm log Nm + O(1)
Use the lemma Proof. (of this last claim)
X
S2Nm (f )(0) = S2Nm ( αk PNk )(0)
k
X
= αk S2Nm PNk (0)
X X
= αm S2Nm PNm (0) + αk PNk (0) + αk PNk (0)
m<k m>k
X
= αm P̃Nm (0) + 0 + αk PNk (0)
m>k
Here’s how those by the lemma.
two claims
into play.
come
Now, P̃Nm (0) = f˜Nm (0) which, recall, is of modulus ≈
c log Nm , and
X X
| αk PNk (0)| ≤ B| αk | ≤ BA
m>k k
since the PNk (0) = fNk (0), which were uniformly bounded
P
(Recall the αk =
A was a convergent (by B, say) in N . 
positive series.)
Thus the partial sums of the Fourier series diverge at 0,
despite the function being continuous everywhere.
Remark. It is possible to see, using the Baire category the-
orem and Banach-Steinhaus theorem, that the set of con-
tinuous functions whose Fourier series diverge at a point
is actually dense in the set of continuous functions (e.g.,
L2 RECOVERY OF THE FUNCTION 5

http://www.math.uchicago.edu/ may/VIGRE/VIGRE2010/
REUPapers/Stratmann.pdf for an undergraduate’s REU
summary of the argument).
LECTURE 12: FOURIER SERIES AND THE
ISOPERIMETRIC INEQUALITY

1. Basic knowledge about curves.


Definition 1.1. a C 1 mapping
γ : [a, b] → R2 ,
such that γ 0 (s) 6= 0, is called a parametrized curve. We call parametrized curve
the image of γ a curve (denoted by Γ, say). If γ is 1 − 1,
we call Γ simple; if γ(a) = γ(b) we call Γ closed.
Remarks.
(1) Can extend γ to a b − a periodic function on R.
(2) The smoothness conditions ensure the existence of a
continuous tangent vector.
Definition 1.2. We define the length of the curve Γ, parametrized
by γ(s) = (x(s), y(s)) by length of a curve
Z b Z b
`= |γ 0 (s)| ds = (x0 (s)2 + y 0 (s)2 )1/2 ds
a a

Definition 1.3. Let s : [c, d] → [a, b] be a bijective, C 1


mapping. Then we call
η(t) := γ ◦ s(t)
a (re-)parametrization of Γ. [Note length is independent of
parametrization.] If, further, reparametrization of
a curve
|η 0 (t)| = 1 for all t
we call η the arclength parametrization of Γ. arclength
parametrization
Definition 1.4. We define, as the area of the region en-
closed by the simple closed curve Γ, the value area enclosed by a
curve
2 ISOPERIMETRIC INEQUALITY, WEYL’S EQUIDISTRIBUTION THEOREM
Z
1
A = (x dy − y dx)
2 Γ
1 b
Z
0 0

= x(s)y (s) − y(s)x (s) ds
2 a
2. The Isoperimetric inequality and
Parseval’s identity
Theorem 2.1. Let Γ ⊂ R2 be a simple closed curve. Let
` denote the length of Γ, A the area of its enclosed region.
Then
`2
A≤ ,

with equality if and only if Γ is a circle.
Statement of
inequality
Proof. Part I. WLOG (by scaling the plane by (x, y) →
The parametrization (δx, δy) one may assume that ` = 2π.
is the arclength
parametrization.
Take the arclength parametrization γ(s) = (x(s), y(s)) =
of Γ; then
Z 2π
1
[x0 (s)2 + y 0 (s)2 ] ds = 1
2π 0
i.e., ||x0 ||22 + ||y 0 ||22 = 1.
Translation of this x(s) and y(s) are 2π periodic functions and have Fourier
fact in terms of
Fourier series (via
coefficients {an } and {bn }, respectively; their derivatives
Parseval). have coefficients {inan } and {inbn }, respectively. (Note
that an = a−n and bn = b−n since x, y are real-valued.)
Using Parseval’s identity on the above, we get
(1) ||{inan }||2`2 + ||{inbn }||2`2 = 1,
X∞
(2) i.e., |n|2 [|an |2 + |bn |2 ] = 1.
n=−∞
The area is what. Now, the area is by “definition”
Z b
1 1
x(s)y 0 (s) − y(s)x0 (s) ds

A = 2π
2 2π a
ISOPERIMETRIC INEQUALITY, WEYL’S EQUIDISTRIBUTION THEOREM 3

Using Parseval again, we get Translation of above


via Parseval.
X
(3) A = π an (−inbn ) − bn (−inan )


n∈Z
X
(4) ≤π |n||an bn − an bn |
n∈Z

Now, let’s bound this using the previous observation. Since


trivial quadratic in-
equality
(5) |an bn − an bn | ≤ 2|an | |bn | ≤ |an |2 + |bn |2
we see that (observe |n| ≤ |n|2 ) (with equality iff
X |an | = |bn |)
(6) A≤π |n|(|an |2 + |bn |2 )
n∈Z
X
(7) ≤π |n|2 (|an |2 + |bn |2 ) = π
n∈Z

using the statement (2) above about the arclength parametriza-


tion for the final equality. So we’ve shown the isoperimetric
inequality.
the case A = π
Part II. Why can A = π only in the case of a circle? We
make the following observations:
i. First: equality in (6, 7) could only occur if we have no
terms for |n| ≤ 2, i.e,
x(s) = a−1 e−is + a0 + a1 eis and
y(s) = b−1 e−is + b0 + b1 eis .
ii. Further, since a−1 = a1 and b−1 = b1 , (7) implies that
2(|a1 |2 + |b1 |2 ) = 1.
iii. Now, by (5), equality can hold only if
1
|a1 b1 − a1 b1 | = 2|a1 | |b1 | = |a1 |2 + |b1 |2 =
2
4 ISOPERIMETRIC INEQUALITY, WEYL’S EQUIDISTRIBUTION THEOREM

and thus, since (|an | − |bn |)2 = 0 implies |an | = |bn |,


|a1 | = |b1 | = 1/2. So
1 1
a1 = eiα and b1 = eiβ
2 2
for some α, β.
iv. In this notation, 1 = 2|a1 b1 − a1 b1 | (from (iii) above) is
equivalent to

1 i(α−β) −i(α−β)

[e
2 − e ] =1
i.e., | sin(α − β)| = 1.
so α − β = kπ/2 for some k ∈ Z.
So
1 1
x(s) = e−iα e−is + a0 + eiα eis = a0 + cos(α + s)
2 2
1 1
and y(s) = e−iβ e−is + b0 + eiβ eis = b0 + cos(β + s)
2 2

= b0 + cos(α + + s) = b0 ± sin(α + s)
2
- i.e., Γ is a circle, as desired. 
LECTURE 13: WEYL’S EQUIDISTRIBUTION
THEOREM

1. Number theory:
Weyl’s equidistribution theorem
(Another easily-
1.1. Basic knowledge. attained result)

Definition 1.1. Let x be a real number. Then


(1) Let [x], the integer part of x, denote the greatest in-
teger less than or equal to x.
(2) Let hxi := x − [x] denote the fractional part of x. basic definitions
(3) Given x, y ∈ R, if x − y ∈ Z we say
x ≡ y mod Z or x ≡ y mod 1.
Of course x ≡ y mod Z iff hxi = hyi.
1.2. Main theorem. The question: is
The problem: consider the collection {hnγi : n ∈ N}: is {hnγi} dense?

it dense in [0, 1)? Kronecker’s theorem: yes, if γ ∈


/ Q.
Definition 1.2 (Definition of equidistributed sequence).
If, for every interval (a, b) ⊂ [0, 1), then we call {ξn } an equidistributed
equidistributed sequence. sequence
Probability that an
#{n ∈ {1, 2, . . . , N } : ξn ∈ (a, b)} element of the (first
lim =b−a N points of the) se-
N →∞ N quence lies in the in-
Theorem 1.3 (Weyl’s Equidistribution theorem). If γ ∈ /Q terval

then {hnγi} is equidistributed in [0, 1). main theorem

Corollary 1.4 (Kronecker’s theorem).


1.3. Translation from number theory to analysis.
We rephrase the main theorem as follows. Let χ(a,b) de-
note the characteristic function of (a, b) ⊂ [0, 1), extended
as a 1-periodic function. Then we observe that Rephrasal of prob-
lem as an analysis
question
2 WEYL’S EQUIDISTRIBUTION THEOREM

N
X
#{1 ≤ n ≤ N : hnγi ∈ (a, b)} = χ(a,b) (nγ).
n=1

So the theorem can be rephrased as follows:


Theorem 1.5. Given any γ ∈ / Q, and any (a, b) ⊂ [0, 1),
N Z 1
1 X
lim χ(a,b) (nγ) = χ(a,b) (x) dx
N →∞ N 0
n=1
We show the cuba-
ture formula for con- 1.4. Main lemma.
tinuous functions.
Lemma 1.6 (Main lemma). f continuous and periodic
on [0, 1). If γ ∈
/ Q, then
N Z 1
1 X
f (nγ) → f (x) dx.
N n=1 0

Proof. (trivial)
Technique: state- i. Case: f ∈ {1, e2πix , . . . , e2πikx , . . . }. R1
ment trivial for
trigonometric
f = 1 is obvious. Otherwise, the integral 0 f = 0, so
polynomials, which we need to see that the average tends to 0. But, since
are L∞ dense in the e2πikγ 6= 1,
continuous periodic
functions. N N
1 X 1 X 2πiknγ
f (nγ) := e
N n=1 N n=1
e2πikγ 1 − e2πikN γ
=
N 1 − e2πikγ
(geometric sum) which goes to 0 as N → ∞.
ii. Case: trigonometric polynomials.
The problem is linear, so the lemma holds for linear
combinations of exponentials.
(by the uniform iii. Case: f a continuous periodic function. We know that
convergence of
the Cesaro means
there exists a trigonometric polynomial P such that
for continuous ||f (x) − P (x)||∞ < /3 and, by step i, that the lemma
functions, i.e,. the
goodness of the
Fejer kernel - here’s
where the Fourier
analysis enters)
WEYL’S EQUIDISTRIBUTION THEOREM 3

holds for P . Then



1 X N Z 1 N
1 X
f (nγ) − f (x) dx ≤ |f (nγ) − P (nγ)|+

N N

0
n=1 n=1
1 X N Z 1 Z 1
P (nγ) − P (x) dx + |P (x) − f (x)| dx.

N

n=1 0 0

The first and last are average differences between f


and P (and thus are controlled by 3 ); the middle term
is smaller than 3 for sufficiently large N by part i.

1.5. Proof of main theorem.
Proof of Weyl’s equidistribution theorem. Proof of Weyl’s
For each  > 0, let f+ and f− be continuous, 1-periodic equidistribution
theorem
functions which
(i) approximate χ(a,b) from above and below
(ii) are bounded by 1 (draw picture)
(iii) agree with χ(a,b) except on intervals of total length 2.
Then the averages of f− , χ(a,b) , and f+ have the following
relation:
N N N
1 X − 1 X 1 X +
f (nγ) ≤ χ(a,b) (nγ) ≤ f (nγ)
N n=1  N n=1 N n=1 
Now, the lemma implies (taking the limit as N → ∞) lim inf and lim sup!
Z 1 N
X N
X Z 1
f− (x) dx ≤ lim inf χ(a,b) (nγ) ≤ lim sup χ(a,b) (nγ) ≤ f+ (x) dx
0 N →∞ N →∞ 0
n=1 n=1
Since
Z 1 Z 1
b − a − 2 ≤ f− (x) dx and f+ (x) dx ≤ b − a + 2
0 0
and the above is true for all , we see that the desired limit
exists, and equals the desired b − a. 
4 WEYL’S EQUIDISTRIBUTION THEOREM

1.6. Observations.
Corollary 1.7. In fact, the main lemma holds even if f is
merely Riemann integrable.
Proof. Approximate f such a step function s such that ||f −
s||∞ < /3 (see the proof of the L1 approximation lemma).
Then
N Z 1 N N

1 X 1 X 1 X
f (nγ) − f (x) dx ≤ f (nγ) − s(nγ)

N N N n=1

n=1 0 n=1

N Z 1
1 X Z 1 Z 1

+ s(nγ) − s(x) dx + s(x) dx − f (x) dx

N 0 0 0
n=1
Since s is a finite linear combination of characteristic func-
tions of intervals, the middle term can be made smaller
than /3 by taking N sufficiently large; the other terms are
both also smaller than /3. 
Remark. Connection with dynamical systems: the system
is “ergodic”; that is, for all irrational γ, denoting
ρ(θ) = θ + 2πγ mod 2π,
the “time average”
N
1 X
lim f (ρn (θ))
N →∞ N
n=1
exists for each θ, and equals the “space average”
Z 2π
1
f (θ) dθ
2π 0
Remark. Notice that along the way, we proved the forward
direction of the following statement:
Theorem 1.8. Weyl’s criterion: A sequence of real num-
bers {ξi } in (0, 1) is equidistributed if and only if for all
WEYL’S EQUIDISTRIBUTION THEOREM 5

k ∈ Z,
1 X 2πikξn
lim e → 0.
N →∞ N
LECTURE 14: A CONTINUOUS, NOWHERE
DIFFERENTIABLE FUNCTION

1. Cool examples
i. Weierstrass’s example: Let a ∈ N, a > 1; b ∈ (0, 1).
Then if ab > 1 + 3π
2 ,

X
W (x) := bn cos(an x)
n=1

is a nowhere differentiable function (!).


ii. Riemann’s near-example:

X sin(n2 x)
R(x) :=
n=1
n2
πp
is differentiable att the points q where p, q are odd “at and only at”
integers. Using a lacunary
Fourier series to
Theorem 1.1. fα (x) = ∞ −nα i2n x
P
n=0 2 e , for α ∈ (0, 1) is a create a cts nowhere
differentiable
continuous, nowhere differentiable function. function

2. The main idea: delayed means

Recall that we have a couple of ways of summing the


Fourier series: first, the standard way, taking SN (g) = g ∗
DN with DN the Dirichlet kernel; second, the Cesaro way,
taking σN (g) = g ∗ FN , with FN the Fejer kernel. How things look
If we look at the partial sum SN on the Fourier coefficient on the Fourier co-
efficient side
side, we see that
\
SN (g)(n) = ĝ(n)DN (n)
d
2 A CONTINUOUS, NOWHERE DIFFERENTIABLE FUNCTION

where

1 if |n| ≤ N
D
d N (n) =
0 if |n| > N
Partial sums: trun- (Draw this!) I.e., we “chop off” the Fourier series for the
cation terms with indices |n| > N .
Doing the same thing for the Cesaro sums, we see that
on the Fourier coefficient side one weights them as follows:
Cesaro sums:
weighted truncation
σ\
N (g)(n) := g ∗ FN (n)
\
1 c
= ĝ(n) [D 0 (n) + D1 (n) + · · · + DN −1 (n)]
c \
 N 1
ĝ(n) N [N − |n|] for |n| ≤ N
=
0 for |n| > N.
Equivalently,
S0 (g)(x) + S1 (g)(x) + · · · + SN −1 (g)(x)
σN (g)(x) :=
N
N −1
1 XX
= ak eikx
N
`=0 |kl≤`
1 X
= (N − |n|)an einx
N
|n|≤N
X  |n|

= 1− an einx
N
|n|≤N

Definition 2.1. We define the delayed means ∆N (g) of the


Fourier series of g by
∆N (g) := 2σ2N (g) − σN (g)
= 2g ∗ F2N − g ∗ FN = g ∗ (2F2N − FN ).

Delayed means: Big On the Fourier coefficient side, it is easy to see that
triangle - little trian-
gle = trapezoid
A CONTINUOUS, NOWHERE DIFFERENTIABLE FUNCTION 3


 ĝ(n) if |n| ≤ N
\ |n|
∆N (g)(n) = ĝ(n)2(1 − 2N ) if N ≤ |n| ≤ 2N
0 if |n| > 2N

3. Getting the contradiction


Let’s recall our function:

n
X
fα (x) = 2−nα ei2 x , with α ∈ (0, 1)
n=0
the frequencies are of
Observation: For fixed N , if we choose the largest k for the form 2n , n ∈ N

which 2k ≤ N , then
∆2k (fα ) = SN (fα )
Even though 2k ≤ N , there are no frequencies between 2k We don’t miss any-
and N anyway, because of the lacunary nature of the series. thing, because of the
lacuna.
Dumb observation: If 2N = 2n , then
n
∆2N (f ) − ∆N (f ) = 2−nα ei2 x
(We can catch the
top term in an obvi-
Question. Why not do this using the partial sum operator? ous way.)

Our contradiction will be obtained as follows. By the


above dumb observation, for any point x0 ,
n
(1) |∆2N (f )0 (x0 ) − ∆N (f )0 (x0 )| = |i2n 2−nα ei2 x0 |
(2) = 2n(1−α) = (2N )1−α .
However, we shall prove the following
Lemma 3.1. Let g be continuous. If g is differentiable at
x0 , then
σN (g)0 (x0 ) = O(log N ).
Since
∆N (g)0 (x0 ) = 2σ2N (g)0 (x0 ) − σN (g)0 (x0 ) = O(log N ),
if fα is differentiable at some point x0 we would have a
contradiction with (??) above.
4 A CONTINUOUS, NOWHERE DIFFERENTIABLE FUNCTION

Proof of Lemma. Recall: σn g(x0 ) := FN ∗ g(x0 ) (with FN


the Fejer kernel), so
Z π
σN (g)0 (x0 ) = FN0 (x0 − t)g(t) dt
Z−ππ
= FN0 (t)g(x0 − t) dt
−π

Take the derivative by change of variables. Using the fact that −π FN0 (t) dt =
inside. 0, we see that we can slide in a constant to the above: and
Standard trick: use
cancellation to slide thus, using the differentiability of g at x0 ,
in a constant to Z π
take advantage of
smoothness = FN0 (t)[g(x0 − t) − g(x0 )] dt
−π

Z π
0
|σN (g) (x0 )| ≤ C |FN0 (t)| |t| dt.
−π
Useful estimates
FACTS: Useful estimates on FN0
i. |FN0 (t)| ≤ AN 2
ii. |FN0 (t)| ≤ |t|A2
Break up the inte- Putting it all together, we see
gral
Z π
0
|σN (g) (x0 )| ≤ C |FN0 (t)| |t| dt
Z−π Z
0
≤C |FN (t)| |t| dt + C |FN0 (t)| |t| dt
|t|≥ 1 |t|≤ N1
Z N Z
A 2 1
≤C 2
|t| dt + C AN dt
|t|≥ N1 |t| |t|≤ N1 N

The first term is O(log N ), and the second O(1). 


A CONTINUOUS, NOWHERE DIFFERENTIABLE FUNCTION 5

problem
Proof of the second fact. WTS that |FN0 (t)| ≤ A
|t|2 .
Well
1 sin2 (N t/2)
FN (t) = ,
N sin2 (t/2)
sin(N t/2) cos(N t/2) 1 cos(t/2) sin2 (N t/2)
so FN0 (t)= − .
sin2 (t/2) N sin3 (t/2)
1 cos(t/2) sin2 (N t/2)
 
1 sin(N t/2) cos(N t/2)
= −
sin2 (t/2) 1 N sin(t/2)
Now | sin(N t/2)| ≤ CN |t| and | sin(t/2)| ≤ c|t| for |t| ≤ π
so we get
0 CN |t| cos(N t/2) 1 cos(t/2)C 2 N 2 |t|2
so |FN (t)| ≤ − ....
c2 |t|2 N c3 |t|3
Remark. Something seems wrong here.

LECTURE 15: FINALLY, THE FOURIER TRANSFORM!

Remark. Normally, one defines the Fourier transform on


L2 (R). However, we cannot define this space (without first
defining the Lebesgue integral). Instead, we’ll work on the
Schwartz class S(R). When you are older (Book III) you’ll
see that this space is dense in L2 (R), and that one can ex-
tend uniquely our Fourier transform to L2 (R). By the way,
restricting ourselves to the Schwartz space “is a device that
allows us to come quickly to the main conclusions, formu-
lated in a direct and transparent fashion” (but in some
sense oversimplifies the matter).

1. Basic definitions improper integrals


Definition 1.1. We define (if the limit exists)
Z ∞ Z N
f (x) dx = lim f (x) dx
−∞ N →∞ −N

Definition 1.2. Let f : R → C be continuous. if there


exists A > 0 such that Vector space M (R)
of Functions of Mod-
A erate Decrease
|f (x)| ≤
1 + x2
for all x ∈ R, then we say f is of moderate decrease, and
denote by M (R) the (vector) space of such functions. Could use 1 +  in-
stead of 2.
R
Remark. It is easy to see that the improper integral Rf
converges for f ∈ M (R) R
N
(Exercise: show that then {IN := −N f } forms a Cauchy sequence.
2 LECTURE 15: FINALLY, THE FOURIER TRANSFORM!
Well, given  > 0, we see that
Z

|IM − IN | ≤ f

N ≤|x|≤M
Z  
1 1 1
≤A 2
dx = 2A −
N ≤|x|≤M x N M
2A
≤ ,
N
which we can make smaller than  as long as N is sufficiently large.)
Basic properties of
improper integration
Proposition 1.3. (Properties of the improper integral) Let
f, g ∈ M (R), and α, β ∈ C. Then
i. (Linearity)
Z ∞ Z Z
[αf + βg] = α f + β g.
−∞ R R
ii. (Translation invariance)
Z ∞ Z ∞
f (x − α) dx = f (x) dx.
−∞ −∞
⇒ L1 norm doesn’t iii. (Scaling under dilations) Given any δ > 0,
change under dila- Z Z ∞
tion δ f (δx) dx = f (x) dx.
R −∞
iv. (Continuity)
Z ∞
lim |f (x − h) − f (x)| dx = 0.
h→0 −∞
Stein will start to Proof. The proofs are straightforward.
leave out details Proof of ii. It suffices to show that
now...but this is Z N Z N 
nothing.
lim f (x − α) dx − f (x) dx = 0.
N →∞ −N −N
Well, via change of variables,
Z N Z N Z N −α Z N
f (x − α) dx − f (x) dx = f (x) dx − f (x) dx
−N −N −N −α −N
Z −N Z N
= f+ f
−N −α N −α
LECTURE 15: FINALLY, THE FOURIER TRANSFORM! 3

Now, take N > 2h. Then


Z −N Z N Z
A 3A
|f | + |f | ≤ dx = ,
−N −α N −α 1
2 N ≤|x|≤2N
x2 N
which goes to 0 as N → ∞.
Proof of iv. Given  > 0, we want to show there exists
H > 0 such that if |h| < H, then
Z N
lim |f (x − h) − f (x)| dx < .
N →∞ −N

WLOG take |h| ≤ 1 and N0 large enough that


Z Z
 
|f | ≤ and |f (x − h)| dx ≤
|x|≥N0 4 |x|≥N0 4
By the uniform continuity of f , we can choose H such that Point: f ∈ M (R)
for h < H, implies f lives inside
a lemon.

sup |f (x − h) − f (x)| <
|x|≤N0 4N0
Then for any N > N0 , we see that
Z N Z N0
|f (x − h) − f (x)| dx ≤ |f (x − h) − f (x)| dx
−N −N0
Z Z
+ |f | + |f (x − h)| dx
|x|≥N0 |x|≥N0
Z N0

≤ |f (x − h) − f (x)| dx +
−N0 2
 
≤ + = .
2 2
Thus
Z ∞
|f (x − h) − f (x)| dx ≤ 
−∞

for all h < H. 


4 LECTURE 15: FINALLY, THE FOURIER TRANSFORM!
2. The Fourier Transform
Definition 2.1. Given f ∈ M (R), we define the Fourier
transform fˆ of f by
Z ∞
fˆ(ξ) := f (x)e−2πixξ dx
−∞
Fourier transform

Remark. Easy to see that fˆ is a bounded, continuous func-


tion that vanishes at ∞. However, fˆ is not necessarily in
M (R) itself. This lack motivates the definition of the fol-
lowing class of functions.
Definition 2.2. We call a function rapidly decreasing if for
Rapidly decreasing every k ≥ 0, we have
function
sup |x|k |f (x)| < ∞,
x∈R
i.e., the function shrinks faster than the reciprocal of any
polynomial function.
Definition 2.3. Let f be an infinitely differentiable (C ∞ )
S(R) function. If f and all of its derivatives are rapidly decreas-
ing, we call f a Schwartz class function and write f ∈ S(R).
Remark. Observe that S(R) is a vector space (over C) and
is closed under both differentiation and multiplication by
polynomials.
Principle: decay of
the Fourier trans-
form corresponds to 3. Fourier transform on S(R)
the smoothness of f
(connects with 2.4 in Definition 3.1. For f ∈ S(R) we define the Fourier trans-
chapter II).
form of f by
Z ∞
fˆ(ξ) = f (x)e−2πixξ dx
−∞

Proposition 3.2. Let f ∈ S(R), h ∈ R, and δ > 0. Then


(the Fourier transform maps the following):
LECTURE 15: FINALLY, THE FOURIER TRANSFORM! 5

i. f (x + h) −→ fˆ(ξ)e2πihξ (translation becomes modula-


tion)
ii. f (x)e−2πixh −→ fˆ(ξ + h)
iii. f (δx) −→ δ −1 fˆ(δ −1 ξ) (dilation)
iv. f 0 (x) −→ 2πiξ fˆ(ξ) (differentiation becomes polynomial
multiplication)
v. −2πixf (x) −→ dξd fˆ(ξ)
Proof. The only one of interest is (v).
We want to show that
d
fˆ(ξ) + 2πixf

\ (ξ) → 0

Fix  > 0. Consider:

fˆ(ξ + h) − fˆ(ξ)
\
+ 2πixf (ξ)

h


Z Z
1
= f (x) [e−2πix(ξ+h) − e−2πixξ ] dx + 2πixf (x)e−2πixξ dx

h
ZR  −2πixh  R
e −1
= f (x)e2πixξ

+ 2πix dx
R h

...(to be continued). 
LECTURE 16: BASIC PROPERTIES
OF THE FOURIER TRANSFORM

Question (Badgering). What’s the difference between the


theory we’re developing now and the theory we developed
before? What was the “best” result we had for convergence
of Fourier series? Do we have anything parallel here?
1. Fourier transform on S(R)
Recall the Schwartz class S(R). Notice that it is closed
under differentiation and under multiplication by polyno-
mials.
Definition 1.1. For f ∈ S(R) we define the Fourier trans-
form of f by definition of the
Z ∞ Fourier transform:
fˆ(ξ) = f (x)e−2πixξ dx analogous to def-
inition of Fourier
−∞
coefficients
Proposition 1.2. Let f ∈ S(R), h ∈ R, and δ > 0. Then
(the Fourier transform maps the following): interactions with the
Fourier transform
i. f (x + h) −→ fˆ(ξ)e2πihξ (translation becomes modula-
tion)
ii. f (x)e−2πixh −→ fˆ(ξ + h)
iii. f (δx) −→ δ −1 fˆ(δ −1 ξ) (dilation)
iv. f 0 (x) −→ 2πiξ fˆ(ξ) (differentiation becomes polynomial
multiplication)
v. −2πixf (x) −→ dξd fˆ(ξ)
Proof. The main proof of interest is (v).
We want to show
that
d
fˆ(ξ) + 2πixf

\ (ξ) → 0

2 BASIC PROPERTIES

Fix  > 0. Consider:



fˆ(ξ + h) − fˆ(ξ)
\
+ 2πixf (ξ)

h


Z Z
1
= f (x) [e−2πix(ξ+h) − e−2πixξ ] dx + 2πixf (x)e−2πixξ dx

h
ZR  −2πixh  R
e −1
= f (x)e−2πixξ

+ 2πix dx
R h

We do need the sec- Now, f (x) and xf (x) are both rapidly decreasing, so
ond estimate
Z N ∈ N such that
there exists Z
|f | <  and |x||f (x)| dx < 
|x|≥N |x|≥N
and, by L’Hôpital’s Rule, for sufficiently small h we have,
For each fixed x0 , for the compact set |x| ≤ N ,
we can find an h. −2πixh
By continuity, that h e − 1 ≤ 

will will in a small
+ 2πix
neighborhood of x0 .
h N
Cover [−N, N ] with Outside of |x| ≤ N , we have the bound
such neighborhoods; −2πixh
choose the minimum e − 1 2 |sin(−πixh)|
h, which is indepen-
+ 2πix = + 2πix
dent of x.
h h
eix − 1 = 2| sin x2 | ≤ A + 2π|x|
sin h
since his bounded. Thus we have

fˆ(ξ + h) − fˆ(ξ)
\ (ξ)
+ 2πixf
h


Z N  −2πixh 
f (x)e−2πixξ
e − 1
≤ + 2πix dx + 
−N
h
≤ C

Corollary 1.3. f ∈ S(R) implies fˆ ∈ S(R).
BASIC PROPERTIES 3

Proof. Recall that S(R) is closed under differentiation and


multiplication by polynomials; thus if f ∈ S(R), then so is
Immediate conse-
 k quence of the above
1 d and the closure of
k
[(−2πix)l `f (x)]. S(R)
(2πi) dx
Thus the Fourier transform of the above, namely (by the
previous proposition)
 `
d
ξ k
fˆ(ξ)

is bounded (since the Fourier transform of any Schwartz
class function is bounded). Thus fˆ and all its derivatives
are rapidly decreasing, as desired. 
2. Creating an approximation of the identity
using dilated Gaussians
2
Definition 2.1. We call f (x) = e−x the Gaussian.
Remark. The Gaussian is a Schwartz class function; in fact
2
e−ax is in S(R) for all a > 0. The choice a = π is particular
because (You probably saw
Z ∞ 2 Z ∞ Z ∞ this in multivariable
−πx2 2 2
e dx = e−π(x +y ) dx dy calculus.)
−∞ −∞ −∞
Z 2π Z ∞
2
= e−πr r dr dθ
Z0 ∞ 0
2 2
= 2πre−πr dr = [−e−πr ]∞ 0 = 1.
0

Theorem 2.2. Let f (x) = e−πx . Then fˆ = f .


2

Proof. We shall show that fˆ satisfies a certain boundary


value problem. Let
Z ∞
2
F (ξ) := fˆ(ξ) = e−πx e−2πixξ dx
−∞
4 BASIC PROPERTIES

Then by property (v) in Proposition 1.2, we have


Z ∞
0
F (ξ) = [−2πixf (x)]e−2πixξ dx, which
−∞
Z ∞
=i f 0 (x)e−2πixξ dx since f 0 (x) = −2πxf (x)
−∞
= i2πiξ fˆ(ξ) = −2πξF (ξ).
(the Fourier trans- Elementary differential equations (separation of variables)
form of f 0 (x) is 2

2πiξ fˆ(ξ))
implies that F (ξ) = Ce−πξR ∞ , −πx
and, plugging in ξ = 0 we
ˆ
see C = F (0) = f (0) = −∞ e
2
dx = 1 by the previous
calculation. 
LECTURE 17: FOURIER INVERSION

1. Creating an approximation of the identity


using dilated Gaussians
Recall:
2
Definition 1.1. We call f (x) = e−x the Gaussian.
2
Remark. The Gaussian is a Schwartz class function; in fact e−ax is in S(R) for all a > 0.
The choice a = π is particular because (You probably saw
Z ∞
2
2 Z ∞ Z ∞
2 2
this in multivariable
e−πx dx = e−π(x +y ) dx dy calculus.)
−∞ −∞ −∞
Z 2π Z ∞ 2
= e−πr r dr dθ
Z0 ∞ 0
2 2
= 2πre−πr dr = [−e−πr ]∞
0 = 1.
0

−πx2
Theorem 1.2. Let f (x) = e . Then fˆ = f .

Proof. We shall show that fˆ satisfies a certain boundary value problem. Let
Z ∞
2
F (ξ) := fˆ(ξ) = e−πx e−2πixξ dx
−∞

Then by property (v) in Proposition ??, we have


Z ∞
F 0 (ξ) = [−2πixf (x)]e−2πixξ dx, which
−∞
Z ∞
=i f 0 (x)e−2πixξ dx since f 0 (x) = −2πxf (x)
−∞

= i2πiξ fˆ(ξ) = −2πξF (ξ).


2
Elementary differential equations (separation of variables) implies that F (ξ) = Ce−πξ , (the Fourier trans-
2
form of f 0 (x) is
R∞
and, plugging in ξ = 0 we see C = F (0) = fˆ(0) = −∞ e−πx dx = 1 by the previous
calculation.  2πiξ fˆ(ξ))

1.1. Creating the approximation of the identity.



Now, let, for δ > 0, Kδ denote a dilated (by δ) Gauss-
ian:
Kδ (x) = δ −1/2 f (δ −1/2 x)
2
= δ −1/2 e−πx /δ

Then, by the interaction of dilation and Fourier transform Miraculously (?), di-
lations of this eigen-
function form an ap-
proximation of the
identity.
2 LECTURE 17: FOURIER INVERSION

(function dilated by ρ maps to Fourier transform scaled by


ρ−1 ) we see
cδ (ξ) = fˆ(δ 1/2 ξ) = e−πδξ 2 .
Corollary 1.3. K
We do get an ap-
proximation of the
identity. Theorem 1.4. {Kδ }δ>0 is, as δ ↓ 0, an approximation of
the identity.
Proof. Easy. Kδ is a dilation of the Gaussian (a positive
function), which has L1 norm of 1, so the first two condi-
tions are satisfied. For the last condition,
Z Z
1 2
|Kδ (x)| dx = √ e−πx /δ dx
|x|>η δ
Z|x|>η
2 √
= e−πy dy (let x = δy)
|y|> √ηδ

which obviously (the integrand is rapidly decreasing) goes


to 0 as δ does. 
Remark. Consider: what would be the Fourier transform of
the Dirac delta function? As expected (?), on the Fourier
cδ (ξ) = e−πδξ 2 , which converges
transform side, we see that K
pointwise to the constant (≡ 1) function.
Convolution of S(R)
functions
Definition 1.5. Given f, g ∈ S(R), we define the convolu-
tion f ∗ g by
Z ∞
(f ∗ g)(x) := f (x − t)g(t) dt.
−∞

Corollary 1.6. Given any f ∈ S(R),


lim+ f ∗ Kδ (x) = f (x)
δ→0
uniformly.
The approximation
of the identity works
as before. Proof. First, we note that any Schwartz class function f ∈
S(R) is uniformly continuous on the real line. For given
any  > 0, we can choose an interval [−R, R] outside of
LECTURE 17: FOURIER INVERSION 3

which |f (x)| ≤ /4. f being continuous, it is uniformly


continuous on [−R, R], and thus onR all of R. f ∈ S(R) ⇒ f is

Now then, using the fact that −∞ Kδ = 1 and that uniformly uous.
contin-
(Recall the
Point: we needed Kδ (x) ≥ 0, Fourier coefficient
uniform continuity Z ∞ result.)
(first on the circle,
now on R) and the |(f ∗ Kδ )(x) − f (x)| = Kδ (t)[f (x − t) − f (x)] dt
decay of the good Z −∞ Z
kernel.
≤ + Kδ (t)|f (x − t) − f (x)| dt
|t|≥η |t|≤η

Both of these can be made small, independently of x (ex-


ercise). (Away from the origin, use the vanishing of the
approximation of the identity; near the origin, use the uni-
form continuity of f . The end.) 

2. Fourier inversion formula


Proposition 2.1 (“Multiplication formula”). Let f, g ∈
S(R). Then
Z ∞ Z ∞
f (x)ĝ(x) dx = fˆ(y)g(y) dy
−∞ −∞
Multiplication
formula: forerunner
to Plancherel’s
Remark. Recall Fubini’s theorem: If F (x, y) is a continuous theorem
function on R2 satisfying the condition
A
|F (x, y)| ≤
(1 + x2 )(1 + y 2 )
then
Z ∞ Z ∞ Z ∞ Z ∞
F (x, y) dy dx = F (x, y) dx dy.
−∞ −∞ −∞ −∞

Proof. Let F (x, y) = f (x)g(y)e−2πixy . Then certainly F


satisfies the decay conditions for Fubini’s theorem, and Just a consequence
of Fubini’s theorem
4 LECTURE 17: FOURIER INVERSION
Z ∞ Z ∞ Z ∞ Z ∞
−2πixy
f (x)g(y)e dy dx = f (x)g(y)e−2πixy dx dy,
−∞ −∞
Z ∞Z ∞ Z−∞ −∞
∞ Z ∞
i.e., f (x)ĝ(x) dx = fˆ(y)g(y) dy,
−∞ −∞ −∞ −∞
which is what we wanted to show. 
Theorem 2.2 (Fourier inversion). Let f ∈ S(R). Then
Z ∞
f (x) = fˆ(ξ)e2πixξ dξ.
−∞

Proof. We first prove this for x = 0.


Let Gδ (x) be what should be the inverse Fourier trans-
2
form of Kδ : i.e., let Gδ (x) = e−πδx ; then Gcδ (ξ) = Kδ (ξ)
(see scratchwork below). Then the multiplication formula
Use i. multiplication says
formula Z ∞ Z ∞
ii. that the Gaussian f (x)Kδ (x) dx = fˆ(ξ)Gδ (ξ) dξ.
behaves nicely, −∞ −∞
Let δ → 0 on both sides. The LHS is f ∗ Kδ (0), which
converges to f (0) (since the family is an approximation
R∞ of
the identity). The RHS “clearly” converges to −∞ fˆ(ξ) dξ
as δ ↓ 0 (exercise - recall fˆ ∈ S(R)), so the statement is
and iii. that we have proven for x = 0.
an approximation of For a general x, let F (y) = f (y+x). Then by the previous
the kernel.
case,
Z ∞ Z ∞
f (x) = F (0) = F̂ (ξ) dξ = fˆ(ξ)e2πixξ dξ.
−∞ −∞

−πx2
Scratchwork: Let γ(x) = e√ , i.e., the normalized Gaussian. Then our
above choice of Gδ (x) = γ( δx), and thus (scaling becomes dilation)
 
1 ξ 1 2
Gδ (ξ) = √ γ̂ √
c = √ e−πξ /δ = Kδ (ξ)
δ δ δ

Definition 2.3. Given g ∈ S(R), we define the inverse


Inverse Fourier Fourier transform ǧ of g by
transform
LECTURE 17: FOURIER INVERSION 5

Z ∞

F (g)(x) = ǧ(x) := g(ξ) e2πixξ dξ
−∞
Note that F(f )(y) =
Thus the Fourier inversion theorem can be written: for F ∗ (f )(−y).
f ∈ S(R),
ˇ
f (x) = fˆ(x).
It is easy to see that for g ∈ S(R), ǧˆ(ξ) = g(ξ) (i.e., F ◦F ∗ =
F ∗ ◦ F = I) and thus the Fourier transform is bijective on
S(R).
3. Plancherel’s theorem

Theorem 3.1 (Plancherel’s theorem). For any f ∈ S(R),


we have ||fˆ||L2 (R) = ||f ||L2 (R) .
Actually we don’t really need the above properties: Plancherel
can be obtained directly from the multiplication and inver-
sion formulae as follows. Simple proof of
Plancherel’s the-
Proof. The multiplication theorem says that for f, g ∈ S(R), orem: basically
Z ∞ Z ∞ just multiplication
f (x)ĝ(x) dx = fˆ(y)g(y) dy theorem
−∞ −∞

So that g such that ĝ = f¯, i.e., g = fˇ¯. Then we get ˆ ¯ = f¯


fˇ¯ = f¯; fˇ ˆ
Z ∞ Z ∞
f (x)f¯(x) dx = fˆ(y)fˇ¯(y) dy
−∞
Z−∞

¯
= fˆ(y)fˆ(y) dy,
−∞
as desired. 
LECTURE 18: WEIERSTRASS APPROXIMATION;
HEAT EQUATION ON THE LINE

1. F and convolutions: Plancherel’s theorem

Some further properties of the Fourier transform: Convolutions of


S(R) functions;
Proposition 1.1 (Fourier transform and convolutions). plus the Fourier
transform
Let f, g ∈ S(R). Then
i. f ∗ g ∈ S(R)
ii. f ∗ g = g ∗ f .
\
iii. (f ∗ g)(ξ) = fˆ(ξ)ĝ(ξ)
(Please read the proofs yourselves.)
These properties, plus the inversion formula, can be used
to give us the following useful theorem:
Theorem 1.2 (Plancherel’s theorem). For any f ∈ S(R),
we have ||fˆ||L2 (R) = ||f ||L2 (R) .

Proof. Let f [ (x) := f (−x), and h = f ∗ f [ (which is in


S(R), by the above proposition).
Applying the inversion formula to h at x = 0, we get
Z ∞
h(0) = ĥ(ξ) dξ
Z−∞

[
i.e., (f ∗ f )(0) = |fˆ(ξ)|2 dξ
Z ∞ Z−∞

i.e., 2
|f (x)| dx = |fˆ(ξ)|2 dξ
−∞ −∞

The moral: always
make your mistakes
in pairs, so that
one might cancel the
other one out.
2 WEIERSTRASS APPROX’N; HEAT EQUATION

2. Application: Weierstrass Approximation


Theorem
Any continuous
Theorem 2.1. Let f : [a, b] → C. If f is continuous, then function on a closed
given any  > 0, there exists a polynomial P such that and bounded inter-
val can be uniformly
sup |f (x) − P (x)| < . approximated by a
x∈[a,b] polynomial.
Create an extension
to all of R Proof. Let [−M, M ] be an interval containing [a, b]; let g
be a continuous function on R that agrees with f on [a, b]
and vanishes outside of [−M, M ]; let B denote a bound for
Approximate the ex- g.
tension. Since g is uniformly continuous, we can choose δ0 such
that for all x ∈ R,

|g(x) − (g ∗ Kδ0 )(x)| <
Approximate the ap- 2
proximation. Now,

1 −πx 2
1 X (−πx20 /δ0 )n
Kδ0 (x) = √ e δ0 =√ .
δ0 δ0 n=0 n!
The series expression converges uniformly on every compact
Actually we approx- interval of R, so there exists an N0 such that
imate the kernel us-
N0 2 n
ing a polynomial
1 X (−πx /δ0 ) 
Kδ0 (x) − √ ≤

δ0 n=0 n! 4M B
for all x ∈ [−2M, 2M ]; denote the finite sum by R(x).
Then, for x ∈ [−M, M ] (so that x − t ∈ [−2M, 2M ] for
and then notice that t ∈ [−M, M ]), using the above bound we see
the convolution with Z M
the polynomial ap-
proximates the con- |(g ∗ Kδ0 )(x) − (g ∗ R)(x)| = g(t)[Kδ0 (x − t) − R(x − t)] dt
volution with the −M
kernel
Z M
≤ |g(t)| |Kδ0 (x − t) − R(x − t)| dt
−M
≤ 2M B sup |Kδ0 (z) − R(z)|
z∈[−2M,2M ]
< /2
WEIERSTRASS APPROX’N; HEAT EQUATION 3

and thus g is uniformly approximated by g∗R on [−M, M ]. Putting the two


g is, of course, f on [a, b] ⊂ [−M, M ]. bounds together,

The last thing we need to show is that g ∗ R is actually


a polynomial. Well, Was that thing a
Z M polynomial?

(g ∗ R)(x) = g(t)R(x − t) dt
−M
2
PN0(−π(x−t) /δ0 )n
and R(x−t) = √1δ n=0 n! which is a polynomial
P2N00 n
(of the form n an (t)x ) in x 

3. Application to PDEs
3.1. Time-dependent Heat equation on the line.
Remark. Crucial property of Fourier Transform: interchanges
differentiation with multiplication by polynomials.
The problem: given an infinite rod and an initial tem-
perature distribution f (x) at t = 0, what is u(x, t), the
temperature at point x ∈ R at time t > 0? Physical con-
siderations imply that

∂u ∂ 2 u
= 2.
∂t ∂x
The heat equation

3.2. Finding the solution via the Fourier transform.


Taking (formally - assuming that, in particular, a solution
exists and that it is in S(R)) the Fourier transform (in the (formally = assum-
first variable) of both sides, we get ing that everything
works)

∂ û
(ξ, t) = −4π 2 ξ 2 û(ξ, t)
∂t
and thus, fixing ξ, one gets a trivial differential equation,
viz.
4 WEIERSTRASS APPROX’N; HEAT EQUATION

∂ û
∂t (ξ, t)
= −4π 2 ξ 2 ,
û(ξ, t)

2 2
⇒ û(ξ, t) = A(ξ)e−4π ξ t
.

Taking then the Fourier transform of the initial condition,


we get û(ξ, 0) = fˆ(ξ), so A(ξ) = fˆ(ξ). Thus
2 2
û(ξ, t) = fˆ(ξ)e−4π ξ t ,

and (taking the inverse Fourier transform of both sides)


we see that a solution, if it exists, must be of the form
The solution is con- u(x, t) = f ∗ Ht (x), where
volution with a par-
ticular kernel (which
we call the heat ker-
Ht (x) = K4πt (x).
nel).
We call Ht (x) the heat kernel of the line.

Theorem 3.1. Let f ∈ S(R), and let u(x, t) := (f ∗Ht )(x).


Then
i. u(x, t) is C 2 (R2+ ) and solves the heat equation,
ii. u(x, t) → f (x) uniformly in x as t → 0 (and thus is
continuous on R2+ )
iii. u(x, t) → f (x) in L2 as t → 0.

Proof. Using the Fourier inversion formula, we see that


Z ∞
u(x, t) = fˆ(ξ)H
ct (ξ)e−2πξx dξ;
−∞

differentation under the integral sign proves that it (is not


only infinitely differentiable but also) solves the heat equa-
tion. (ii) is immediate, since {Kδ } is an approximation of
Main part: showing the identity.
convergence in L2 ,
using Plancherel
WEIERSTRASS APPROX’N; HEAT EQUATION 5

How do we show L2 convergence? By Plancherel’s theo-


rem,
Z ∞ Z ∞
2
|u(x, t) − f (x)| dx = |fˆ(ξ)H
ct (ξ) − fˆ(ξ)|2 dξ
−∞
Z−∞

2 2
= |fˆ(ξ)|2 |e−4π tξ − 1| dξ.
−∞
Let  > 0 be fixed. Using the rapid decrease of f ∈ S(R),
choose N suchZ that Bound the part away
2 2 from the origin
(1) |fˆ(ξ)|2 |e−4π tξ − 1| dξ < .
|ξ|≥N

Then for all t small enough (note that fˆ is bounded), we


have Bound the part near
2 2  the origin
(2) sup |fˆ(ξ)|2 |e−4π tξ − 1| < .
|ξ|≤N 2N
So Z
2
tξ 2
(3) |fˆ(ξ)|2 |e−4π − 1| dξ < 
|ξ|≤N
for all small t, so the L2 difference is smaller than /2. 
LECTURE 19: THE FOURIER TRANSFORM AND
PARTIAL DIFFERENTIAL EQUATIONS, CT’D.

1. The Heat Equation, ct’d.


Recall: last time we proved the following. We defined the
heat kernel by Ht (x) := K4πt (x), i.e., We solved the heat
equation using the
1 −x2 /4t
Ht (x) = e appropriate kernel.
(4πt)1/2
ct (ξ) = e−4π2 tξ 2
H
Theorem 1.1. Let f ∈ S (R), and let u(x, t) := (f ∗
Ht )(x). Then
i. u(x, t) is C 2 (R2+ ) and solves the heat equation,
ii. u(x, t) → f (x) uniformly in x as t → 0 (and thus is
continuous on R2+ )
iii. u(x, t) → f (x) in L2 as t → 0.
We also saw that the solution could be expressed as:
Z ∞
u(x, t) = fˆ(ξ)H
ct (ξ)e−2πξx dξ;
−∞
and noted that for each fixed t, the convolution f ∗ Ht was
in S (R). In fact, we have something stronger: For fixed t, f ∗
Ht ∈ S (R) (con-
Corollary 1.2. u(·, t) ∈ S (R) uniformly in t in the sense volution of Schwartz
functions).
that given any T > 0,
` In fact, uniformly in
k ∂ S (R).

sup |x| ` u(x, y) < ∞
x∈R;t∈(0,T ) ∂x
for each k, ` ≥ 0.
Proof. We shall show that u(x, t) is rapidly decreasing, uni-
∂`
formly for t ∈ (0, T ); the argument is identical for the ∂x` u. Do it for u.
We bound |u| as follows: Break into parts
where |x−y| ≈ |x|....
2 FOURIER TRANSFORM AND PDES

Z Z
|u(x, t)| ≤ |f (x − y)|Ht (y) dy + same
|y|≤ |x|
2 |y|≥ |x|
2

So now, think: |y| ≤ |x| 2 , so |x − y| ≈ |x|. Thus, using the


Note that we rapid decay of f , we see that for the first integral,
couldn’t have used Z Z
this bound for all y. CN
|f (x − y)|Ht (y) dy ≤ Ht (y) dy
|x| (1 + |x|)N
|y|≤ |x|
2 |y|≤ 2
CN
≤ ;
(1 + |x|)N
that is, for any N ∈ N the first integral shrinks faster than
1
(1+|x|)N (is rapidly decreasing).
For the second integral, we see that (recall Ht (y) :=
−y2
1 |x|
(4πt)1/2 e 4t ) for |y| ≥ 2 , we have, for t ∈ (0, T ),
C −cx2
Ht (y) ≤ e t ,
t1/2
Note that we thus
couldn’t have used
this bound for all y Z Z
either. C −cx2
|f (x − y)|Ht (y) dy ≤ 1/2 e t |f (x − y)| dy
|x|
|y|≥ 2 t |y|≥ |x|
2
1 −cx2
≤ C 0 1/2 e t
t
is rapidly decreasing in x.... 

Theorem 1.3 (Uniqueness of solution). Suppose u(x, t)


satisfies the following conditions:
i. u solves the heat equation on R2+
ii. u(x, 0) = 0
iii. u is continuous on R2+
iv. u(·, t) ∈ S (R) uniformly in t (in the sense of Corollary
(1.2))
then u ≡ 0.
FOURIER TRANSFORM AND PDES 3

Proof. Define the energy of the solution u(x, t) at time t by


Z
E(t) := ||u(x, ·)||L2 (R) := |u(x, t)|2 dx
R
We notice that E(0) = 0 and that E ≥ 0; now we claim
that E 0 (t) ≤ 0. Differentiating under the integral sign, we This is so cool.
get
Z
dE
= [∂t u(x, t)ū(x, t) + u(x, t)∂t ū(x, t)] dx
dt R
Since u solves the heat equation, we see that ∂t u = ∂x2 u; so
(passing the derivatives over using integration by parts)
Z
dE
= [∂x2 u(x, t)ū(x, t) + u(x, t)∂x2 ū(x, t)] dx
dt RZ

=− [∂x u(x, t)∂x ū(x, t) + ∂x u(x, t)∂x ū(x, t)] dx


ZR
= −2 |∂x u(x, t)|2 dx ≤ 0.
R
The end. 
2. Steady-state heat equation in UHP
The boundary value problem we will examine now is the
following. Steady-state heat
(  2  equation in R2+ .
∂ ∂2
∆u := ∂x2 + ∂y2 u = 0 on R2+
u(x, 0) = f (x)
As before, we proceed formally. Taking the Fourier trans-
form in the first variable of the data above, we get
2 2 ∂2
−4π ξ û(ξ, y) + 2 û(ξ, y) = 0
∂y
û(ξ, 0) = fˆ(ξ)
As before, elementary ODE theory gives that the solution
must be of the form
û(ξ, y) = A(ξ)e−2π|ξ|y + B(ξ)e2π|ξ|y .
4 FOURIER TRANSFORM AND PDES

for some functions A(ξ), B(ξ). We assume B(ξ) ≡ 0, since


otherwise we’d have to take the inverse Fourier transform
of an exponentially growing function.
The boundary condition û(ξ, 0) = fˆ(ξ) implies then that
A(ξ) = fˆ(ξ), i.e.,
û(ξ, y) = fˆ(ξ)e−2π|xi|y
i.e., u(x, y) = f ∗ Py (x)
cy (ξ) = e−2π|ξ|y .
for Py satisfying P
Definition 2.1. We define the Poisson kernel on the UHP
The Poisson kernel by
on R2+ (is there some
1 y
relation?) Py (x) :=
π x2 + y 2
LECTURE 20: THE STEADY-STATE HEAT EQUATION
IN THE UPPER HALF PLANE.

Recall the problem: Steady-state heat


equation in R2+ .

∆u = 0 on R2+
u(x, 0) = f (x)
Taking the Fourier transform in the first variable, we ul-
timately saw that
û(ξ, y) = fˆ(ξ)e−2π|xi|y
i.e., u(x, y) = f ∗ Py (x)
cy (ξ) = e−2π|ξ|y .
for Py satisfying P
Definition 0.1. We define the Poisson kernel on the UHP
by The Poisson kernel
on R2+ (Obvious
1 y question: is there
Py (x) :=
π x2 + y 2 some relation
between the Poisson
Lemma 0.2 (Py (x) is what we would want it to be). kernel on the circle
Z ∞ and on the line?)

e−2π|ξ|y e2πiξx dξ = Py (x)


Z −∞

Py (x)e−2πixξ dx = e2π|ξ|y .
−∞
Inverse Fourier
transfprm of
Proof. RJust a calculation. Dividing the integral into two e−2π|ξ|y is Py (x),
0 R∞ and tautologically
parts, −∞ and 0 , we see: equivalent statement
Z ∞ Z ∞
−2πξy 2πiξx
e e dξ = e2πi(x+iy)ξ dξ
0
0 2πi(x+iy)ξ ∞
e 1
= =−
2πi(x + iy) 0 2πi(x + iy)
2 FOURIER TRANSFORM AND PDES

Similarly, Calculus I exercise.


Z 0
1
e2πξy e2πiξx dξ = ;
−∞ 2πi(x − iy)
together these two calculations give the first identity. The
second identity is equivalent to the first via the Fourier
inversion theorem (which we can use since e−2π|ξ|y and Py
are both of moderate decrease). 
As before, we are extremely lucky: the kernel with which
we convolve to obtain a solution of our equation is again
The question is: an approximation of the identity:
for what PDEs is
one guaranteed this Lemma 0.3. Py is an approximation of the identity on R
phenomenon? Cf.
Green’s functions. as y ↓ 0.
Proof. Trivial. We have already shown above that ||Py ||1 =
1, and the Poisson kernel is positive, so the only property we
need prove is the third, which is an(other) integral calculus
More calculus. problem:
Z ∞  
y π −1 δ
dx = · · · = − tan
δ x2 + y 2 2 y
which goes to 0 as y → 0. 
Now let us show that our intuited solution actually is one:
Theorem 0.4. Let f ∈ S (R); let u(x, y) := (f ∗ Py )(x)
for (x, y) ∈ R2+ . Then
i. u(x, y) ∈ C 2 (R2+ ) and ∆u = 0
ii. u(x, y) → f (x) uniformly as y → 0.
iii. u(·, y) converges to f in L2 as y → 0.
iv. Letting u(x, 0) = f (x), then u is continuous on R2+ and
vanishes at infinity in the sense that
u(x, y) → 0
as |x| + y → ∞.
FOURIER TRANSFORM AND PDES 3

Proof of (iv). (The rest are similar to the proof in the case
of the heat equation on the line, and left as exercises).
We note that if f is any function of moderate decrease,
then we have the following bound (first estimate): First estimate
 
1 y
|(f ∗ Py )(x)| ≤ C + .
1 + x2 x2 + y 2
proof of first esti-
For mate
Z ∞ Z Z
f (x − t)Py (t) dt = + .
−∞ |t|< |x|
2 |t|> |x|
2

For the first integral, |x − t| ≈ |x|, so


C C
|f (x − t)| ≤ ≈ .
1 + (x − t)2 1 + x2
For the second integral, |t| > |x|
2 , so
y y
Py (t) = 2 < C .
t + y2 x2 + y 2
Second estimate
We also have the second estimate
C
|(f ∗ Py )(x)| ≤ .
y
since supx Py (x) ≤ Cy (and we are “averaging” Py with a
function of moderate decrease - exercise).
To show the vanishing at infinity, then: when |x| ≥ |y|,
as |x| + y → ∞, we see that we have the bound
1 |y| 1 |x|
|(f ∗ Py )(x)| ≤ + ≤ +
1 + x2 x2 + y 2 1 + x2 x2 + y 2
1 |x|
≤ + → 0.
1 + x2 x2
And if |x| ≤ |y|, then
C
|f ∗ Py (x)| ≤ → 0.
y

4 FOURIER TRANSFORM AND PDES

Uniqueness of the solution will follow from the follow-


ing important (and in fact, characterizing) property of har-
monic functions:
Lemma 0.5 (Mean Value Property). Let Ω ⊂ R2 be open;
let u ∈ C 2 (Ω). if ∆u = 0 in Ω, then given any closed disc
MVP BR (x, y) ⊂ Ω, one has, for all r ∈ [0, R],
Z 2π
1
u(x, y) = u(x + r cos θ, y + r sin θ) dθ.
2π 0
Proof. Let U (r, θ) = u(x+r cos θ, y+r sin θ). Then ∆u = 0,
expressed in polar form, is equivalent to (easy exercise)
∂ 2U
 
∂ ∂U
0= +r r .
∂θ2 ∂r ∂r
Point: a con- Average
R 2π the above over the circle; then letting F (r) =
sequence of the 1
Laplacian being 2π 0 U (r, θ) dθ, we get
equal to 0. Z 2π
∂ 2U
 
∂ ∂F 1
r r = − 2 (r, θ) dθ
∂r ∂r 2π 0 ∂θ
Since ∂U
∂θ is periodic, the RHS of the above is 0; thus the
continuous function r ∂F
∂r is constant, and thus (take r = 0)
∂F ∂F
r ∂r = 0 (implying ∂r = 0); i.e., F is constant. Since
F (0) = u(x, y), we see that F (r) = u(x, y) for all r ∈ [0, R].
The end. 
LECTURE 21: THE STEADY-STATE HEAT EQUATION
IN THE UPPER HALF PLANE (END); POISSON
SUMMATION FORMULA

1. Mean Value Property and Uniqueness of


Solutions
Recall: we were about to prove the uniqueness of solu-
tions for the steady-state heat equation in the UHP using
the following important property of harmonic functions.
Lemma 1.1 (Mean Value Property). Let Ω ⊂ R2 be open;
let u ∈ C 2 (Ω). if ∆u = 0 in Ω, then given any closed disc
BR (x, y) ⊂ Ω, one has, for all r ∈ [0, R], MVP gives
Z 2π
1
u(x, y) = u(x + r cos θ, y + r sin θ) dθ.
2π 0
Remark. In fact, if a function is continuous on an (open,
connected) domain in Rn and satisfies the Mean Value
Property, then it is harmonic and in C ∞ .
Theorem 1.2 (Uniqueness of solutions to the steady-state
heat equation on R2+ ). Let u be a solution to ∆u = 0 on
R2+ . If Uniqueness of solu-
tions
i. u is continuous on R2+ ,
ii. u(x, 0) = 0, and
iii. u(x, y) vanishes at infinity
then u ≡ 0.
Proof. By contradiction. Suppose u(x, y) is (WLOG) real-
valued and u(x0 , y0 ) > 0 for some (x0 , y0 ) ∈ R2+ .
+
Choose a semi-disc DR := DR ∩ R2+ with R sufficiently
large that u(x, y) ≤ 12 u(x0 , y0 ) in the complement (note
+ +
(x0 , y0 ) ∈ DR ). Since u is continuous on DR , it attains
2 POISSON SUMMATION FORMULA

+
its maximum M at some point (x1 , y1 ) ∈ DR . Notice that
Observe: where a u ≤ M throughout R2+ . Now, by the MVP, we know that
max is attained, by Z 2π
the MVP the func- 1
tion must attain that u(x1 , y1 ) = u(x1 + ρ cos θ, y1 + ρ sin θ) dθ
max on every cir- 2π 0
cle centered at that
point. for, in particular, ρ ∈ (0, y1 ). Since u(x1 , y1 ) = M , u ≡ M
on that entire circle. Let ρ → y1 ; we see that then (since u
is continuous on R2+ ) u(x1 , 0) = M also: ※. 

Other properties of Remarks (Other properties of harmonic functions).


harmonic functions
i. Maximum principle: If u is continuous on the closure
of a bounded domain D and harmonic on the interior,
then the maximum must be attained on the boundary
(unless u is constant).
ii. Liouville’s theorem: A harmonic function on Rn which
is bounded must be constant.

Connection between
2. Poisson summation formula
analysis on circle
and R.
Definition 2.1. Let f ∈ S (R). We define the periodiza-
tion of f to be the (continuous) 1-periodic function F1 :
Periodization of a [0, 1] → C defined by
function

X
F1 (x) := f (x + n)
n=−∞

Remark. The sum converges absolutely and uniformly on


every compact subset of R, so converges to a continuous
function.
Poisson summation

Theorem 2.2 (Poisson summation formula). Let f ∈ S (R).


Then
X X
f (x + n) = fˆ(n)e2πinx ,
n∈Z n∈Z
The periodic func-
tion is the one we get
via a discrete version
of the Fourier trans-
form.
POISSON SUMMATION FORMULA 3

Remark. In particular,
X X
f (n) = fˆ(n).
n∈Z n∈Z

Proof. Call the second function F2 . Notice that F2 is (be-


cause fˆ ∈ S (R) also) again absolutely and uniformly con-
verging (on all of R), and so is continuous. Continuous ⇒ STS
Since F1 and F2 are continuous functions on the circle, Fourier coefficients
equal.
showing that they have the same Fourier coefficients would i.e., Fourier coeffi-
force their difference to be 0 everywhere. So we calculate cients as functions
on [0,1]
the Fourier coefficients of F1 : using uniform convergence,
we see
Z 1 X !
XZ 1
−2πimx
f (x + n) e = f (x + n)e−2πimx dx
0 n∈Z n∈Z 0
XZ n+1
= f (y)e−2πimy dy
n∈Z n
Z ∞
= f (y)e−2πimy dy = fˆ(m),
−∞

which is the m-th Fourier coefficient of F2 . 

Remark. Theorem (and proof) holds if f, fˆ are of moderate


decrease.

3. Applications of Poisson summation Using Poisson sum-


3.1. Heat Kernels on the Circle and on the Line. mation formula

Recall from Chapter IV: given a function u(x, t) where


t ≥ 0, x ∈ [0, 1] describing the temperature distribution on
a ring (with initial distribution f (x)), it can be shown that
u must satisfy the following problem: Recall heat kernel on
 ∂u ∂ 2 u the circle.

∂t = ∂x2
u(x, 0) = f (x)
4 POISSON SUMMATION FORMULA

As usual, we look for standing wave solutions: u(x, t) =


A(x)B(t), resulting in (an infinite superposition)
2 2
X
u(x, t) = an e−4π n t e2πinx .
n∈Z

Setting t = 0 shows us that an = fˆ(n).


Definition 3.1. We notate
2 2
X
Ht (x) = e−4π n t e2πinx
n∈Z
heat kernel on the and call Ht the heat kernel for the circle.
circle
Then the solution for the heat equation on [0, 1] with initial
data f can be written as u(x, t) = (f ∗ Ht )(x) (where the
convolution is on [0,1]).
Recalling that the heat kernel on the line was given by
1 −x2 /4t ct (ξ) = e−4π2 tξ 2 ,
Ht (x) = e ; i.e., H
(4πt)1/2
Heat kernel on the we note the following.
circle is the peri-
odization of the heat Theorem 3.2.
kernel on the line X
Ht (x) = Ht (x + n).
n∈Z

How cool! Proof. This is exactly the Poisson summation formula:


X X
Ht (x + n) = Hct (n)e2πinx
n∈Z n∈Z
2
tn2 2πinx
X
= e−4π e =: Ht (x).
n∈Z

LECTURE 21: THE STEADY-STATE HEAT EQUATION
IN THE UPPER HALF PLANE (END); POISSON
SUMMATION FORMULA

Theorem 0.1 (Uniqueness of solutions to the steady-state


heat equation on R2+ ). Let u be a solution to ∆u = 0 on
R2+ . If
i. u is continuous on R2+ ,
ii. u(x, 0) = 0, and
iii. u(x, y) vanishes at infinity
then u ≡ 0.
Proof. By contradiction. Suppose u(x, y) is (WLOG) real-
valued and u(x0 , y0 ) > 0 for some (x0 , y0 ) ∈ R2+ .
+
Choose a semi-disc DR := DR ∩ R2+ with R sufficiently
large that u(x, y) ≤ 12 u(x0 , y0 ) in the complement (note
+ +
(x0 , y0 ) ∈ DR ). Since u is continuous on DR , it attains
+
its maximum M at some point (x1 , y1 ) ∈ DR . Notice that
u ≤ M throughout R2+ . Now, by the MVP, we know that Observe: where a
Z 2π max is attained, by
1 the MVP the func-
u(x1 , y1 ) = u(x1 + ρ cos θ, y1 + ρ sin θ) dθ tion must attain that
2π 0 max on every cir-
for, in particular, ρ ∈ (0, y1 ). Since u(x1 , y1 ) = M , u ≡ M cle centered at that
point.
on that entire circle. Let ρ → y1 ; we see that then (since u
is continuous on R2+ ) u(x1 , 0) = M also: ※. 
Maximum principle:
Recall: If u is continuous
on the closure of
1. Poisson summation formula a bounded domain
D and harmonic on
Theorem 1.1 (Poisson summation formula). Let f ∈ S (R). the interior, then
the maximum must
Then X X be attained on the
f (x + n) = fˆ(n)e2πinx , boundary (unless u
is constant).
n∈Z n∈Z
Connection between
analysis on circle
and R.
Poisson summation
The periodic func-
tion is the one we get
via a discrete version
of the Fourier trans-
2 POISSON SUMMATION FORMULA

2. Applications of Poisson summation


Using Poisson sum-
mation formula 2.1. Heat Kernels on the Circle and on the Line.
Recall from Chapter IV: given a function u(x, t) where t ≥ 0, x ∈ [0, 1] describing the
temperature distribution on a ring (with initial distribution f (x)), it can be shown that
Recall heat kernel on u must satisfy the following problem:
the circle.  ∂u 2
= ∂∂xu2
∂t
u(x, 0) = f (x)
As usual, we look for standing wave solutions: u(x, t) = A(x)B(t), resulting in (an
infinite superposition)
X 2 2
u(x, t) = an e−4π n t e2πinx .
n∈Z

Setting t = 0 shows us that an = fˆ(n).

Definition 2.1. We notate


2 2
X
Ht (x) = e−4π n t e2πinx
n∈Z
heat kernel on the and call Ht the heat kernel for the circle.
circle
Then the solution for the heat equation on [0, 1] with initial data f can be written as
u(x, t) = (f ∗ Ht )(x) (where the convolution is on [0,1]).
Recalling that the heat kernel on the line was given by
1 −x2 /4t ct (ξ) = e−4π2 tξ 2 ,
Ht (x) = e ; i.e., H
(4πt)1/2
Heat kernel on the we note the following.
circle is the peri-
odization of the heat Theorem 2.2.
kernel on the line X
Ht (x) = Ht (x + n).
n∈Z

A cool consequence of the above expression:


Corollary 2.3. The heat kernel {Ht } on the circle is an
approximation of the identity (on the circle) as t ↓ 0.
Proof was too hard
to do until now.
Proof. Using uniform convergence, it is immediate that
Z 1/2
Ht (x) dx = 1.
−1/2
Since Ht ≥ 0, the above theorem implies that Ht ≥ 0
(not at all obvious otherwise); so the first two properties of
POISSON SUMMATION FORMULA 3

good kernels are satisfied. It remains to see that given any


η < 1/2,
Z
|Ht (x)| dx → 0 as t → 0.
η<|x|≤ 12

Well, consider:
X
Ht (x) = Ht (x + n)
n∈Z
X
= Ht (x) + Ht (x + n) =: Ht (x) + Et (x).
n∈Z∗

Since {Ht } is a good kernel, it suffices to show that This is so cool...we


can estimate the dif-
Z ference between the
|Et (x)| dx → 0 (good) heat kernel
on the line and its
|x|≤ 21
periodization.

as t → 0. We shall see that (claim:)


c
|Et (x)| ≤ Ce− t .

For, consider:

1 X −(x+n)2
Et (x) := √ e 4t
4πt n∈Z ∗
C X −cn2
≤ √ e t .
t n∈Z∗

since |x| ≤ 21 . Now, for t ∈ (0, 1] (which we can assume) we


see that

n2
 
1 1 2
≥ +n
t 2 t
4 POISSON SUMMATION FORMULA

(the n2 /t is greater than either of the terms averaged) and


so
C X − cn2
|Et (x)| ≤ √ e t
t n∈Z∗
C − c X − cn2
≤ √ e 2t e 2
t n∈Z∗
c
≤ Ce− t .
R
This bound implies the desired control on |x|≤ 1 |Et (x)| dx
2
and thus the third property of good kernels. 
2.2. Poisson kernels on the disc and upper half plane.
Showing Poisson Recall the Poisson kernels on the disc and upper half plane:
kernels related
1 − r2 1 y
Pr (ϑ) = and P y (x) = .
1 − 2r cos ϑ + r2 π y 2 + x2
Corollary 2.4. With r = e2πy ,
X
Pr (2πx) = Py (x + n)
n∈Z
Don’t need to men-
tion this in class.
Proof. Use the Poisson summation formula. 

3. Digression into analytic number theory


(Reference: Whittaker, E.T., and G.N. Watson, A Course
of Modern Analysis: An Introduction to the General Theory
of Infinite Processes and of Analytic Functions; With an
Account of the Principal Transcendental Functions, Cam-
bridge University Press, 1902.)
Definition 3.1. For s > 0, the theta function ϑ(s) is de-
theta function fined

2
X
ϑ(s) := e−πn s
n=−∞
Functional relation:
consequence of Pois-
son summation
POISSON SUMMATION FORMULA 5

Theorem 3.2 (Functional relation for ϑ). For s > 0,


 
1
s−1/2 ϑ = ϑ(s).
s
2
Proof. Consider the function f (x) = e−πsx ; its Fourier
transform (exercise) is
πξ2
1
fˆ(ξ) = s− 2 e− s .
Then, by Poisson summation,
∞ ∞
1 πn2
−πs(x+n)2
X X
e = s− 2 e− s e2πinx .
n=−∞ n=−∞
Evaluating at x = 0 yields the desired relation. 
Another theta func-
tion
Definition 3.3. We define the theta function Θ(z|τ ) for
z ∈ C, =(τ ) > 0 by

2
X
Θ(z|τ ) := eiπn τ e2πinz .
n=−∞

Remarks.
i. Θ(0|is) = ϑ(s).
ii. Θ(x|4πit) = Ht (x)
Digressing even fur-
ther....
Definition 3.4. For s ∈ C such that <(s) > 1, we define
the celebrated Riemann zeta function by The Riemann zeta
∞ function
X 1
ζ(s) =
n=1
ns

It can be shewn that ϑ, ζ, and Γ are related by


Z ∞
1 s
π −s/2 Γ(s/2)ζ(s) = t 2 −1 (ϑ(s) − 1) dt.
2 0
Remark. This will become more relevant later (in your life).
6 POISSON SUMMATION FORMULA

4. The Heisenberg Uncertainty Principle


Remark (Motivation). To what extent can one simultane-
ously locate the position and momentum of a particle?
In quantum mechanics, a particle has associate with it a
state function state function ψ (of L2 norm 1) which governs the position
in the sense that the probability that the particle lies in
aR particular region (a, b) ∈ R (one-dimensional space) is
2
(a,b) |ψ| . Then the expectation (expected position) is given
expected position by
Z ∞
x := x|ψ(x)|2 dx,
−∞
and the variance (uncertainty of the expectation) is given
variance of position by
Z ∞
(x − x)2 |ψ(x)|2 dx.
−∞
One has an analogous function describing the momentum
of the particle. Importantly, it turns out that the proba-
bility
R of the momentum belonging to an interval (a, b) is
2
(a,b) |ψ̂(ξ)| dξ. We shall now see the Heisenberg Uncer-
Heisenberg Uncer- tainty Principle, i.e., that
tainty Principle
1
Variance of position × Variance of momentum & .
16π 2
Theorem 4.1. Let ψ ∈ S (R), and suppose ||ψ||2 = 1.
Then
Z ∞  Z ∞ 
1
x2 |ψ(x)|2 dx ξ 2 |ψ̂(ξ)|2 dξ ≥
−∞ −∞ 16π 2
2
−Bx
and
p equality holds iff ψ(x) = Ae where B > 0, A2 =
2B/π.
In fact, we have, for every x0 , ξ0 ∈ R, blahblahblah (with
the individual terms minimized when x0 = x, ξ0 = ξ.
Easy calculation: in- Proof. Integration by parts implies the following.
tegration by parts
POISSON SUMMATION FORMULA 7

Z ∞
1= |ψ(x)|2 dx
−∞
Z ∞
d
=− x |ψ(x)|2 dx
dx
Z−∞
∞  
0 0
=− xψ (x)ψ(x) + xψ (x)ψ(x) dx.
−∞
Thus Z

1≤2 |x||ψ(x)||ψ 0 (x)| dx
−∞
Z ∞ 1/2 Z ∞ 1/2
2 2 0 2
≤2 x |ψ(x)| dx |ψ (x)| dx
−∞ −∞
Z ∞ 1/2  Z ∞ 1/2
=2 x2 |ψ(x)|2 dx 4π 2 ξ 2 |ψ̂(ξ)|2 dξ ,
−∞ −∞
using the Plancherel theorem (and the basic properties of Followed by
the Fourier transform) for the equality in the last line. Now, Cauchy-Schwartz
and Plancherel’s
equality can hold only if equality held in the application of theorem
the Cauchy-Schwartz inequality. which implies that the
functions must be scalar multiples of each other:
ψ 0 (x) = βxψ(x)
for some scalar β. Again, elementary ODE theory implies
2
ψ(x) = Aeβx /2 .
To ensure the function is in S (R), we requirep
β = −2B for
2
some positive B; then ||ψ||2 = 1 forces A = 2B/π. 
LECTURE 21: THE STEADY-STATE HEAT EQUATION
IN THE UPPER HALF PLANE (END); POISSON
SUMMATION FORMULA

Recall:

1. Poisson summation formula Connection between


Theorem 1.1 (Poisson summation formula). Let f ∈ S (R). analysis
and R.
on circle

Then Poisson summation


X X
f (x + n) = fˆ(n)e2πinx ,
n∈Z n∈Z
The periodic func-
tion is the one we get
2. Applications of Poisson summation via a discrete version
2.1. Heat Kernels on the Circle and on the Line. of the Fourier trans-
Recall from Chapter IV: given a function u(x, t) where t ≥ 0, x ∈ [0, 1] describing the form.
temperature distribution on a ring (with initial distribution f (x)), it can be shown that Using Poisson sum-
u must satisfy the following problem: mation formula
2
Recall heat kernel on
 ∂u
∂t
= ∂∂xu2
u(x, 0) = f (x) the circle.
As usual, we look for standing wave solutions: u(x, t) = A(x)B(t), resulting in (an
infinite superposition)
X 2 2
u(x, t) = an e−4π n t e2πinx .
n∈Z

Setting t = 0 shows us that an = fˆ(n).

Definition 2.1. We notate


2 2
X
Ht (x) = e−4π n t e2πinx
n∈Z
and call Ht the heat kernel for the circle. heat kernel on the
circle
Then the solution for the heat equation on [0, 1] with initial data f can be written as
u(x, t) = (f ∗ Ht )(x) (where the convolution is on [0,1]).
Recalling that the heat kernel on the line was given by
1 −x2 /4t ct (ξ) = e−4π2 tξ 2 ,
Ht (x) = e ; i.e., H
(4πt)1/2
we note the following. Heat kernel on the
circle is the peri-
odization of the heat
kernel on the line
2 POISSON SUMMATION FORMULA

Theorem 2.2.
X
Ht (x) = Ht (x + n).
n∈Z

A cool consequence of the above expression:

Corollary 2.3. The heat kernel {Ht } on the circle is an


approximation of the identity (on the circle) as t ↓ 0.
Proof was too hard
to do until now.
Proof. Using uniform convergence, it is immediate that
Z 1/2
Ht (x) dx = 1.
−1/2

Since Ht ≥ 0, the above theorem implies that Ht ≥ 0


(not at all obvious otherwise); so the first two properties of
good kernels are satisfied. It remains to see that given any
η < 1/2,
Z
|Ht (x)| dx → 0 as t → 0.
η<|x|≤ 21

Well, consider:
X
Ht (x) = Ht (x + n)
n∈Z
X
= Ht (x) + Ht (x + n) =: Ht (x) + Et (x).
n∈Z∗

This is so cool...we Since {Ht } is a good kernel, it suffices to show that


can estimate the dif- Z
ference between the
(good) heat kernel |Et (x)| dx → 0
on the line and its |x|≤ 21
periodization.
as t → 0. We shall see that (claim:)
c
|Et (x)| ≤ Ce− t .
POISSON SUMMATION FORMULA 3

For, consider:
1 X −(x+n)2
Et (x) := √ e 4t
4πt n∈Z ∗
C X −cn2
≤√ e t .
t n∈Z∗

since |x| ≤ 21 . Now, for t ∈ (0, 1] (which we can assume) we


see that
n2
 
1 1
≥ + n2
t 2 t
(the n2 /t is greater than either of the terms averaged) and
so
C X − cn2
|Et (x)| ≤ √ e t
t n∈Z∗
C c
X cn2
≤ √ e− 2t e− 2
t n∈Z∗
c
≤ Ce− t .
R
This bound implies the desired control on |x|≤ 1 |Et (x)| dx
2
and thus the third property of good kernels. 

2.2. Poisson kernels on the disc and upper half plane.


Recall the Poisson kernels on the disc and upper half plane: Showing Poisson
kernels related
1 − r2 1 y
Pr (ϑ) = and P y (x) = .
1 − 2r cos ϑ + r2 π y 2 + x2
Corollary 2.4. With r = e2πy ,
X
Pr (2πx) = Py (x + n)
n∈Z
Don’t need to men-
tion this in class.
Proof. Use the Poisson summation formula. 
4 POISSON SUMMATION FORMULA

3. Digression into analytic number theory


(Reference: Whittaker, E.T., and G.N. Watson, A Course
of Modern Analysis: An Introduction to the General Theory
of Infinite Processes and of Analytic Functions; With an
Account of the Principal Transcendental Functions, Cam-
bridge University Press, 1902.)
Definition 3.1. For s > 0, the theta function ϑ(s) is de-
theta function fined

2
X
ϑ(s) := e−πn s
n=−∞
Functional relation:
consequence of Pois-
son summation Theorem 3.2 (Functional relation for ϑ). For s > 0,
 
−1/2 1
s ϑ = ϑ(s).
s
2
Proof. Consider the function f (x) = e−πsx ; its Fourier
transform (exercise) is
πξ 2
1
fˆ(ξ) = s− 2 e− s .
Then, by Poisson summation,
∞ ∞
1 πn2
−πs(x+n)2
X X
e = s− 2 e− s e2πinx .
n=−∞ n=−∞
Evaluating at x = 0 yields the desired relation. 
Another theta func-
tion
Definition 3.3. We define the theta function Θ(z|τ ) for
z ∈ C, =(τ ) > 0 by

2
X
Θ(z|τ ) := eiπn τ e2πinz .
n=−∞

Remarks.
i. Θ(0|is) = ϑ(s).
ii. Θ(x|4πit) = Ht (x)
Digressing even fur-
ther....
POISSON SUMMATION FORMULA 5

Definition 3.4. For s ∈ C such that <(s) > 1, we define


the celebrated Riemann zeta function by The Riemann zeta
∞ function
X 1
ζ(s) =
n=1
ns

It can be shewn that ϑ, ζ, and Γ are related by


Z ∞
1 s
π −s/2 Γ(s/2)ζ(s) = t 2 −1 (ϑ(s) − 1) dt.
2 0
Remark. This will become more relevant later (in your life).
4. The Heisenberg Uncertainty Principle
Remark (Motivation). To what extent can one simultane-
ously locate the position and momentum of a particle?
In quantum mechanics, a particle has associate with it a
state function ψ (of L2 norm 1) which governs the position state function
in the sense that the probability that the particle lies in
Ra particular region (a, b) ∈ R (one-dimensional space) is
2
(a,b) |ψ| . Then the expectation (expected position) is given
by expected position
Z ∞
x := x|ψ(x)|2 dx,
−∞
and the variance (uncertainty of the expectation) is given
by variance of position
Z ∞
(x − x)2 |ψ(x)|2 dx.
−∞
One has an analogous function describing the momentum
of the particle. Importantly, it turns out that the proba-
bility
R of the momentum belonging to an interval (a, b) is
2
(a,b) |ψ̂(ξ)| dξ. We shall now see the Heisenberg Uncer-
tainty Principle, i.e., that Heisenberg Uncer-
tainty Principle
1
Variance of position × Variance of momentum & .
16π 2
6 POISSON SUMMATION FORMULA

Theorem 4.1. Let ψ ∈ S (R), and suppose ||ψ||2 = 1.


Then 
Z ∞  Z ∞ 
1
x2 |ψ(x)|2 dx ξ 2 |ψ̂(ξ)|2 dξ ≥
−∞ −∞ 16π 2
2
−Bx
and
p equality holds iff ψ(x) = Ae where B > 0, A2 =
2B/π.
In fact, we have, for every x0 , ξ0 ∈ R, blahblahblah (with
the individual terms minimized when x0 = x, ξ0 = ξ.
Easy calculation: in- Proof. Integration by parts implies the following.
tegration by parts Z ∞
1= |ψ(x)|2 dx
−∞
Z ∞
d
=− x |ψ(x)|2 dx
dx
Z−∞∞  
0 0
=− xψ (x)ψ(x) + xψ (x)ψ(x) dx.
−∞
Thus Z

1≤2 |x||ψ(x)||ψ 0 (x)| dx
−∞
Z∞ 1/2 Z ∞ 1/2
2 2 0 2
≤2 x |ψ(x)| dx |ψ (x)| dx
−∞ −∞
Z ∞ 1/2  Z ∞ 1/2
=2 x2 |ψ(x)|2 dx 4π 2 ξ 2 |ψ̂(ξ)|2 dξ ,
−∞ −∞
Followed by using the Plancherel theorem (and the basic properties of
Cauchy-Schwartz the Fourier transform) for the equality in the last line. Now,
and Plancherel’s
theorem equality can hold only if equality held in the application of
the Cauchy-Schwartz inequality. which implies that the
functions must be scalar multiples of each other:
ψ 0 (x) = βxψ(x)
for some scalar β. Again, elementary ODE theory implies
2
ψ(x) = Aeβx /2
.
POISSON SUMMATION FORMULA 7

To ensure the function is in S (R), we requirep


β = −2B for
2
some positive B; then ||ψ||2 = 1 forces A = 2B/π. 
LECTURE 22: POISSON SUMMATION FORMULA,
CONTINUED

1. Review from last lecture Connection between


analysis on circle
Recall: and R.
Poisson summation
Theorem 1.1 (Poisson summation formula). Let f ∈ S (R).
Then
X X
f (x + n) = fˆ(n)e2πinx ,
n∈Z n∈Z
The periodic func-
tion is the one we get
Definition 1.2. We notate via a discretization
X 2 2 of the Fourier trans-
Ht (x) = e−4π n t e2πinx form.
n∈Z
and call Ht the heat kernel for the circle. heat kernel on the
circle
Then the solution for the heat equation on [0, 1] with initial
data f can be written as u(x, t) = (f ∗ Ht )(x) (where the
convolution is on [0,1]).
Recalling that the heat kernel on the line was given by
1 2
ct (ξ) = e−4π2 tξ 2 ,
Ht (x) = 1/2
e−x /4t ; i.e., H
(4πt)
we note the following. Heat kernel on the
circle is the peri-
Theorem 1.3. odization of the heat
X kernel on the line
Ht (x) = Ht (x + n).
n∈Z

2. The Heat Kernel on the Line is Good


A cool consequence of the above expression:
Corollary 2.1. The heat kernel {Ht } on the circle is an
approximation of the identity (on the circle) as t ↓ 0.
2 POISSON SUMMATION FORMULA

Proof was too hard


to do until now.
Proof. Using uniform convergence, it is immediate that
Z 1/2
Ht (x) dx = 1.
−1/2
Since Ht ≥ 0, the above theorem implies that Ht ≥ 0
(not at all obvious otherwise); so the first two properties of
good kernels are satisfied. It remains to see that given any
η < 1/2,
Z
|Ht (x)| dx → 0 as t → 0.
η<|x|≤ 21
Well, consider:
X
Ht (x) = Ht (x + n)
n∈Z
X
= Ht (x) + Ht (x + n) =: Ht (x) + Et (x).
n∈Z∗
This is so cool...we Since {Ht } is a good kernel, it suffices to show that
can estimate the dif- Z
ference between the |Et (x)| dx → 0
(good) heat kernel
|x|≤ 21
on the line and its
periodization. as t → 0. We shall see that (claim:)
c
|Et (x)| ≤ Ce− t .
Error here?
[Proof of claim.] Consider:
1 X −(x+n)2
Et (x) := √ e 4t
4πt n∈Z ∗
C X −cn2
≤√ e t .
t n∈Z∗
since |x| ≤ 21 . Now, for t ∈ (0, 1] (which we can assume) we
see that
n2
 
1 1 2
≥ +n
t 2 t
POISSON SUMMATION FORMULA 3

(the n2 /t is greater than either of the terms averaged) and


so
C X − cn2
|Et (x)| ≤ √ e t
t n∈Z∗
C − c X − cn2
≤ √ e 2t e 2
t n∈Z∗
c
≤ Ce− t .
R
This bound implies the desired control on |x|≤ 1 |Et (x)| dx (Seems to be some
2
error here (in
and thus the third property of good kernels.  the last inequal-
ity), but showing
2.1. Poisson kernels on the disc and upper half plane. c
|Et (x)| ≤ √Ct e− 2t
Recall the Poisson kernels on the disc and upper half plane: gives the desired
bound anyway.)
1 − r2 1 y
Pr (ϑ) = and P y (x) = . Showing Poisson
1 − 2r cos ϑ + r2 π y 2 + x2 kernels related

Corollary 2.2. With r = e2πy ,


X
Pr (2πx) = Py (x + n)
n∈Z
Don’t need to men-
tion this in class.
Proof. Use the Poisson summation formula. 

3. Digression into analytic number theory


(Reference: Whittaker, E.T., and G.N. Watson, A Course
of Modern Analysis: An Introduction to the General Theory
of Infinite Processes and of Analytic Functions; With an
Account of the Principal Transcendental Functions, Cam-
bridge University Press, 1902.)
Definition 3.1. For s > 0, the theta function ϑ(s) is de-
fined theta function

2
X
ϑ(s) := e−πn s
n=−∞
Functional relation:
consequence of Pois-
son summation
4 POISSON SUMMATION FORMULA

Theorem 3.2 (Functional relation for ϑ). For s > 0,


 
1
s−1/2 ϑ = ϑ(s).
s
2
Proof. Consider the function f (x) = e−πsx ; its Fourier
transform (exercise) is
πξ 2
1
fˆ(ξ) = s− 2 e− s .
Then, by Poisson summation,
∞ ∞
1 πn2
−πs(x+n)2
X X
e = s− 2 e− s e2πinx .
n=−∞ n=−∞
Evaluating at x = 0 yields the desired relation. 
Another theta func-
tion
Definition 3.3. We define the theta function Θ(z|τ ) for
z ∈ C, =(τ ) > 0 by

2
X
Θ(z|τ ) := eiπn τ e2πinz .
n=−∞

Remarks.
i. Θ(0|is) = ϑ(s).
ii. Θ(x|4πit) = Ht (x)
Digressing even fur-
ther....
Definition 3.4. For s ∈ C such that <(s) > 1, we define
The Riemann zeta the celebrated Riemann zeta function by
function ∞
X 1
ζ(s) =
n=1
ns
It can be shewn that ϑ, ζ, and Γ are related by
Z ∞
1 s
π −s/2 Γ(s/2)ζ(s) = t 2 −1 (ϑ(s) − 1) dt.
2 0
Remark. This will become more relevant later (in your life).
LECTURE 23: HEISENBERG UNCERTAINTY;
BACKGROUND FOR F ON Rd

1. The Heisenberg Uncertainty Principle


Remark (Motivation). To what extent can one simultane-
ously specify the position and momentum of a particle? (Or, phrase in terms
In quantum mechanics, a particle has associated with it a of time-frequency lo-
calization.)
state function ψ (of L2 norm 1) which governs the position state function
in the sense that the probability that the particle lies in
Ra particular region (a, b) ∈ R (one-dimensional space) is
2
(a,b) |ψ| . Then the expectation (expected position) is given
by expected position
Z ∞
x := x|ψ(x)|2 dx,
−∞
and the variance (uncertainty of the expectation) is given
by variance of position
Z ∞ from the expected
(x − x)2 |ψ(x)|2 dx. position
−∞
One has an analogous function describing the momentum
of the particle. Importantly, it turns out that the proba-
bility
R of the momentum belonging to an interval (a, b) is
2
(a,b) |ψ̂(ξ)| dξ. We shall now see the Heisenberg Uncer-
tainty Principle, i.e., that Heisenberg Uncer-
tainty Principle
1
Variance of position × Variance of momentum & .
16π 2
Theorem 1.1. Let ψ ∈ S (R), and suppose ||ψ||2 = 1.
Then
Z ∞  Z ∞ 
1
x2 |ψ(x)|2 dx ξ 2 |ψ̂(ξ)|2 dξ ≥
−∞ −∞ 16π 2
2 HEISENBERG UNCERTAINTY; F ON RD
2
−Bx
and
p equality holds iff ψ(x) = Ae where B > 0, A2 =
2B/π.
In fact, we have, for every x0 , ξ0 ∈ R,
Z ∞  Z ∞ 
1
(x − x0 )2 |ψ(x)|2 dx (ξ − ξ0 )2 |ψ̂(ξ)|2 dξ & 2
,
−∞ −∞ 16π
with the individual terms (and, subsequently, the product)
minimized when x0 = x, ξ0 = ξ.
Easy calculation: in- Proof. Integration by parts implies the following.
tegration by parts Z ∞
1= |ψ(x)|2 dx
−∞
Z ∞
d
=− x |ψ(x)|2 dx
dx
Z−∞∞  
0 0
=− xψ (x)ψ(x) + xψ (x)ψ(x) dx.
−∞
Thus
Z ∞
1≤2 |x||ψ(x)||ψ 0 (x)| dx
−∞
Z∞ 1/2 Z ∞ 1/2
≤2 x2 |ψ(x)|2 dx |ψ 0 (x)|2 dx
−∞ −∞
Z ∞ 1/2  Z ∞ 1/2
2 2 2 2 2
=2 x |ψ(x)| dx 4π ξ |ψ̂(ξ)| dξ ,
−∞ −∞
Followed by using the Plancherel theorem (and the basic properties of
Cauchy-Schwartz
and Plancherel’s
the Fourier transform) for the equality in the last line. Now,
theorem equality can hold only if equality held in the application of
the Cauchy-Schwartz inequality. which implies that the
functions must be scalar multiples of each other:
ψ 0 (x) = βxψ(x)
for some scalar β. Again, elementary ODE theory implies
2
ψ(x) = Aeβx /2
.
HEISENBERG UNCERTAINTY; F ON Rd 3

To ensure the function is in S (R), we requirep


β = −2B for
2
some positive B; then ||ψ||2 = 1 forces A = 2B/π.
To get the second part of the theorem, apply the first to
−2πixξ0
e ψ(x + x0 ). 
2. Fourier Transform on Rd : Background
Remark. Not much content in this section: basically just
a re-hash of the one-dimensional theory. Only thing of
interest is to understand the details of how one makes that
extension.
Basic terms
d
Let x = (x1 , · · · , xd ) denote a vector in R ; let |x| denote
its magnitude, and x · y denote the standard inner product
on Rn . multi-index

Definition 2.1. Let x ∈ Rd , and let α ∈ Zd be a multi-


index, i.e., a d-tuple of non-negative integers. We notate
xα := xα1 1 xα2 2 · · · xαd d .
and  α  α1  α2  αd
∂ ∂ ∂ ∂
:= ···
∂x ∂x1 ∂x2 ∂xd
∂ |α|
= α1
∂x1 · · · ∂xαd d
rotations

Definition 2.2. Let R : Rd → Rd be a linear transforma-


tion. If
R(x) · R(y) = x · y for all x, y ∈ Rd
(or, equivalently: if Rt = R−1 ) then we call R a rotation.
Of course | det(R)| = 1; if det(R) = 1, we say R is a proper
rotation; otherwise is it called improper.
Remark. Given any orthonormal basis {e1 , . . . , ed }, then
{R(e1 ), . . . , R(ed )} is another orthonormal basis. Conversely,
given any two orthonormal bases {ei }, {e0i }, we can define
a rotation R by letting R(ei ) = e0i .
4 HEISENBERG UNCERTAINTY; F ON RD

Rapidly decreasing Definition 2.3. Let f : Rd → C. If


functions
i. f is continuous, and
ii. for all α, |xα f (x)| is bounded,
then f is called rapidly decreasing. (If |x|α |f (x)| is bounded
for α = d + 1, we call the function of moderate decrease.)
Definition 2.4. Given f a function of rapid (or merely
Integral on R d
moderate) decrease, we define
Z Z
f = lim f (x) dx.
Rd N →∞ QN
Polar coordinate in-
tegration on Rd 2.1. Polar coordinates.
2.1.1. R2 .
Recall: one can, using a change to polar coordinates,
express the integral of a function over the plane as
Z Z 2π Z ∞
f (x) dx = f (r cos θ, r sin θ) r dr dθ.
R2 0 0
bogus With the notation
Z Z 2π
g(γ) dσ(γ) := g(cos θ, sin θ) dθ,
S1 0
we can rewrite the above as
Z Z Z ∞
f (x) dx = f (rγ)r dr dσ(γ)
R2 S1 0

2.1.2. R3 .
In R3 , recall, using spherical coordinates, we can do a
change-of-variables to show that
Z Z 2π Z π Z ∞
f= f (r sin θ cos φ, r sin θ sin φ, r cos θ)r2 dr sin θ dθ dφ.
R3 0 0 0
As above, if we abbreviate (or more honestly, define)
Z Z 2 Z π
g(γ) dσ(γ) = π g(sin θ cos φ, sin θ sin φ, cos θ) sin θ dθ dφ,
S2 0 0
HEISENBERG UNCERTAINTY; F ON Rd 5

then the formula immediately above can be more concisely


expressed as
Z Z Z ∞
f= f (rγ)r2 dr dσ(γ).
R3 S2 0

2.1.3. Rd . General formula


In general, one has the following formula:
Z Z Z ∞
f= f (rγ)rd−1 drdσ(γ).
Rd S d−1 0

3. Elementary Theory of the Fourier


Transform in Rd
Definition 3.1. Let f : Rd → C be an infinitely differ-
entiable function. If, for each pair of multi-indices α and
β,
 
β
α ∂

sup x f (x) < ∞

x∈Rd ∂x
then we say that f ∈ S (Rd ).
Definition 3.2. Given f ∈ S (Rd ), we define its Fourier
transform fˆ : Rd → C by
Z
fˆ(ξ) := f (x)e−2πix·ξ dx.
Rd

Proposition 3.3. Let f ∈ S (Rd ), h ∈ Rd , and δ > 0.


Then
i. f (x + h) → fˆ(ξ)e2πiξ·h
ii. f (x)e−2πix·h → ˆ
 f(x + h) Just as in the
one-dimensional
iii. f (δx) → δ1d fˆ ξδ case. Interchange
∂ α
 αˆ of differentiation
iv. ∂x f (x) → (2πiξ) fα(ξ) and multiplication
by corresponding
v. (−2πix)α f (x) → ∂ξ ∂
fˆ(ξ) monomials, etc.

vi. f (Rx) → fˆ(Rξ) for all rotations R.


6 HEISENBERG UNCERTAINTY; F ON RD

Proof of (vi). A calculation:


Z
F [f (Rx)](ξ) := f (R(x))e−2πix·ξ dx
Z Rd
−1
= f (y)e−2πi(R y)·ξ |det(R−1 )|dy (let y = Rx)
ZRd
= f (y)e−2πiy·(Rξ) dy = fˆ(Rξ), as desired.
Rd


As before, we have the following:
Corollary 3.4. F maps S (Rd ) to itself.
3.1. Slight digression: Radial functions.
Definition 3.5. A function f on Rd is called radial if its
value is constant on spheres about the origin, i.e., there
exists f0 : R≥0 → C such that f (x) = f0 (|x|).
Remark. Obviously, f is radial ⇐⇒ f (Rx) = f (x) for all
rotations R.
Corollary 3.6. If f is radial, then fˆ is also.
Proof. WTS fˆ(Rξ) = fˆ(ξ) for all rotations R. By the
above,
fˆ(Rξ) = F [f (Rx)](ξ) = F (f )(ξ) = fˆ(ξ).

3.2. Fourier inversion and Plancherel on Rd .
Theorem 3.7. Let f ∈ S (Rd ). Then the Fourier inver-
sion formula
Z
f (x) = fˆ(ξ)e2πix·ξ dξ
Rd
holds, as does the Plancherel theorem, ||f ||2 = ||fˆ||2 .
HEISENBERG UNCERTAINTY; F ON Rd 7

Proof. The proof is basically analogous to that of the one-


dimensional case:
i. First one shows (via iteration) that the d-dimensional
Gaussian is an eigenfunction of F , i.e.,
2 2
F (e−π|x| )(ξ) = e−π|ξ| .
ii. Using the interaction of dilation and F , we next see
that
2 1 π|ξ|2
F[e−πδ|x| ](ξ) = d/2 e− δ .
δ
π|x|2
1 − δ
iii. Then, letting Kδ (x) = δd/2 e , one shows that {Kδ }
is an approximation of the identity.
iv. Multiplication formula: for f, g ∈ S (Rd ),
Z Z
f ĝ = fˆg
Rd Rd
(proof obtained via Fubini’s theorem exactly as in the
one-dimensional result).
Once one has (iii) and (iv) above, the proof of the inversion
formula follows exactly as in the one-dimensional case; once
one has (iv) and the inversion formula, one can analogously
play the “hats game” to get Plancherel’s theorem (or go via
convolutions). 
Remark. Not all that interesting. However, what we will
explore next, the wave equation in Rd × R and how the
theory differs in odd and even dimensions, is quite subtle.
LECTURE 24:
THE WAVE EQUATION ON Rd × R

1. Radial functions and the Fourier transform


Definition 1.1. A function f on Rd is called radial if its
value is constant on spheres about the origin, i.e., there
exists f0 : R≥0 → C such that f (x) = f0 (|x|).
Radial functions

Remark. Obviously, f is radial ⇐⇒ f (Rx) = f (x) for all


rotations R.
Corollary 1.2. If f is radial, then fˆ is also.
Fourier transform of
radial function is ra-
Proof. WTS fˆ(Rξ) = fˆ(ξ) for all rotations R. Well, dial.
Z
fˆ(Rξ) = f (x)e−2πiRξ·x dx
ZRd
−1
= f (x)e−2πiξ·R x dx
ZRd
= f (Ry)e−2πiξ·y |det(R−1 )| dy
ZRd
= f (y)e−2πiξ·y dy = fˆ(ξ).
Rd


2. The Wave Equation


Definition 2.1. We define the d-dimensional Laplacian by
Laplacian
∂2 ∂2
∆= + · · · + .
∂x21 ∂x2d
2 LECTURE 24: THE WAVE EQUATION ON RD × R
Definition 2.2. The Cauchy problem for the wave equation
is the following: The Cauchy problem
 2 for the wave equa-
 ∆u = ∂∂tu2 tion
u(x, 0) = f (x)
 ∂u
∂t (x, 0) = g(x)

As usual, we first run a formal argument. Taking the


Fourier transform in the first variable, we see that the wave
The formal ar- equation becomes
gument to find a
solution: 2 2 2 ∂ 2 û
−4π (ξ1 + · · · + ξd )û(ξ, t) = 2 (ξ, t)
∂t
where in the above theˆindicates the Fourier transform in
the first variable. For each fixed ξ, then, one obtains an
ordinary differential equation in t with solution
û(ξ, t) = A(ξ) cos(2π|ξ|t) + B(ξ) sin(2π|ξ|t).
As usual, taking the Fourier transforms of the initial con-
Fourier transform of ditions yields
initial conditions
û(ξ, 0) = fˆ(ξ)

∂ û
∂t (ξ, 0) = ĝ(ξ);
then, of course
A(ξ) = fˆ(ξ) and 2π|ξ|B(ξ) = ĝ(ξ).
The guessed solu- Thus the solution is expected to be
tion.
ˆ
g(ξ)
ˆ
û(ξ, t) = f (ξ) cos(2π|ξ|t) + sin(2π|ξ|t);
2π|ξ|
and, in fact, it is:
Theorem 2.3. Given the Cauchy problem for the wave
It actually is a solu- equation
tion.  2
 ∆u = ∂∂tu2
u(x, 0) = f (x)
 ∂u
∂t (x, 0) = g(x),
LECTURE 24: THE WAVE EQUATION ON Rd × R 3
the following is a solution:
Z  
ĝ(ξ)
u(x, t) := fˆ(ξ) cos(2π|ξ|t) + sin(2π|ξ|t) e2πix·ξ dξ.
Rd 2π|ξ|
Proof. Differentiate under the integral signs. First, differ-
Proof is an exer- entiating with respect to x1 , . . . , xd , we get
cise in differentiation !
(using the fact that
Z   d
ĝ(ξ) X
we can differentiate ∆u(x, t) = fˆ(ξ) cos(2π|ξ|t) + sin(2π|ξ|t) (2πi)2 ξk2 e2πix·ξ dξ.
under the integral Rd 2π|ξ|
k=1
sign.) Z  
ĝ(ξ)
= fˆ(ξ) cos(2π|ξ|t) + sin(2π|ξ|t) (−4π 2 i|ξ|2 )e2πix·ξ dξ.
Rd 2π|ξ|
Diffeentiating u(x, t) with respect to t also yields the same
thing, obviously; so our guessed solution does solve the
wave equation. Checking the initial conditions is equally
clear:
Z
u(x, 0) := fˆ(ξ)e2πix·ξ dξ = f (x)
Rd
by Fourier inversion. Similarly,
Z  
∂u ĝ(ξ)
(x, t) = fˆ(ξ)(−2π|ξ|) sin(2π|ξ|t) + (2π|ξ|) cos(2π|ξ|t) e2πix·ξ dξ;
∂t Rd 2π|ξ|
evaluating at t = 0 shows the initial velocity condition
∂u
(x, 0) = g(x)
∂t
is also satisfied. 
Remark. In fact, the solution is unique. This fact can be
shown via a conservation of energy argument.
Uniqueness of solu-
tion is a conservation
of energy argument.
3. Conservation of Energy
Definition 3.1. Let u be a solution of the wave equation.
We define the (total = kinetic + potential)) energy of the
4 LECTURE 24: THE WAVE EQUATION ON RD × R
solution as
2
∂u 2 ∂u 2
Z
∂u
E(t) := ∂x1 + · · · + ∂xd dx.
+
d
∂t
R
Definition of energy
of a solution
Lemma 3.2. Let a, b ∈ C; let α ∈ R. Then
|a cos α + b sin α|2 + | − a sin α + b cos α|2 = |a|2 + |b|2
Proof. (cos α, sin α) and (− sin α, cos α) are orthonormal in
Pythagorean C2 ; so by the Pythagorean Theorem,
theorem again.
|(a, b) · (cos α, sin α)|2 + |(a, b) · (− sin α, cos α)|2 = |(a, b)|2 .

Theorem 3.3. For the aforementioned solution u of the
wave equation, E(t) is constant.
Proof. Recall
ĝ(ξ)
û(ξ, t) = fˆ(ξ) cos(2π|ξ|t) + sin(2π|ξ|t);
2π|ξ|
thus, using Plancherel’s theorem,
d
∂u 2
Z X Z 2
dx = ˆ
2π|ξ|f (ξ) cos(2π|ξ|t) + ĝ(ξ) sin(2π|ξ|t) dξ.

d

R j=1 ∂xj d R

Note: first deriva- and


tives Z 2 Z
∂u 2
dx = ˆ
−2π|ξ|f (ξ) sin(2π|ξ|t) + ĝ(ξ) cos(2π|ξ|t) dξ.

Rd ∂t

Rd

Then, by the lemma (a = 2π|ξ|fˆ(ξ), b = ĝ(ξ)), we get


Z 2
∂u 2 ∂u 2

∂u
E(t) = + + ··· + dx
d
∂t ∂x1 ∂xd
ZR
= (4π 2 |ξ|2 |fˆ(ξ)|2 + |ĝ(ξ)|2 ) dξ,
Rd
which is independent of t. 
LECTURE 24: THE WAVE EQUATION ON Rd × R 5

4. Wave Equation in R3 × R
motivation Recall d’Alembert’s solution to the wave equation:
u(x + t) + u(x − t) 1 x+t
Z
u(x, t) = + g(y) dy.
2 2 x−t
“This suggests a generalization to higher dimensions, where
we might expect to write the solution of our problem as
averages of the initial data.”
Definition 4.1. Let f : R3 → C. We define the spherical
mean of f at x with radius t by spherical mean: first
example of a convo-
Z lution of a function
1 with a measure.
Mt (f )(x) = f (x − tγ)dσ(γ),
4π S 2
that is, the average of f over the sphere of radius t centered
at x.
Some useful lemmata:
Lemma 4.2. Let f ∈ S(R3 ). Then for each fixed t, Mt (f ) ∈
S(R3 ). Further, Mt (f ) is infinitely differentiable in t, and
each t-derivative is in S(R3 ).
Lemma 4.3 (Fourier transform of the surface measure).
Z
1 sin(2π|ξ|)
e−2πiξ·γ dσ(γ) = .
4π S 2 2π|ξ|
Remark. Denote the LHS by dσ(ξ).
c We notice that (it is
the Fourier transform of a radial “function.”): the above
lemma shows that it is then radial.
Proof. We first observe the formula is true for ξ = (0, 0, ρ)
for any ρ > 0 (ρ = 0 is immediate); then we show the left
hand side is a radial function. By definition,
Z Z 2π Z π
1 −2πiξ·γ 1
e dσ(γ) := e−2πiξ·γ sin θ dθ dφ
4π S 2 4π 0 0
6 LECTURE 24: THE WAVE EQUATION ON RD × R
(where γ = (sin θ cos φ, sin θ sin φ, cos θ) in the right-hand
side).
Z 2π Z π
1
= e−2πiρ cos θ sin θ dθ dφ
4π 0 0
Z π
1
= e−2πiρ cos θ sin θ dθ
2 0
1 1 2πiρu
Z
= e du (C.O.V.: u = − cos θ)
2 −1
1  2πiρu 1 sin(2πρ)
= e −1
= ;
4πiρ 2πρ
so the lemma is proven for ξ = (0, 0, ρ) for ρ ≥ 0.
(To be continued....) 
LECTURE 25:
THE WAVE EQUATION ON R3 × R

In order to solve the initial value for the wave equation in di-
mensions bigger than one, we shall use the method of spher-
ical means, due to Hadamard. The intuitive grounds for this
methodology lies in our conception of waves as being made
of a superposition of spherically symmetric fronts emanating
from point sources. Huygens was a pioneer of this view of
waves, which he used to show that the laws of optics sug-
gested that light was a wave phenomenon.
(http://www.math.nyu.edu/faculty/tabak/PDEs/WE.pdf)

1. Wave Equation in R3 × R
Recall d’Alembert’s solution to the wave equation: motivation

u(x + t) + u(x − t) 1 x+t


Z
u(x, t) = + g(y) dy.
2 2 x−t
“This suggests a generalization to higher dimensions, where
we might expect to write the solution of our problem as
averages of the initial data.”
Definition 1.1. Let f : R3 → C. We define the spherical
mean of f at x with radius t by spherical mean: first
example of a convo-
Z lution of a function
1 with a measure.
Mt (f )(x) = f (x − tγ)dσ(γ),
4π S 2
that is, the average of f over the sphere of radius t centered
at x.
Some useful lemmata:
Lemma 1.2. Let f ∈ S(R3 ). Then for each fixed t, Mt (f ) ∈
S(R3 ). Further, Mt (f ) is infinitely differentiable in t, and
each t-derivative is in S(R3 ).
2 LECTURE 25: THE WAVE EQUATION ON R3 × R
Lemma 1.3 (Fourier transform of the surface measure).
Z
1 sin(2π|ξ|)
e−2πiξ·γ dσ(γ) = .
4π S 2 2π|ξ|

Remark. Denote the LHS by dσ(ξ).


c We notice that (it is
the Fourier transform of a radial “function.”): the above
lemma shows that it is then radial.

Proof. We first observe the formula is true for ξ = (0, 0, ρ)


for any ρ > 0 (ρ = 0 is immediate); then we show the left
Formula is true for a hand side is a radial function. By definition,
single ξ = (0, 0, ρ). Z Z 2π Z π
1 1
e−2πiξ·γ dσ(γ) := e−2πiξ·γ sin θ dθ dφ
4π S 2 4π 0 0

(where γ = (sin θ cos φ, sin θ sin φ, cos θ) in the right-hand


side).
Z 2π Z π
1
= e−2πiρ cos θ sin θ dθ dφ
4π 0 0
1 π −2πiρ cos θ
Z
= e sin θ dθ
2 0
1 1 2πiρu
Z
= e du (C.O.V.: u = − cos θ)
2 −1
1  2πiρu 1 sin(2πρ)
= e −1
= ;
4πiρ 2πρ
so the lemma is proven for ξ = (0, 0, ρ) for ρ ≥ 0.
START FROM HERE
At this point, it suffices to show that the Fourier trans-
Both sides of the form of the surface measure is a radial function. We recall
equation are radial the fact (see last page of appendix) that given any function
functions; so proving
it for a single vector f of moderate decrease,
proves it for the en- Z Z
tire sphere.
f (R(γ))dσ(γ) = f (γ) dσ(γ).
S d−1 S d−1
LECTURE 25: THE WAVE EQUATION ON R3 × R 3

Then, given any rotation R,


Z
1
dσ(Rξ)
c := e−2πi(Rξ)·γ dσ(γ)
4πZ S 2
1 −1
= e−2πiξ·R γ dσ(γ)
4π ZS 2
1
= e−2πiξ·γ dσ(γ) = dσ(ξ).
c
4π S 2

The next lemma basically says that the Fourier transform
of the spherical averaging operator (which, for t = 1 is
convolution f ∗ dσ) is the product of Fourier transforms.
(fˆ(ξ)dσ(ξt))
c

Lemma 1.4.
\
M ˆ sin(2π|ξ|t) .
t (f )(ξ) = f (ξ)
2π|ξ|t
Fourier transform of
the spherical averag-
Proof. By definition, ing operator: calcu-
lation using the pre-
Z  Z  vious lemma
1
\
M t (f )(ξ) := e−2πix·ξ f (x − γt) dσ(γ) dx
R3 4π S2
Z Z 
1 −2πix·ξ
= f (x − γt)e dx dσ(γ)
4π S 2 R3
Z Z 
1
= f (y)e−2πi(y+γt)·ξ dy dσ(γ) (let y = x − γt)
4π S 2 R3
Z
1
= fˆ(ξ) e−2πi(γt)·ξ dσ(γ)
4π S 2
Z
1 sin(2π|ξ|t)
= fˆ(ξ) e−2πiγ·tξ dσ(γ) = fˆ(ξ)
4π S 2 2π|ξ|t
by the previous lemma. 
Once we have the above lemmata, the solution becomes
clear:
4 LECTURE 25: THE WAVE EQUATION ON R3 × R
Theorem 1.5. In R3 × R, the solution to the Cauchy prob-
The explicit formula lem for the wave equation is
of the solution

u(x, t) = (tMt (f )(x)) + tMt (g)(x)
∂t
Easy once we have
the above lemmata Proof. We break the problem into two subproblems: the
case when g = 0, and the case when f = 0. Using the
Fourier inversion expression of the solution that we worked
Calculus I trick out before, we see in the first case,
Z h i
u(x, t) = fˆ(ξ) cos(2π|ξ|t) e2πix·ξ dξ
R3
Z   
∂ sin(2π|ξ|t)
= t fˆ(ξ) e2πix·ξ

∂t R 3 2π|ξ|t

= (tMt (f )(x)),
∂t
Elementary school and in the second case,
arithmetic trick Z  
sin(2π|ξ|t) 2πix·ξ
u(x, t) = ĝ(ξ) e dξ
R3 2π|ξ|
Z  
sin(2π|ξ|t) 2πix·ξ
=t ĝ(ξ) e dξ
R 3 2π|ξ|t
= tMt (g)(x).
The solution to the general problem (for general f, g ∈
S(R3 )) is then the superposition of these two cases. 
2. Cool observation about the solution:
Huygen’s Principle
Considering the form of the solution given above, i.e.,
that

u(x, t) = (tMt (f )(x)) + tMt (g)(x)
∂t
Huygen’s Principle we see that the solution at (x, t) depends on the averages of
f and g (that is, data on the boundary t = 0) over spheres
(in R3 ) centered at x of radius t; equivalently, “the data at
a point x0 in the plane t = 0 influences the solution on the
LECTURE 25: THE WAVE EQUATION ON R3 × R 5
boundary of a forward light cone originating at x0 . (See
http://en.wikipedia.org/wiki/Huygens-Fresnel principle)

3. The Wave Equation in R2 × R:


Hadamard’s Method of Descent
Definition 3.1. We define, for F : R2 → C, a weighted
ft (F )(x) over the disk of radius t centered at x ∈
average M
2
R by Relevant weighted
Z averaging operator
1 1
M
ft (F )(x) := F (x − ty) dy.
2π| |y|≤1 (1 − |y|2 )1/2

Theorem 3.2. Let f, g ∈ S(R2 ) be initial data for the


Cauchy problem for the wave equation on R2 × R. Then
a solution is
∂ f
u(x, t) = (tMt (f )(x)) + tMt (g)(x).
f
∂t
Proof. We use f, g ∈ S(R2 ) to create a Cauchy problem for
the wave equation in R3 × R as follows. Fix some T > 0, We turn the
and let η ∈ S(R) be a (bump) function such that two-dimensional
problem into a
three-dimensional
η(x) = 1 whenever |x| ≤ 3T. one.
We create f [ , g [ ∈ S(R3 ) by defining
f [ (x1 , x2 , x3 ) := f (x1 , x2 )η(x3 )
g [ (x1 , x2 , x3 ) := g(x1 , x2 )η(x3 ).
Let, now, u[ be the solution to the Cauchy problem for
the wave equation on R3 × R with initial data f [ , g [ . By
Huygen’s principle, we see that for |t| ≤ T , u[ (x, t) is con-
stant in x3 for all |x3 | ≤ T : after all, the backwards light
cone for such (x, t) is contained in R2 × [−3T, 3T ], over
which region the initial data f [ , g [ is constant in x3 . (Draw
picture: the solution depends on data which is constant in
x3 .) We already can solve
the R3 × R equation;
ignoring the third
variable (it’s con-
stant in that vari-
able) gives the R2 ×
R solution.
6 LECTURE 25: THE WAVE EQUATION ON R3 × R
Now, define
u(x1 , x2 , t) := u[ (x1 , x2 , 0, t).
u solves the 2-dimensional Cauchy problem for |t| < T ; let’s
call it uT . Notice now that if we take a T2 > T1 , then uT2
agrees with uT1 for |t| < T1 . Since T was arbitrary, we thus
obtain a well-defined solution u(x1 , x2 , t) for all t > 0.
Now we need to show that the solution actually has the
Now we need to desired form.
show that the
spherical averages Lemma 3.3. Let H be a function on the sphere S 2 . If there
of the “extended to
R3 data” are the
exists some two-variable function h such that H(x1 , x2 , x3 ) =
weighted averages of h(x1 , x2 ), then
the original R2 data.
Mt (H)(x1 , x2 , 0) = M
ft (h)(x1 , x2 ).
Proof of lemma. By definition, Mt (H)(x1 , x2 , 0)
Z 2π Z π
1
= h(x1 − t sin θ cos φ, x2 − t sin θ sin φ) sin θ dθ dφ
4π 0 0
Z 2π Z π/2
1
= h(x1 − t sin θ cos φ, x2 − t sin θ sin φ) sin θ dθ dφ
4π 0 0
Z 2π Z π
1
+ h(x1 − t sin θ cos φ, x2 − t sin θ sin φ) sin θ dθ dφ
4π 0 π/2
Calculus II Letting r = sin θ, we get
Z 2π Z π/2
1
= h(x1 − t sin θ cos φ, x2 − t sin θ sin φ) sin θ dθ dφ
4π 0
Z 2π Z 0π
1
+ h(x1 − t sin θ cos φ, x2 − t sin θ sin φ) sin θ dθ dφ
4π 0 π/2
Z 2π Z 1
1 1
= h(x1 − tr cos φ, x2 − tr sin φ) √ rdr dφ
2π 0 0 1 − r 2
Z
1 1
= h(x − ty) p dy = Mft (h)(x)
2π |y|≤1 1 − |y| 2

as desired (taking y = (r cos φ, r sin φ) in the last change of


variables). 
LECTURE 25: THE WAVE EQUATION ON R3 × R 7

Once we have the above lemma, since


u(x1 , x2 , t) := u[ (x1 , x2 , 0, t)

= (tMt (f [ )(x1 , x2 , 0) + tMt (g [ )(x1 , x2 , 0)
∂t
∂ f
= (tM t (f )(x1 , x2 ) + tMt (g(x1 , x2 ),
f
∂t

our solution indeed has the form that we claimed. 


4. Comments about the solutions Difference between
odd and even
Since the propagation of light is governed by the dimensions
three-dimensional wave equation, if at t = 0 a
point of light flashes at the origin, after a finite
amount of time an observer will see the flash
only for an instant. However, if we drop a stone
in a lake, after a finite amount of time any point
on the surface will begin to undulate and will
continue to do so indefinitely (in principle).
LECTURE 26: RADIAL SYMMETRY, THE FOURIER
TRANSFORM, AND BESSEL FUNCTIONS; THE
RADON TRANSFORM

1. f0 and F0 , Bessel Functions, and Parity


Seems sort of a di-
d
Question. Recall: if f is a radial function on R (i.e., f (x) = gression for now....

f0 (|x|), then so is fˆ (fˆ(ξ) = F0 (ξ)). What is the relation


between f0 and F0 ?
1.1. Case R. In one dimension, “radial” is the same as
even; so let |ξ| = ρ and consider: A trivial calculation
Z ∞ using the evenness of
F0 (ρ) := fˆ(ξ) := f (x)e−2πix|ξ| dx the function.
−∞
Z ∞  
−2πir|ξ| 2πir|ξ|
= f0 (r) e +e dr
0Z

=2 cos(2πρr)f0 (r) dr
0

1.2. Case R3 . In R3 , we (obviously) use the polar integra-


tion formula, and then the formula for the Fourier trans-
2
form of the surface element
Z on S .
F0 (ρ) = fˆ(ξ) := f (x)e−2πix·ξ dx
Z ∞ RZ3

= f0 (r) e−2πirγ·ξ dσ(γ)r2 dr


Z0 ∞ S2
2
= f0 (r)4π dσ(rξ)r
c dr
0
Z ∞
4π sin(2πρr) 2
= f0 (r) r dr
0Z 2πρr
2 ∞
= sin(2πρr)f0 (r)r dr.
ρ 0
1
2 RADIAL SYMMETRY; RADON TRANSFORM

1.3. Case R2 .
Definition 1.1. For each n ∈ Z, let the nth Bessel function
Bessel function: you Jn (ρ) denote the nth Fourier coefficient of eiρ sin θ ; that is,
might have seen Z 2π
them in ODEs 1
Jn (ρ) = eiρ sin θ e−inθ dθ, or
2π 0
X∞
iρ sin θ
e = Jn (ρ)einθ .
n=−∞

Polar coordinates: Then we have the following relation:


x = (r cos θ, r sin θ) Z
ˆ
f (ξ) = f (x)e−2πix·(0,−ρ) dx
2
ZR2π Z ∞
= f0 (r)e2πirρ sin θ r dr dθ
0 Z 0

= 2π J0 (2πrρ)f0 (r)r dr.
0
Fubini’s theorem In general, the relation between f0 and F0 involves the
Bessel function of order d2 − 1 (one needs a more general
definition in the odd dimensions for the Bessel functions of
fractional order).

2. The Radon Transform


2.1. Various forms of the Radon Transform.
Definition 2.1. Let ρ be a function on R2 . For each line
The so-called X-ray L ⊂ R2 , we define the Radon transform X of ρ on L by
transform Z
X(ρ)(L) := ρ.
L

Definition 2.2. Let G2,3 denote the Grassmannian (mani-


fold) of two-dimensional affine planes in R3 .
Definition 2.3. Let f be a function on R3 (f ∈ S(R3 ),
The Radon Trans- say). We define the Radon transform R(f ) on G2,3 by
form in R3
RADIAL SYMMETRY; RADON TRANSFORM 3

Z
R(f )(P) = f
P

Remark. Usually Radon transform means integration over


planes of co-dimension 1. One uses the term k-plane Radon
transform for integrals over k-planes, and the term X-ray
transform for integrals over lines.

2.2. Calculation of the Radon Transform in R3 .

Remark. One can parametrize the elements ot G2,3 as fol-


lows: for γ ∈ S 2 and t ∈ R, let Pt,γ denote the plane
{x ∈ R3 : x · γ = t}. I.e., Pt,γ is the plane orthogonal to γ
and passing through (the terminal point of) tγ. Note that Pt,γ =
P−t,−γ .

Definition 2.4. Given f ∈ S(R3 ), we define its integral


over Pt,γ by
Z Z
f := f (tγ + u1 e1 + u2 e2 ) du1 du2
Pt,γ R2

where {γ, e1 , e2 } is an orthonormal basis of R3 . That is, {e1 , e2 } is


an ONB of the plane
P0,γ .
Proposition
R 2.5. Let f ∈ S(R3 ). Then the above defini-
tion of Pt,γ f is independent of the choice of e1 and e2 .

Proof. Trivial. 

Lemma 2.6.
!
Z ∞ Z Z
f dt = f (x) dx
−∞ Pt,γ R3

Proof. Just a calculation. Let R be the rotation taking the


standard basis vectors of R3 to γ, e1 , e2 . Then Intuitively obvious
4 RADIAL SYMMETRY; RADON TRANSFORM

Z Z
f (x) dx = f (Rx) dx
R3 R3
Z
= f (x1 γ + x2 e1 + x3 e2 ) dx1 dx2 dx3
R3 !
Z ∞ Z
= f dt.
−∞ Pt,γ


Definition 2.7 (Alternate definition of Radon Transform).
Let f ∈ S(R3 ). Then for (t, γ) ∈ R × S 2 we define the
Radon Transform of f by
Z
R(f )(t, γ) := f.
Pt,γ
Again, we note
that R(f )(t, γ) =
R(f )(−t, −γ); that We shall need an appropriately-defined Schwartz space.
is, really
Definition 2.8. Let F be a continuous function on R2 ×S 2
Relevant Schwartz that is infinitely differentiable in t. If
class S(R × S 2 ) `
k
∂ F
sup |t| ` (t, γ) < ∞
t∈R,γ∈S 2 ∂t
for all nonnegative k, ` ∈ Z (i.e., F (·, γ) is in S(R) uni-
formly in γ) then we say F ∈ S(R × S 2 ).
Lemma 2.9 (Fourier Slice, or Projection Theorem). If f ∈
THE key lemma: S(R3 ), then R(f )(t, γ) ∈ S(R) for each γ, and
The Fourier Slice
theorem b )(s, γ) = fˆ(sγ)
R(f
where ˆ denotes the one-dimensional Fourier transform in
the first variable in the LHS, and the three-dimensional
Fourier transform on the RHS.
Remark. In other words, if you project f onto the line {tγ}
and then take the 1-D Fourier transform, it’s the same as
taking the slice of the 2-D Fourier transform parallel to γ.
LECTURE 27: FOURIER SLICE THEOREM AND
RADON INVERSION FORMULA

1. Fourier Slice Theorem


Recall:
Definition 1.1 (Definition of Radon Transform). Let f ∈
S(R3 ). Then for (t, γ) ∈ R×S 2 we define the Radon Trans-
form of f by
Z
R(f )(t, γ) := f.
Pt,γ
Again, we note
that R(f )(t, γ) =
We shall need an appropriately-defined Schwartz space. R(f )(−t, −γ); that
is, really
Definition 1.2. Let F be a continuous function on R2 ×S 2
that is infinitely differentiable in t. If Relevant Schwartz
` class S(R × S 2 )
k ∂ F

sup |t| ` (t, γ) < ∞
t∈R,γ∈S 2 ∂t
for all nonnegative k, ` ∈ Z (i.e., F (·, γ) is in S(R) uni-
formly in γ) then we say F ∈ S(R × S 2 ).
Lemma 1.3 (Fourier Slice, or Projection Theorem). If f ∈
S(R3 ), then R(f )(t, γ) ∈ S(R) for each γ, and THE key lemma:
The Fourier Slice
b )(s, γ) = fˆ(sγ)
R(f theorem

where ˆ denotes the one-dimensional Fourier transform in


the first variable in the LHS, and the three-dimensional
Fourier transform on the RHS.
Remark. In other words, if you project f onto the line {tγ}
and then take the 1-D Fourier transform, it’s the same as
taking the slice of the 3-D Fourier transform parallel to γ.
1
2 FOURIER SLICE THEOREM AND RADON TRANSFORM INVERSION

Proof of the formula.


Z ∞ Z !
R(f
b )(s, γ) := f e−2πist dt
−∞ Pt,γ
Z∞ Z
:= f (tγ + u1 e1 + u2 e2 ) du1 du2 e−2πist dt
2
Z −∞ R
= f (tγ + u)e−2πist du dt
ZR3
= f (tγ + u)e−2πisγ·(tγ+u) du dt
ZR3
= f (x)e−2πisγ·x dx (C.O.V.: x = tγ + u)
R3
= fˆ(sγ).

Remark. The Fourier Slice theorem plus the (one-dimensional)
inversion formula yields the following:
Z ∞
R(f )(t, γ) = fˆ(sγ)e2πits ds;
−∞
in other words, the Radon transform of f at (t, γ) is ac-
tually the inverse along the line {sγ : s ∈ R} of the (3-
dimensional) Fourier transform of f , evaluated at t.
Corollary 1.4. Let f, g ∈ S(R3 ). If R(f ) = R(g) then
f = g.
Uniqueness theorem
(not at all so simple
in general; a topic of Proof. Since R(f ) = R(g), R(f − g)(t, γ) ≡ 0. But then
serious study) − g(sγ) = 0 for all s, γ; so f[
Fourier slice implies f[ − g(ξ) ≡
0. Fourier inversion implies f − g ≡ 0, so f = g. 
2. Filtered Backprojection Inversion formula
2.1. The dual Radon transform.
Definition 2.1. Let F : R × S 2 → R. We define the dual
Dual Radon Trans- Radon transform R∗ (F ) : R3 → R of F by
form
FOURIER SLICE THEOREM AND RADON TRANSFORM INVERSION 3

Z

R (F )(x) := F (x · γ, γ) dσ(γ).
S2
Remark. If F (t, γ) is the Radon transform R(f )(t, γ) then What is the dual
Radon transform
F (x · γ, γ) = R(f )(x · γ, γ) of the Radon
transform?
Z
= f,
Px·γ,γ
the integral over the plane with normal vector γ of distance
x · γ from the origin: in other words, the plane passing
through x with normal vector γ. Thus R∗ (F )(x) is the
integral of f over all planes passing through x: the so-called
backprojection of f . The backprojection
operator
2.2. Why is it called the dual? Let V1 = S(R3 ), with
inner product
Z
(f, g)1 = f g;
R3
2
let V2 = S(R × S ) with inner product
Z Z
(F, G)2 = F (t, γ)G(t, γ) dσ(γ) dt.
R S2
Then Duality formula

(R(f ), F )2 = (f, R (F ))1 .
a.k.a. the filtered
2.3. The inversion formula. backprojection
inversion formula
Theorem 2.2. Let f ∈ S(R3 ). Then
∆(R∗ R(f )) = −8π 2 f.
Proof. As we noted earlier, the Fourier slice theorem (plus
the one-dimensional inversion formula) imply 1. Fourier slice theo-
Z ∞ rem
R(f )(t, γ) = fˆ(sγ)e2πits ds;
−∞
so Z Z ∞

R R(f )(x) = fˆ(sγ)e2πix·γs ds dσ(γ).
S2 −∞
4 FOURIER SLICE THEOREM AND RADON TRANSFORM INVERSION

2. differentiation un- Then, differentiation under the integral sign (plus the fact
der the integral sign that γ ∈ S 2 ) imply
Z Z ∞

∆(R R(f ))(x) = fˆ(sγ)(−4π 2 s2 )e2πix·γs ds dσ(γ)
S 2 −∞
Z Z ∞
= −4π 2
fˆ(sγ)e2πix·γs s2 ds dσ(γ)
2
ZS Z−∞ ∞
= −4π 2
fˆ(sγ)e2πix·γs s2 ds dσ(γ)
2
Z SZ 0 0
− 4π 2 fˆ(sγ)e2πix·γs s2 ds dσ(γ)
S 2 −∞
Z Z ∞
= −8π 2 fˆ(sγ)e2πix·γs s2 ds dσ(γ)
S2 0
2
= −8π f (x).
Notes on the above calculation:
invariance under ro- 1. The second-to-last inequality follows since
tation Z Z 0
−4π 2 fˆ(sγ)e2πix·γs s2 ds dσ(γ)
S 2 −∞
Z Z ∞
= −4π 2
fˆ(−sγ)e−2πix·γs s2 ds dσ(γ)
2
ZS Z0 ∞
= −4π 2 fˆ(sγ)e2πix·γs s2 ds dσ(γ)
S2 0
3. polar integration where one rotates γ to −γ in the last step.
2. The last equality follows from (reverting) the polar
integration formula:
Z Z ∞ Z
fˆ(sγ)e2πix·γs 2
s ds dσ(γ) = fˆ(ξ)e2πix·ξ dξ
S2 0 R3
= f (x).

Remark. In general, the reconstruction formula for the Radon
Reconstruction for- transform is as follows:
mula in Rd
FOURIER SLICE THEOREM AND RADON TRANSFORM INVERSION 5

(2π)1−d (d−1)
(−∆) 2 R∗ (R(f )) = f,
2
where the fractional Laplacian is defined by Fractional Lapla-
Z cian, or inverse
(−∆)α f (x) := (2π|ξ|)2α fˆ(ξ)e2πiξ·x dξ Riesz transform
Rd
d
for f ∈ S(R ). Note that the formula is (again) more simple
for the odd dimensions than for the even.
Remark. In using the Schwartz class, we have actually swept
an entire world of details under a rug. For the interested
reader, please see, for example, http://equinto.math.tufts.edu/
research/sc-article.pdf, an introductory article to the field Reference to
Quinto’s
by Todd Quinto of Tufts (whose thesis advisor was Cor- from the workshoparticle

mack...wait, no, that’s false (http://equinto.math.tufts.edu/


CV/vitatuft.pdf)).
3. Wave Equation and Radon Transform
Definition 3.1. Let F : R → R be a C 2 (R) function, and
u a function on Rd × R. If there exists a vector γ ∈ S d−1
such that Plane wave

u(x, t) = F ((x · γ) − t),


then we call F a plane wave.
Remarks. (Plane waves and the wave equation.) Plane waves and the
wave equation
i. Such functions are solutions of the wave equation in Rd .
ii. Such u are constant on planes perpendicular to γ.
iii. As t increases, the wave travels in the γ direction.
iv. In fact, for d > 1 the solution of the wave equation can
be expressed as an integral of plane waves (by using the
Radon transform).
Reference: Helgason, Sigurdur. Radon Transforms and
Wave Equations, Springer Lecture Notes in Mathematics,
1996. Full text is available at http://www.springerlink.com/
content/d664n43145015772/fulltext.pdf.
LECTURE 28: FINITE FOURIER ANALYSIS:
BASIC DEFINITIONS

1. Background knowledge: The Group Z/N Z


roots of unity
Definition 1.1. Let z ∈ C, N ∈ N. If z N = 1, we say z is
an N th root of unity.
1.1. Z(N ). Z(N )
Note that the set, which we’ll notate as Z(N ), of N th
roots of unity is exactly
{1, e2πi/N , e2πi2/N , . . . , e2πi(N −1)/N }.
Also note that Z(N ) is, with complex multiplication as its
group law, an abelian group, i.e., it is: Definition of an
abelian group
i. (Closed under group law:) If z, w ∈ Z(N ) then zw ∈
Z(N )
ii. (Abelian:) If z, w ∈ Z(N ) then zw = wz
iii. (Identity) 1 ∈ Z(N )
iv. (Inverses) If z ∈ Z(N ) then there exists z −1 ∈ Z(N )
such that zz −1 = 1.
1.2. Z/N Z. Z/N Z

Definition 1.2. Let N ∈ N, and x, y ∈ Z. If x − y is


divisible by N , we say x is congruent to y mod N :
x≡y mod N.
It is obvious that the relation is Equivalence relation
i. (reflexive) x ≡ x mod N for all x ∈ Z
ii. (symmetric) x ≡ y mod N implies y ≡ x mod N
iii. (transitive) x ≡ y mod N and y ≡ z mod N imply
x ≡ z mod N ;
in other words, an equivalence relation on Z.
1
2 THE FAST FOURIER TRANSFORM

Definition 1.3. One calls the set of equivalence classes on


Z/N Z Z modulo this relation the integers modulo N , denoted by
Z/N Z.
Now, for each x ∈ Z, let R(x) denote the equivalence class
corresponding to x. It is easy to see that one can define
a group law (addition) on the set of equivalence classes by
Additive group law defining
for Z/N Z
R(x) + R(y) = R(x + y).
It is easy to see that if x0 ∈ R(x) and y 0 ∈ R(y) (i.e.,
x0 ≡ x mod N , y 0 ≡ y mod N ) then x0 + y 0 ∈ R(x + y)
Isomorphism (i.e., x0 + y 0 ≡ x + y mod N ).
Proposition 1.4. The association R(k) ↔ e2πik/N gives
a correspondence (in fact, a group isomorphism) between
Z/N Z and Z(N ).
Remark. In the same way, we can create an identification
between the functions on Z/N Z and Z(N ).
2. The characters on Z(N )
The space of (cts.)
functions on Z(N ) Notation 2.1. We let V denote the (N -dimensional) in-
ner product space of functions F : Z(N ) → C, with inner
product
N
X −1
(F, G) := F (k)G(k).
k=0
Defining the (obvi- and the associated norm.
ous) inner product
Question. What should be the analogues, for Fourier anal-
ysis on Z(N ), of the functions en (x) = e2πinx on the circle?
Desirable properties The key properties of those functions are the following:
of characters
i. {en }n∈Z is an orthonormal set, with respect to the inner
product
THE FAST FOURIER TRANSFORM 3

ii. The collection of finite linear combinations of the {en }


Note that if we had is dense in the space of continuous functions on the
a set whose span was circle
dense in V , then the
span would have to iii. en (x + y) = en (x)en (y)
be all of V .
Notation 2.2. Let ζ = e2πi/N . We define, for ` = 0, . . . , N −
1, the functions e` on Z(N ) by The characters...?
k (Not quite.)
e` (k) := ζ ` = e2πi`k/N .


Lemma 2.3. The set {e0 , . . . , eN −1 } is orthogonal (and


thus a basis of V ).
Proof. A calculation:
N
X −1 N
X −1
mk −`k
(em , e` ) := ζ ζ = ζ (m−`)k ,
k=0 k=0
which, if m 6= ` yields (a geometric sum)
1 − (ζ m−` )N
= 0,
1 − ζ m−`
and yields N if m = `. 
Thus if we normalize the {e` }, we obtain an orthonormal
basis of V :
Notation 2.4 (Orthonormal basis of V ). Let e∗` = √1 e` .
N
The ONB
3. Finite Fourier Analysis
Definition 3.1. Give F ∈ V , we define the nth Fourier
coefficient of F by Fourier coefficients
on Z(N )
1
an = √ (F, e∗n )
N
N −1
1 X
= F (k)e−2πink/N
N
k=0
LECTURE 29: FINITE FOURIER ANALYSIS:
THE FAST FOURIER TRANSFORM

1. Finite Fourier Analysis


Definition 1.1. Give F ∈ V , we define the nth Fourier
coefficient of F by Fourier coefficients
on Z(N )
1
an = √ (F, e∗n )
N
N −1
1 X
= F (k)e−2πink/N
N
k=0
“Obviously” because
Finite Fourier inversion and Plancherel are (obviously) Fourier analysis =
linear algebra statements about orthonormal bases. infinite-dimensional
linear algebra; so
finite-dimensional
Theorem 1.2 (Fourier inversion). Let F ∈ V . Then Fourier analysis =
N
X −1 linear algebra.
F (k) = an e2πink/N .
n=0
Fourier inversion on
Z(N )
Proof. (Do this backwards.)
N −1 N −1
X X 1
an e 2πink/N
:= √ (F, e∗n )e2πink/N
n=0 n=0
N
N
X −1
= (F, e∗n )e∗n (k) = F (k)
n=0
since {e∗n } is an ONB of V . 
Theorem 1.3 (Parseval-Plancherel formulae).
N −1 N −1
X
2 1 X
|an | = |F (n)|2 .
n=0
N n=0
Again, just proper-
1 ties of ONBs.
2 THE FAST FOURIER TRANSFORM

Proof.
N −1
1 X 1
|F (n)|2 =: ||F ||2
N n=0 N
N −1
1 X
= |(F, e∗n )|2
N n=0
N −1 2 N −1
X 1 ∗
X
|an |2 .

= √N (F, en ) =:

n=0 n=0


2. FFT (the Fast Fourier Transform)


Question. How does one best calculate the Fourier coeffi-
cients of a function F on Z(N )?
The Question

2.1. Naive calculation.


It is easy to see that if one is given the values of F (0), . . . , F (N −
1) and ωN = e−2πi/N , then calculating the N Fourier coeffi-
N −1
cients {aNk (F )}k=0 of F on Z(N ) requires at most 2N + N
2

operations. After all, by definition,


N −1
1 X
aN
k (F ) := kr
F (r)ωN .
N r=0
Straightforward Then
counting 2 N −1
i. Calculating ωN , . . . , ωN takes at most N − 2 multi-
plications
ii. For each aNk (F ), one needs at most 1+N multiplications
1
(first by N , then the N products inside the sum) and
N − 1 additions (the sum).
iii. Thus we have N − 2 operations at the beginning, and
then N × 2N operations afterwards, totalling 2N 2 +
N − 2.
However, one can actually do much better than this.
THE FAST FOURIER TRANSFORM 3

2.2. The fast Fourier transform.


Theorem 2.1 (FFT). Given ωN = e−2πi/N (with N = 2n ),
it takes at most Much better ver-
sion.
4 · 2n n = 4N log2 (N ) = O(N log N )
operations to calculate the Fourier coefficients of a function
on Z(N ).
Notation 2.2. Let #(M ) denote the minimum number
of operations needed to calculate all the Fourier coeffi- Notation: mini-
mum number of
cients of any function on Z(M ). operations
Lemma 2.3 (The key lemma). If we are given ω2M =
e−2πi/(2M ) then
#(2M ) ≤ 2#(M ) + 8M.
2M
Proof of lemma. First note that to calculate ω2M , . . . , ω2M
2
takes no more than 2M operations; also observe that ω2M =
−2πi/M
e =: ωM .
Now, given any function F on Z(2M ) define F0 and F1
on Z(M ) by The main idea:
to divide F into
F0 (n) = F (2n), F1 (n) = F (2n + 1); even and odd parts,
whose Fourier
by definition, it is possible to calculate the Fourier coeffi- coefficients can be
cients of F0 and F1 with at most #(M ) operations each. obtained in ≤ #(M )
steps
Notating Fourier coefficients corresponding to Z(2M ) and
Z(M ) as a2Mk and aM k , we claim: and notice that the
Fourier coefficients
1 M
a2M M k

(F ) = a (F ) + a (F )ω of F can be obtained
k 0 1 2M ;
2 k k from the Fourier
coefficients of the
thus after obtaining the Fourier coefficients for F0 and F1 , even and odd parts.
each a2M
k (F ) can be obtained in three operations (one mul-
tiplication, one addition, one multiplication) and thus
#(2M ) ≤ 2M = 2#(M ) + 3 × 2M = 2#(M ) + 8M,
k
(steps to calculate the {ω2M }2M
k=1 , the Fourier coefficients
of F0 and F1 , and then finally the coefficients a2M
k (F )) as
desired.
4 THE FAST FOURIER TRANSFORM

So the only thing left to do is to prove the claim. But


this is basically a tautology: just break the sum into its
The first step we do even and odd terms:
nothing; the second 2M −1
we do even less. 2M 1 X kr
ak (F ) := F (r)ω2M
2M r=0
−1 −1
M M
!
1 1 X k(2`) 1 X k(2m+1)
= F (2`)ω2M + F (2m + 1)ω2M
2 M M m=0
`=0
−1 −1
M M
!
1 1 X k(2`) 1 X k(2m+1)
=: F0 (`)ω2M + F1 (m)ω2M .
2 M M m=0
`=0

Noticing that
k(2`) k`
ω2M = ωM and
k(2m+1) 2
mk k mk k
ω2M = ω2M ω2M = ωM ω2M
by the observation in the first paragraph of the proof fin-
ishes the claim.

Remark. Notice that the algorithm is built into the proof
of the lemma. The FFT was discovered by Cooley and
Tukey in 1965; however, in 1984 it was discovered that it
(as usual) had already been known to Gauss around 1805.
LECTURE 30: FINITE FOURIER ANALYSIS:
THE FAST FOURIER TRANSFORM

1. Proof of FFT
Theorem 1.1 (FFT). Given ωN = e−2πi/N (with N = 2n ),
it takes at most Much better ver-
sion.
4 · 2n n = 4N log2 (N ) = O(N log N )
operations to calculate the Fourier coefficients of a function
on Z(N ).
Recall: the key was that calculation of the coefficients
on Z(2M ) could be done by calculating the coefficients of
related functions (the odd and even parts) on Z(M ):
Lemma 1.2 (The key lemma). If we are given ω2M =
e−2πi/(2M ) then
#(2M ) ≤ 2#(M ) + 8M.
Once we have the key lemma, the theorem follows imme-
diately: Proof of theorem fol-
lows from lemma by
Proof of Theorem. Let N = 2n ; we induct on n. In the case induction trivially.
n = 1 (N = 2), by definition
1 1
aN N
0 (F ) = [F (1) + F (−1)] and a1 (f ) = [F (1) − F (−1)],
2 2
which requires 5 < 8 operations; so case n = 1 is verified.
For the inductive step, we assume that the theorem is
true for N = 2n−1 ; i.e., that
#(N ) ≤ 4 · 2n−1 (n − 1).
Then, by the lemma,
#(2N ) ≤ 2 · 4 · 2n−1 (n − 1) + 8 · 2n−1 = 4 · 2n n,
as desired. 
1
2 THE FAST FOURIER TRANSFORM

2. Fourier Analysis on (finite) Abelian Groups


Remark. We’re just going to set up the background knowl-
edge necessary. See how we extend the notions of Fourier
analysis; what’s needed, etc.
In fact one can
create a theory for
locally compact Definition 2.1. An abelian group is a set with a binary
(i.e., each point
has a neighborhood
operation (“group law”) on pairs of elements of G,
contained in a G×G→G
compact set) abelian
groups. (a, b) 7−→ a · b,
Definition of abelian that satisfies the follwing properties:
group
i. (commutativity) a · b = b · a for all a, b ∈ G;
ii. (associativity) a · (b · c) = (a · b) · c for all a, b, c ∈ G;
iii. (identity) there exists an element u ∈ G such that a·u =
a for all a ∈ G; and
iv. (inverses) given any a ∈ G, there exists an element
a−1 ∈ G such that aa−1 = u.
Examples and non-examples: (R, ·), (R∗ , ·), (R, +), SL2 (R),
SO2 (R), etc.
Definition 2.2. We say n ∈ Z(q) (q ∈ N) is a unit if there
Example: Z∗ (q) exists an m ∈ Z(q) such that
(1) nm ≡ 1 mod q.
Remark that The set of all units in Z(q) is denoted Z∗ (q); it is an abelian
multiplication is group under multiplication mod q.
well-defined on the
equivalence classes
of Z(n) Definition 2.3. The number #(G) of elements in a group
order G is called the order of G, and denoted |G|.
Definition 2.4. Let G, H be (abelian) groups. If a map
f : G → H “preserves the group law” i.e., satisfies
f (a · b) = f (a) · f (b),
homomorphism then we call f a (group) homomorphism.
THE FAST FOURIER TRANSFORM 3

Definition 2.5. A homomorphism f : G → H, if one-to-


one and onto, is called an isomorphism; and in that case
one says “G is isomorphic to H” and writes G ≈ H. isomorphism

Example: the exponential function exp : R → R+ (R


equipped with addition and R+ with multiplication, respec-
tively, as the group laws) is a group isomorphism.
Remark. The existence of an isomorphism is equivalent to
the existence of an inverse homomorphism.
2.1. Structure Theorem for Finite Abelian Groups.
Definition 2.6. Given two finite abelian groups G1 and
G2 , we define the direct product G1 × G2 to be the set of
cartesian pairs direct product

{(g1 , g2 ) : g1 ∈ G1 , g2 ∈ G2 }
with the group law given by
(g1 , g2 ) · (g10 , g20 ) := (g1 · g10 , g2 · g20 ),
with which the set becomes (check) itself a finite abelian
group.
Theorem 2.7 (Structure theorem for finite abelian groups).
Any finite abelian group is isomorphic to a direct product
of groups of the form Z(N ).
Example of struc-
Example. Consider Z∗ (8), the multiplicative group of ture theorem
units of Z/8Z; namely, Z∗ (8) = {1, 3, 5, 7}. Z∗ (8) can
be shown to be isomorphic to Z(2) × Z(2) via, for ex-
ample, the mapping under which {1, 3, 5, 7} correspond to
{(0, 0), (1, 0), (0, 1), (1, 1)} respectively.

3. Characters
Notation 3.1. Let S 1 denote the unit circle in C, equipped
with complex multiplication as the group law.
4 THE FAST FOURIER TRANSFORM

Definition 3.2. Let G be a finite abelian group, and e :


G → S 1 . If for all a, b ∈ G,
e(a · b) = e(a)e(b),
character (in other words, if e is a homomorphism) then we call e a
character.
Lemma 3.3. Let G b denote the set of all characters of G.
dual group If we define, for e1 , e2 ∈ G,
b the product e1 · e2 by
(e1 · e2 )(a) := e1 (a)e2 (a) for all a ∈ G,
then G
b becomes itself an abelian group, called the dual group.

Useful lemma:
Lemma 3.4. Let G be a finite abelian group, and e : G →
Useful lemma: any C\{0}. If e is multiplicative, i.e.,
multiplicative, non-
vanishing map is a e(a · b) = e(a)e(b) for all a, b ∈ G,
character
then e is a character.
Proof. Notice that the function |e| is bounded both above
and below (away from 0) on G (since G is finite). If |e(g)| >
1, then |e(g n )| = |e(g)|n would go to infinity with n: im-
possible. Similarly for |e(g)| < 1. Thus e maps into S 1 and
is a character. 
LECTURE 31:
FOURIER ANALYSIS ON FINITE ABELIAN GROUPS

Recall that we had, in the distant past, just introduced


the notion of characters and defined the dual group G b of
characters on G.

1. Characters
Definition 1.1. Let G be a finite abelian group, and e : G → S 1 . If for all
a, b ∈ G,
e(a · b) = e(a)e(b),
(in other words, if e is a homomorphism) then we call e a character. character

Lemma 1.2. Let G b denote the set of all characters of G. If we define, for
e1 , e2 ∈ G, the product e1 · e2 by
b dual group

(e1 · e2 )(a) := e1 (a)e2 (a) for all a ∈ G,

then G
b becomes itself an abelian group, called the dual group.

Lemma 1.3. Let G be a finite abelian group, and e : G → C\{0}. If e is


multiplicative, i.e., Useful lemma: any
multiplicative, non-
e(a · b) = e(a)e(b) for all a, b ∈ G, vanishing map is a
character
then e is a character.

2. The characters are what we want


Definition 2.1. Given a finite abelian group G, let V de-
note the |G|-dimensional (complex) vector space of func-
tions f : G → C with inner product given by The inner product
space of (continu-
1 X ous) functions.
(f, g) := f (a)g(a) for f, g, ∈ V.
|G|
a∈G

Theorem 2.2. With the above inner product, G


b is an or-
thonormal family.
1
2 FINITE ABELIAN GROUPS

Lemma 2.3. Let e ∈ G


b be a non-trivial character. Then Key lemma: show-
X ing the cancellation
e(a) = 0. property (moment
a∈G condition) for
characters
Proof. Since e is non-trivial, there exists an element b such
that e(b) 6= 1. Then
X X X
e(b) e(a) = e(ba) = e(a),
a∈G a∈G a∈G
the last following since multiplying
P by b is an invertible
(and thus 1-1) map. Thus a∈G e(a) = 0. 
Proof of the theorem. Let’s first show that for e ∈ G,
b (e, e) =
That the norms are 1.
1 is obvious. 1 X 1 X
(e, e) := e(a)e(a) = |e(a)|2 .
|G| |G|
a∈G a∈G
1
since e : G → S the first step is done.
Now it remains to be seen that (e, e0 ) = 0 for e 6= e0 ; that
is,
X
e(a)e0 (a) = 0.
a∈G
1
Well, in the group S , complex conjugation of an element
corresponds to taking the inverse of that element (since
zz = |z|2 = 1 for z ∈ S 1 ), so we can rewrite the above
equivalently as:
X
e(a)(e0 (a))−1 = 0.
a∈G

I.e. - expressing the above using the group law for G


b - we
want to show that
X
[e · (e0 )−1 ](a) = 0.
a∈G

Since e · (e0 )−1 is by construction (e =


6 e0 ) a non-trivial
character, the previous lemma finishes the proof. 
FINITE ABELIAN GROUPS 3

Remark. Thus the characters are linearly independent el-


ements of V . Since dim(V ) = |G|, we see |G|
b ≤ G. In
fact....
3. G
b is an ONB of V
Theorem 3.1. Let G be a finite abelian group. G b is a
(orthonormal) basis for the vector space of functions on G.
unitary transforma-
3.1. Some linear algebra: the Spectral Theorem. tions

Definition 3.2. Let V be a d-dimensional inner product


space and T : V → V be a linear transformation. If, for all
v, w ∈ V , (T v, T w) = (v, w) then we call T unitary.
The Spectral Theo-
rem: unitary ⇒ di-
Theorem 3.3 (The Spectral Theorem). Given any unitary agonalizable
transformation T : V → V , there exists a basis {v1 , . . . , vd }
of V of eigenvectors of T .
Corollary 3.4 (Simultaneous diagonalization). Let V be a
finite-dimensional inner product space, and {T1 , . . . , Tk } be Consequence: simul-
taneous diagonaliza-
a family of linear transformations on V . If all {Ti } com- tion of commuting
mute, then there exists a basis for V consisting of eigenvec- unitary transforma-
tors for every Ti . tions

Proof. By induction. Suppose the lemma is true for any


family of k − 1 commuting unitary transformations. (Base case is the
Consider the family {T1 , . . . , Tk }. By the spectral theo- spectral theorem.)

rem, we can decompose V into a direct sum of eigenspaces


of the last transformation Tk , Decompose V into
eigenspaces of Tk .
V = Vλ1 ⊕ · · · ⊕ Vλs .
Now, given any Tj , 1 ≤ j ≤ k − 1, we notice that for any
v ∈ Vλi , and any 1 ≤ i ≤ s, Notice (by com-
mutation) that
Tk Tj (v) = Tj Tk (v) = Tj (λi v) = λi Tj (v); Tj preserve those
eigenspaces
That is, Tj (v) ∈ Vλi . In other words, each of the Tj pre-
serves each of the eigenspaces.
4 FINITE ABELIAN GROUPS

Now, by induction hypothesis, the family {T1 , . . . , Tk−1 }


is simultaneously diagonalizable on each subspace Vλi ; i.e., Then we can
each Vλi is decomposable into a direct sum of subspaces decompose those
eigenspaces (of
which are eigenspaces for all of the {T1 , . . . , Tk−1 }. Those Tk ) into fur-
eigenspaces are of course contained in Vλi , the eigenspace ther eigenspaces
of the family
for Tk ; so they provide a decomposition of V into eigenspaces {T1 , . . . , Tk−1 }
for the entire family. 
Thus ends the linear algebra portion of this show.
LECTURE 32:
FOURIER ANALYSIS ON FINITE ABELIAN GROUPS

1. G
b is an ONB of V
Theorem 1.1. Let G be a finite abelian group. G b is a
(orthonormal) basis for the vector space of functions on G.

Corollary 1.2 (Simultaneous diagonalization). Let V be


a finite-dimensional inner product space, and {T1 , . . . , Tk } Consequence: simul-
be a family of linear transformations on V . If all {Ti } taneous diagonaliza-
tion of commuting
are unitary and commute, then there exists a basis for V unitary transforma-
consisting of eigenvectors for every Ti . tions

Proof of theorem. Recall that dim(V ) = |G|, and that the


dual group Gb of characters has order |G|
b ≤ |G|. Recall also
that the characters form an orthonormal family; to show
they form a basis, it suffices to show that |G|
b = |G|. Need to check that
there are sufficient
Sketch: we consider the (finite) family of linear transfor- characters to form a
mations (“left-translation”) on V which are not only uni- basis of V

tary, but also commute (because G is abelian). We shall see


that the basis of V which simultaneously diagonalizes the
family consists (essentially) of characters: thus the number
of characters equals dim(V ) = |G|, as desired.
To each a ∈ G, associate a linear transformation Ta :
V → V defined by, for f ∈ V , Create commuting,
unitary transforma-
tions: “translation”
(Ta f )(x) = f (a · x) for all x ∈ G. operators

With the inner product defined on V , we note that


1
Why unitary: if we
shift both functions
over by the same
translation, then
the inner product
doesn’t change.
2 G
b IS AN ORTHONORMAL BASIS

1 X
(Ta f, Ta g) := Ta f (b)Ta g(b)
|G|
b∈G
1 X
= Ta f (b)Ta g(b)
|G|
b∈G
1 X
= f (a · b)g(a · b)
|G|
b∈G
1 X
= f (b)g(b) = (f, g);
|G|
b∈G
i.e., the Ta are all unitary transformations. Further, they
commute (since G is abelian); so by the previous lemma,
The previous corol- they are simultaneously diagonalizable.
lary yields a basis (of Thus we have a basis for V of functions {vb : G → C}b∈G ,
V ) of eigenfunctions.
each of which is an eigenfunction for all of the {Ta }a∈G . We
now
Claim: For each of the {vb : G → C}b∈G , let
vb (x)
wb (x) :=
vb (u)
In fact, those where u denotes the identity element of G. Then
|G| eigenfunctions,
properly normalized,
wb is a character.
are characters; so
we have enough Remark. Why was this the obvious thing to do? Well, think
characters to form a of what a character of G must do: it must, first, preserve
basis.
the group law:
χ(a · x) = χ(a)χ(x);
second, it must preserve the identity:
χ(uG ) = uS1 = 1.
The first statement is equivalent to saying that χ must be
an eigenfunction for the family of translations Ta (for all
a ∈ G); the second forces the normalization required of w
above. Can we get such eigenfunctions? Yes, since the Ta
are unitary and commute.
G
b IS AN ORTHONORMAL BASIS 3

Proof of claim: Let’s use v to denote one of the ba-


sis eigenfunctions, and w for the corresponding normalized
eigenfunction. We first notice that v(u) 6= 0: if v(u) = 0,
then
v(a) = v(a · u) = Ta v(u) = λa v(u) = 0
and then v(a) ≡ 0 for all a ∈ G. In fact, v can never vanish,
for if v(a) = 0, then w never vanishes.

v(u) = v(a−1 a) = Ta−1 v(a) = λa−1 v(a) = 0


Similarly, w can never vanish.
It thus suffices to show that w : G → C\{0} is multi-
plicative; the lemma at the beginning of this lecture would
then imply that it must be a character. Well, letting λa
denote the eigenvalue of v for Ta , we observe w is multiplicative.

v(a · b)
w(a · b) :=
v(u)
Ta v(b) λa v(b) λa λb v(u)
= = =
v(u) v(u) v(u)
= λa λb = w(a)w(b),
using the fact that
v(x) Tx v(u)
w(x) := = = λx ;
v(u) v(u)
so we’re done. Thus there are in fact dim(V ) = |G| char-
acters, so they do form an ONB of V ) 
Once we have that the characters form an orthonormal
basis of V , our Fourier analysis of functions on G follows
as follows.

2. Fourier Analysis on Finite Abelian Groups


Definition 2.1. Let G be a finite abelian group. Given
any function f : G → C and character e ∈ G,
b we define the Fourier coefficients
on (finite) abelian
groups
4 G
b IS AN ORTHONORMAL BASIS

Fourier coefficient of f w.r.t. e as


1 X
fˆ(e) := (f, e) := f (a)e(a).
|G|
a∈G

Further, we define the Fourier series of f as e∈Gb fˆ(e)e.


P
Fourier series

Of course, since the characters form an ONB, we have


Fourier inversion: immediately that f actually equals its Fourier series:
just the statement
that G
b is an ONB of Theorem 2.2 (Fourier inversion). Let G be a finite abelian
V.
group. Then given any function f : G → C, we have that
X
f= fˆ(e)e.
e∈G
b

As before we also have a Plancherel/Parseval theorem:


Theorem 2.3 (Plancherel/Parseval). Let G be a finite abelian
group. Then given any function f : G → C, we have that
X
||f ||2 = |fˆ(e)|2 .
e∈G
b

Proof. Using Fourier inversion, we see that


 
X X
2
||f || := (f, f ) =  ˆ
f (e)e, fˆ()
e∈G
b ∈G
b
XX X
= fˆ(e)fˆ()(e, ) = |fˆ(e)|2 ,
e∈G
b ∈G
b e∈G
b

since the characters are orthonormal. 

A result that shows


3. Dirichlet’s theorem
the ultimate power Theorem 3.1. Let q, ` ∈ N. If q and ` have no common
of Fourier analysis
factor, then the sequence
`, ` + q, ` + 2q, . . . , ` + kq, . . .
contains infinitely many primes.
G
b IS AN ORTHONORMAL BASIS 5

4. Background knowledge:
The Fundamental Theorem of Arithmetic
Theorem 4.1 (Euclid’s algorithm). Let a, b ∈ Z; b > 0.
Then there exists unique integers q and r, with 0 ≤ r < b The basis of long di-
such that a = qb + r. vision.

Proof. Consider
S := {a − qb : q ∈ Z; a − qb ≥ 0}.
S is non-empty; let r = min S. Of course a = qb + r and
r ≥ 0; we need to show r < b.
If r ≥ b, then r = b + s for some s ≥ 0; so Slight error in text:
need not assume s <
b + s = a − qb, r.
and thus 0 ≤ s = a − (q + 1)b < a − qb = r.
But then s ∈ S and s < r; contradiction.
Proving uniqueness: suppose that a = qb + r = q1 b + r1
with 0 ≤ r, r1 < b. Then
(q − q1 )b = r1 − r.
Since |LHS| is (nonnegative) integral multiple of b, and
|RHS| < b, |LHS| = |RHS| = 0b = 0. 
LECTURE 33:
ELEMENTARY NUMBER THEORY

1. Background knowledge:
The Fundamental Theorem of Arithmetic
Theorem 1.1 (Euclid’s algorithm). Let a, b ∈ Z; b > 0.
Then there exists unique integers q and r, with 0 ≤ r < b The basis of long di-
such that a = qb + r. vision.

Proof. Consider
S := {a − qb : q ∈ Z; a − qb ≥ 0}.
S is non-empty; let r = min S. Of course a = qb + r and
r ≥ 0; we need to show r < b.
If r ≥ b, then r = b + s for some s ≥ 0; so Slight error in text:
need not assume s <
b + s = a − qb, r.
and thus 0 ≤ s = a − (q + 1)b < a − qb = r.
But then s ∈ S and s < r; contradiction.
Proving uniqueness: suppose that a = qb + r = q1 b + r1
with 0 ≤ r, r1 < b. Then
(q − q1 )b = r1 − r.
Since |LHS| is (nonnegative) integral multiple of b, and
|RHS| < b, |LHS| = |RHS| = 0b = 0. 
Definitions. Let a, b ∈ Z. Elementary concepts
from number theory
i. If there exists c ∈ Z such that ac = b, then we say a
divides b (a is a divisor of b) and write a|b.
ii. A prime number is a positive integer, greater than 1,
which has no divisors besides 1 and itself.
iii. Say a, b ∈ N. The greatest common divisor gcd(a, b) of
a and b is the largest integer that divides both a and b.
iv. If gcd(a, b) = 1, we say a and b are relatively prime (i.e., they have no
1 nontrivial common
divisor)
2 ELEMENTARY NUMBER THEORY

Theorem 1.2. Let a, b, ∈ N. Then there exist x, y ∈ Z


gcd(a, b) is in the such that
(Z)-span of {a, b}.
xa + yb = gcd(a, b).
Proof. Consider the (positive) span of a and b:
S := {xa + yb : x, y ∈ Z; xa + yb > 0}.
Let s = min S; then in fact we claim that s = gcd(a, b).
Why? Well, obviously, any divisor of both a and b will
Let’s show that s|a. divide s; so s ≥ gcd(a, b). So STS s|a and s|b.
By Euclid’s algorithm,
a = qs + r with 0 ≤ r < s.
We want to show Let’s think about qs. Since s ∈ S,
that r = 0.
xa + yb = s for some x, y ∈ Z
and thus qxa + qyb = qs.
Together, that gives us
qax + qby = a − r;
i.e. r = (1 − qx)a + (−qy)b;
in other words, 0 ≤ s − r ≤ s and s − r ∈ S. Since s =
min S, this forces r = 0. Similarly, s|b; so s = gcd(a, b). 
Corollary 1.3. Let a, b ∈ N. a and b are relatively prime
if and only if there exist x, y ∈ Z such that ax + by = 1.
Characterization of
relative prime-ness
Proof. Obvious: if relatively prime, then the theorem im-
plies there exist such integers. If such integers exist, then
any divisor of both a and b must divide 1; thus the only
divisor is 1. 
Corollary 1.4. Let a, c be relatively prime. If c|ab, then
c|b.
Useful property of
relatively prime
numbers Proof. gcd(a, c) = 1 means there exist x, y ∈ Z such that
xa+yc = 1. Thus xab+ycb = b, and since c|ab and (clearly)
c|ycb, c also divides b. 
ELEMENTARY NUMBER THEORY 3

Corollary 1.5. Let p be prime. If p|a1 · · · ar , then p|ai for


some i.
Corollary of the pre-
vious corollary
Proof. Suppose p - a1 . Then p|a2 · · · ar . Eventually p must
divide one of the ai . 
Theorem 1.6 (Fundamental Theorem of Arithmetic). Ev-
ery positive integer greater than 1 can be factored uniquely
into a product of primes.
hilarious proof
Proof. First we note that all positive integers have prime
factorizations. For consider the set of positive integers with-
out prime factorizations; and let n = min S. Since n is not
a prime, n = ab for some integers 1 < a, b < n. Since n
was minimal, a, b both have prime factorizations; and so n
does: ※.
Now suppose n has two prime factorizations, n = p1 p2 · · · pr
and n = q1 q2 · · · qs . Since p1 |n = q1 q2 · · · qs , we know p1
must divide one of the qi (relabeling, call it q1 ); thus p1 = q1 .
In this manner we see that r = s and that the factorizations
are identical up to permutation. 

2. Euclid’s proof of the infinitude of primes


Theorem 2.1. There are infinitely many primes
Notice that we do
need the Funda-
Proof. By contradiction: suppose there are finitely many, mental Theorem of
Arithmetic for this
Qn label {p1 , . . . , pn }. Then consider the number
which we proof.
N := ( i=1 pi ) + 1. It is larger than any pi , so must be
non-prime; thus has a prime factorization. Qn However, if the
prime pk divides N , then it divides N −( i=1 pi ) = 1, which
is impossible. 
Corollary 2.2. The number of primes of the form 4k + 3
is infinite.
Variation on the
theme
4 ELEMENTARY NUMBER THEORY

Proof. Suppose there are only finitely many such primes:


{p1 = 7, p2 = 11, . . . , pn } (leave 3 out of the list for now).
Let
N = 4p1 p2 · · · pn + 3.
Since N is of the form 4k +3, and further, N > pn , we know
N cannot be prime, and thus has a prime factorization (not
Observe the odd containing 2, since N is odd). Observe that (4m + 1)(4n +
primes are either of
the form 4k + 1 or
1) = 4(mn + m + n) + 1; so N ’s prime factors must include
4k + 3 a prime of the form 4k + 3. But, just as in the previous
argument, none of the primes in our finite list divide evenly
into N . 

3. One result about infinite products



Q 3.1. Given {An ∈ R}n=1 , we define the infinite
Definition
infinite product product An by

Y N
Y
An = lim An .
N →∞
n=1 n=1

condition implying The main result we shall need is the following:


convergence of
product
P
Theorem 3.2.Q Let A n = 1 + a n . If an converges abso-
lutely, then An converges.

Proof of lemma is Lemma 3.3. For |x| < 21 , | ln(1 + x)| ≤ 2|x|.
via power series ex-
pansion of ln(1 + x)
P
around 0
Proof of theorem. Since an converges, WLOG we may
1
assume |an | < 2 for all n. Then
N
Y N
Y
An := eln(1+an )
n=1 n=1
PN
ln(1+an )
=e n=1 .
PN
Here’s where By the lemma, | ln(1 + an )| ≤ 2|an |; so n=1 ln(1 + an )
we use absolute
convergence
ELEMENTARY NUMBER THEORY 5

converges to a limit, B. Thus, by continuity of the expo-


nential function,
YN PN
lim An = lim e n=1 ln(1+an ) = eB .
N →∞ N →∞
n=1

Remarks.
i. Notice that if the product vanishes, then one of the
initial factors (i.e., before |an | < 1/2) must have been
0, since the infinite tail is eB 6= 0.
ii. Notice that if in addition an 6= 1 for all n, then
Y 1 N
Y 1 1
= lim = lim QN
n
1 − an N
n
1 − an N
n (1 − an )
also converges.
LECTURE 34:
EULER’S PRODUCT FORMULA; FOURIER SERIES
FOR CHARACTERISTIC FUNCTIONS OF POINTS

1. Euler’s product formula for the Riemann


zeta function
Definition 1.1. Let s > 1. We define the zeta function by
zeta function

X 1
ζ(s) := s
.
n=1
n

Remark. That this series converges can be seen via the in-
tegral test: Converges because
of the integral test
∞ ∞ Z n Z ∞
X 1 X dx dx 1
s
≤ 1 + s
= 1 + s
= 1 + .
n=1
n n=2 n−1 x 1 x s − 1

Theorem 1.2 (Euler’s product formula). Let s > 1, and


let ℘ denote the collection of primes. Then Analytic version
of the fundamen-
Y 1 tal theorem of
ζ(s) = s
. arithmetic
p∈℘
1 − 1/p

Remark. The intuition: each of the terms in the product


can be realized as a geometric series:
1 1 1
= 1 + + + ··· ;
1 − p1s ps p2s
Y 1 Y 1 1

so s
= 1 + s + 2s + · · ·
p∈℘
1 − 1/p p
p p
1
2 DIRICHLET CHARACTERS AND REDUCTION OF THEOREM

Now, by the fundamental theorem of arithmetic, each n ∈


N can be written as a product n = pk11 pk22 · · · pkmm , so
1 1
= .
(pk11 pk22 · · · pkmm )s ns
“Thus” (at least formally)
∞ Y 
X 1 1 1
ζ(s) := s
= 1 + s + 2s + · · ·
n=1
n p
p p

Proof. Let N ∈ N, and consider


N
X 1
.
n=1
ns
Now, pick any M > N . Each integer n = 1, 2, . . . , N has
a prime factorization in terms of primes p < N , where p
occurs fewer than M times in that factorization. Thus
N Y 
X 1 1 1 1
s
≤ 1 + s + 2s + · · · + M s
n=1
n p p p
p≤N
Y  1  Y 1 
≤ ≤
1 − p−s p
1 − p−s
p≤N
Letting N → ∞ gives
Y 1

ζ(s) ≤ .
p
1 − p−s
Conversely, by the fundamental theorem of arithmetic, we
have for any N ,
Y  X ∞
1 1 1 1
1 + s + 2s + · · · + M s ≤
p p p n=1
ns
p≤N
Thus, letting M → ∞,
Y  ∞
1 X 1

1 − p−s n=1
ns
p≤N
DIRICHLET CHARACTERS AND REDUCTION OF THEOREM 3

and thus, letting N → ∞,


Y 1  X ∞
1
−s
≤ s
.
p
1 − p n=1
n

Theorem 1.3 (Euler’s analytic version of the infinitude of


primes).
X1
The series diverges.
p∈℘
p
Take that, Euclid!

Lemma 1.4. ln(1 + x) = x + E(x) where |E(x)| ≤ x2 for


|x| < 1/2.

Proof of lemma. Using the power series expansion,


x2 x3 x4
E(x) = ln(1 + x) − x = − + − + −···
2 3 4
x2
so |E(x)| ≤ (1 + |x| + |x|2 + · · · )
2 
x2

1 1
≤ 1 + + 2 + · · · = x2
2 2 2
for |x| ≤ 12 . 

Proof of theorem. By Euler’s formula


Y 1 
ζ(s) = −s
, so
p∈℘
1 − p
X  1

ln ζ(s) = − ln 1 − s
p∈℘
p

By the lemma, Slight error in book:


big O mistake
4 DIRICHLET CHARACTERS AND REDUCTION OF THEOREM
X 1  
1
ln ζ(s) = − − s +E
p∈℘
p ps
X 1
= s
+ C,
p∈℘
p
since

X  1  X 1

X 1 π2
E
s 2s
≤ 2
= .
p∈℘
p p∈℘
p n=1
n 6
I.e.,
X 1
s
= ln ζ(s) − C.
p∈℘
p
Then consider:

X 1
lim inf ζ(s) := lim inf
s→1+ s→1+
n=1
ns
M M
X 1 X 1
≥ lim inf s
= ,
s→1+
n=1
n n=1
n
for every M ∈ N. So then
X 1
lim inf s
≥ lim inf ln ζ(s) = ∞
s→1+
p∈℘
p s→1+

and thus p∈℘ p1 ≥ lim inf s→1+ p∈℘ p1s = ∞ as well.


P P


2. Dirichlet characters and


the reduction of the theorem
The above approach showing that there are infinitely
many primes was extended by Dirichlet to show that there
are an infinitude of primes of the form p ≡ ` mod q. It
suffices to show that
X 1
diverges.
p
p∈℘: p≡` mod q
DIRICHLET CHARACTERS AND REDUCTION OF THEOREM 5

The proof will involve using the Fourier series for functions
on Z∗ (q), the units (invertible elements) of Z(q).
Question. What is Z∗ (q)? a ∈ Z(q) being invertible means
of course that there exists x ∈ Z(q) such that xa ≡ 1
mod q, i.e., that gcd(a, q) = 1. In other words, Z∗ (q) is
precisely the set of (equivalence classes of) numbers rela-
tively prime to q.
minor notation: Eu-
ler phi function
Definition 2.1. Define the Euler ϕ-function by
ϕ(q) := | Z∗ (q)|.
Characteristic func-
Let δ` : Z∗ (q) → R denote the characteristic function χ{`} tions of singletons in
of the singleton ` ∈ Z∗ (q), i.e., Z∗ (q):

1 if n ≡ ` mod q
δ` (n) :=
0 otherwise.
Dumb comment: we extend δ` to all of Z(q) (and thus all Extending δ` to all of
Z - this is just a pe-
of Z) by defining it as 0 on the non-units of Z(q), i.e., riodic extension

χ{`} ([n]) if n and q are relatively prime
δ` (n) =
0 otherwise.
Definition 2.2. Let e ∈ G b be a character. We define the
Dirichlet character modulo q extending e, denoted χ = χ(e) : Extending char-
Z → C by acters to all of Z,
 again periodically
e([m]) if m and q are relatively prime
χ(m) :=
0 otherwise.
We denote the extension of the trivial character of G by χ0 .
Remark. Observe that the Dirichlet characters are multi-
plicative on all of Z.
Expressing δ` in
terms of characters
Lemma 2.3.
1 X
δ` (n) := χ(`)χ(n).
ϕ(q)
χ(e) :e∈G
b
6 DIRICHLET CHARACTERS AND REDUCTION OF THEOREM

Proof. Consider the Fourier coefficients on G of δ` :


1 X 1
δ` (e) :=
b δ` (m)e(m) = e(`).
|G| |G|
m∈G
Fourier series Thus the Fourier inversion formula yields
of characteristic X
function of a δ` (n) := δb` (e)e(n)
singleton
e∈G
b
1 X
= e(`)e(n).
|G|
e∈G
b
Rewriting the above in terms of the extensions of the func-
tions to Z, we have the desired result. 
LECTURE 35:
EULER’S PRODUCT FORMULA; FOURIER SERIES
FOR CHARACTERISTIC FUNCTIONS OF POINTS

1. Reduction of the problem


Recall: our goal is to show Dirichlet’s theorem, i.e., that
for q, ` which are relatively prime,
X 1
diverges.
p
p∈℘: p≡` mod q
We reduced the problem as follows. First, we took the
characteristic function δ` of {`} as a function on Z∗ (q), and
the characters e ∈ Z[ ∗ (q). By our work in the previous

chapter (recall by definition ϕ(q) := | Z∗ (q)|), we have the


following Fourier series expansion:
Lemma 1.1.
1 X
δ` (n) = e(`)e(n).
ϕ(q)
e∈G
b
Perhaps the best
We then extended the definitions of δ` and e to all of Z way is to present
periodically (defining them as 0 whenever n was not rela- this backwards,
i.e., realize the
tively prime with q), and letting χ denote the extension of e sum in terms of
to Z. Then by the above inversion formula, since δ` (p) = 1 the characteristic
function which can
iff p ≡ ` mod q, be thought of
X 1 X δ` (p) as a function on
=: Z∗ (q) and as thus
ps p∈℘
ps possessing a Fourier
p∈℘ s.t. p≡` mod q
series expansion.
1 X X χ(p)
= χ(`) s
.
ϕ(q) p∈℘
p
χe :e∈G
b

Breaking the first sum into the (Dirichlet extension of the)


trivial character and the non-trivial ones, we get that the
1
2 DIRICHLET CHARACTERS AND REDUCTION OF THEOREM

above equals
1 X χ0 (p) 1 X X χ(p)
= + χ(`)
ϕ(q) p∈℘ ps ϕ(q) p∈℘
ps
χe 6=χ0
1 X 1 1 X X χ(p)
= + χ(`) ,
ϕ(q) ps ϕ(q) p∈℘
p s
p∈℘: p-q χe 6=χ0

the last because χ0 (p) is the extension of the trivial char-


acter (taking 1 on all of Z∗ (q)); i.e., it indicates whether or
not p ∈ ℘ is relatively prime with q, which is equilvalent to
saying that p does not divide q.
Now let’s consider the first sum,
X 1
.
ps
p∈℘:p-q

Snce
P all but finitely many primes do not divide q, and since
1
p∈℘ p diverges (Euler’s theorem), we see that this sum
diverges as s → 1+ .
Thus to show that the sum diverges, it suffices to show
that
1 X X χ(p)
χ(`) s
, < ∞,
ϕ(q) p∈℘
p
χe 6=χ0

as s ↓ 1, which we shall do by proving the following.


Theorem 1.2. Let χ be any nontrivial Dirichlet character.
Then
X χ(p)
ps
p∈℘:p≡` mod q

is bounded as s → 1+ .
2. The strategy
The key to proving the reduced problem will be the fol-
lowing.
DIRICHLET CHARACTERS AND REDUCTION OF THEOREM 3

Definition 2.1. For s > 1, χ a Dirichlet character (modulo


q), we define the L-function L by

X χ(n)
L(s, χ) := s
.
n=1
n
Theorem 2.2 (Product formula for L-functions). For s >
1,

X χ(n) Y 1
L(s, χ) := s
= χ(p)
.
n=1
n p∈℘ 1 − s p
Then, formally,
 
X χ(p)
ln L(s, χ) = − ln 1 − s
p∈℘
p
  
X χ(p) 1
=− − s +E
p∈℘
p ps
X χ(p)
= s
+ C;
p∈℘
p

we use this relation to show that lims↓1 p∈℘ χ(p)


P π2
ps is finite. Again, |C| ≤ 6 .

To make the above argument rigorous, we need to do the


following.
i. Extend the logarithm to complex numbers,
ii. Interpret properly the (complex, multi-valued) loga-
rithm of the product,
iii. Show that L(s, χ) is continuous at s = 1, and
iv. Show that L(1, χ) 6= 0.
3. Logarithms and infinite products on C
Question. How do we extend the logarithm to C?
Recall that for |x| < 1 one has the power series Via power series ex-
∞ pansion of the loga-
X (−1)n+1 n rithm.
log(1 + x) = x
n=1
n
4 DIRICHLET CHARACTERS AND REDUCTION OF THEOREM

which implies that (again for |x| < 1)


  X ∞
1 1 n
log = x .
1−x n=1
n
Extension of the Definition 3.1. For z ∈ C, |z| < 1, we define
natural logarithm ∞
zk
 
to....somewhere. 1 X
log1 := .
1−z k
k=1
1
Question. Consider {w = 1−z : |z| < 1}. What is this set
of points?
Question: For what
1
set of points are we Claim: {w = 1−z : |z| < 1} = {w ∈ C : <(w) > 1/2}.
actually defining the
log? Proof. ⊂:
1 1 1 + re−iθ
w= = ·
1−z 1 − reiθ 1 + re−iθ
1 + re−iθ
=
1 − r2  
1 r
= 2
+ 2
eiθ
1−r 1−r
     
1 r r
= + cos θ + i sin θ
1 − r2 1 − r2 1 − r2
Considering the real part of the above, we see that
1−r 1
<(w) ≥ = ;
1 − r2 1+r
since 0 ≤ r < 1, <(w) > 1/2.
1
⊃: Conversely, if <(w) > 1/2, and we write w = 1−z , we
see that z = w−1w . Thus
(w − 1)(w − 1)
|z|2 = zz =
ww
1 − (w + w)
=1+ .
ww
Since w + w = 2<(w) > 1, the second term above is nega-
tive; so 0 ≤ |z| < 1. 
DIRICHLET CHARACTERS AND REDUCTION OF THEOREM 5

Remark. So our above definition is a generalization of the


natural logarithm for x > 1/2.
Question. What happens if we extend the logarithm in a
different way? Do we get the same function?
What properties
does this new
Proposition 3.2 (Properties of log1 ). logarithm have?

i. If |z| < 1, then


1
elog1 ( 1−z ) =
1
.
1−z
ii. If |z| < 1, then
 
1
log1 = z + E1 (z),
1−z
where |E1 (z)| ≤ |z|2 for |z| ≤ 1/2.
iii. If |z| < 1/2, then
 

log1 1
≤ 2|z|.
1−z
Proof of (i). Using polar coordinates, let z = reiθ (0 ≤ r <
1). We want to show that
P∞ (reiθ )k
(1 − reiθ )e k=1 k = 1.
Well, when r = 0, the left hand side does equal 1, so it
d
suffices to show that dr (LHS) = 0. But this is easy (using
the fact that we can differentiate term-by-term in the disk
of convergence):

!0
iθ k
d P ∞ (reiθ )k X (re ) P∞ (reiθ )k
(LHS) = (1 − reiθ )e k=1 k · + (−eiθ )e k=1 k
dr k
k=1
" ∞
!0 #
iθ k
X (re ) P∞ (reiθ )k
= −eiθ + (1 − reiθ ) e k=1 k
k
k=1
6 DIRICHLET CHARACTERS AND REDUCTION OF THEOREM

Differentiating term-by-term, we see that


" ∞
!0 #
iθ k
X (re )
−eiθ + (1 − reiθ )
k
k=1

X
= −eiθ + (1 − reiθ )e iθ
(reiθ )k−1
k=1
1
= −eiθ + (1 − reiθ )eiθ = 0.
1 − reiθ
With the logarithm

in hand, we can now P
prove an analogous Proposition 3.3 (Condition forP convergence). Let an be
theorem for infinite
products of complex
a series of complex numbers. If an converges absolutely,
numbers. and an 6= 1 for all n, then
∞  
Y 1
converges.
n=1
1 − a n

Proof. As before, WLOG assume |an | < 1/2 for all n. Using
the properties of the log1 , and then the exponential function
(note that we use the complex exponential function from
The proof is com- chapter 1 as well)
pletely the same; no
N   Y N
point in showing it. 1
elog1 ( 1−an )
Y 1
=
n=1
1 − an n=1
PN
= e n=1 log1 ( 1−an ) .
1

P
Just like in the earlier proof, we note that since |an |
converges, and since
 

log1 1
≤ 2|an |,
1 − an
the sum in the exponent converges, and thus the limit ex-
ists. 
LECTURE 35:
EULER’S PRODUCT FORMULA; FOURIER SERIES
FOR CHARACTERISTIC FUNCTIONS OF POINTS

1. Reduction of the problem


Recall: our goal is to show Dirichlet’s theorem, i.e., that
for q, ` which are relatively prime,
X 1
diverges.
p
p∈℘: p≡` mod q
As before
P 1 we’ll con-
We reduced the problem as follows. First, we took the sider ps as s ↓ 1.

characteristic function δ` of {`} as a function on Z∗ (q), and


the characters e ∈ Z[ ∗ (q). By our work in the previous

chapter (recall by definition ϕ(q) := | Z∗ (q)|), we have the


following Fourier series expansion:
Lemma 1.1.
1 X
δ` (n) = e(`)e(n).
ϕ(q)
e∈G
b
Perhaps the best way
We then extended the definitions of δ` and e to all of Z is to present this
periodically (defining them as 0 whenever n was not rela- backwards, i.e., real-
ize the sum in terms
tively prime with q), and letting χ denote the extension of of the characteristic
e to Z. Then since δ` (p) = 1 iff p ≡ ` mod q, function which can
be realized as a
X 1 X δ` (p) function on Z∗ (q)
=: and as thus possess-
ps p∈℘
ps ing a Fourier series
p∈℘ s.t. p≡` mod q
expansion.
1 X X χ(p)
= χ(`) s
.
ϕ(q) p∈℘
p
χe :e∈G
b

the second equality being the above Fourier series expan-


sion. Breaking the first sum into the (Dirichlet extension of Step 1: Fourier se-
1 ries expansion
2 DIRICHLET CHARACTERS AND REDUCTION OF THEOREM

the) trivial character and the non-trivial ones, we get that


Step 2: break sum the above equals
into trivial and non-
trivial characters 1 X χ0 (p) 1 X X χ(p)
= + χ(`)
ϕ(q) p∈℘ ps ϕ(q) p∈℘
ps
χe 6=χ0
1 X 1 1 X X χ(p)
= + χ(`) ,
ϕ(q) ps ϕ(q) p∈℘
p s
p∈℘: p-q χe 6=χ0

What’s χ0 ? It the last because χ0 (p) is the extension of the trivial char-
will take the value
1 at numbers rela-
acter (taking 1 on all of Z∗ (q)); i.e., it indicates whether or
tively prime to q; not p ∈ ℘ is relatively prime with q, which is equilvalent to
that is, primes that saying that p does not divide q.
do not divide q.
Observe that almost
Now let’s consider the first sum,
all primes do not di- X 1
vide q
.
ps
p∈℘:p-q

Since P
all but finitely many primes do not divide q, and
since p∈℘ p1 diverges (Euler’s theorem), we see that this
sum diverges as s → 1+ .
Thus to show that the sum diverges, it suffices to show
that
1 X X χ(p)
χ(`) s
, < ∞,
ϕ(q) p∈℘
p
χe 6=χ0

as s ↓ 1, which we shall do by proving the following.

Theorem 1.2. Let χ be any nontrivial Dirichlet character.


Reduction of the Then
problem
X χ(p)
ps
p∈℘:p≡` mod q

is bounded as s → 1+ .
DIRICHLET CHARACTERS AND REDUCTION OF THEOREM 3

2. Dirichlet L-functions; Product formula


Dirichlet L-function
(Generalization of Definition 2.1. For s > 1, χ a Dirichlet character (modulo
zeta function)
q), we define the L-function by

X χ(n)
L(s, χ) := s
.
n=1
n

The key to proving the reduced problem will be the fol-


lowing product formula, which will play a role analogous to
that of Euler’s product formula for the zeta function. Statement of product
formula
Theorem 2.2 (Product formula for L-functions). For s >
1,

X χ(n) Y 1
L(s, χ) := s
= χ(p)
.
n=1
n p∈℘ 1 − s p

To accomplish this theorem will involve extending the no-


tions of logarithm and infinite products to complex num-
bers. What we need isQnot
much: logs and .
3. Logarithms and infinite products on C
Question. How do we extend the logarithm to C?
Recall that for |x| < 1 one has the power series Motivation: power
∞ series expansion of
X (−1)n+1 n the logarithm.
log(1 + x) = x
n=1
n
which implies that (again for |x| < 1)
  ∞
1 X 1 n
log = − log(1 − x) = x .
1−x n=1
n

Definition 3.1. For z ∈ C, |z| < 1, we define Extension of the


∞ natural logarithm
zk
 
1 X to....somewhere.
log1 := .
1−z k
k=1
4 DIRICHLET CHARACTERS AND REDUCTION OF THEOREM
1
Question. Consider {w = 1−z : |z| < 1}. What is this set
of points?
Question: For what
1
set of points are we Claim: {w = 1−z : |z| < 1} = {w ∈ C : <(w) > 1/2}.
actually defining the
log? Proof. ⊂:
1 1 1 + re−iθ
w= = ·
1−z 1 − reiθ 1 + re−iθ
1 + re−iθ
=
1 − r2  
1 r
= 2
+ 2
eiθ
1−r 1−r
     
1 r r
= + cos θ + i sin θ
1 − r2 1 − r2 1 − r2
Considering the real part of the above, we see that
1−r 1
<(w) ≥ = ;
1 − r2 1+r
Some “boring” cal- since 0 ≤ r < 1, <(w) > 1/2.
culations (actually 1
they were slightly
⊃: Conversely, if <(w) > 1/2, and we write w = 1−z , we
w−1
fun). see that z = w . Thus
(w − 1)(w − 1)
|z|2 = zz =
ww
1 − (w + w)
=1+ .
ww
Since w + w = 2<(w) > 1, the second term above is nega-
tive; so 0 ≤ |z| < 1. 
Remark. So our above definition is a generalization of the
natural logarithm for x > 1/2.
multi-valued
logarithm
Question. What happens if we extend the logarithm in a
different way? Do we get the same function? Consider
the complex exponential function. Since ew+2kπi = ew , we
see that e is not a 1-1 function; thus there are multiple
logarithms possible: log1 is one of them.
DIRICHLET CHARACTERS AND REDUCTION OF THEOREM 5

Does this logarithm


behave as it ought
Proposition 3.2 (Properties of log1 ). to? What properties
does this new loga-
i. If |z| < 1, then rithm have?

1
elog1 ( 1−z ) =
1
.
1−z
ii. If |z| < 1, then
 
1
log1 = z + E1 (z),
1−z

where |E1 (z)| ≤ |z|2 for |z| ≤ 1/2.


iii. If |z| < 1/2, then
 

log1 1
≤ 2|z|.
1−z

Proof of (i). (The other two facts follow as before.)


Using polar coordinates, let z = reiθ (0 ≤ r < 1). We
want to show that Use polar coor-
dinates; see that

P∞ (reiθ )k f (r) = 1, f 0 (r) = 0,
(1 − re )e k=1 k = 1. so f (r) ≡ 1.

Well, when r = 0, the left hand side does equal 1, so it


d
suffices to show that dr (LHS) = 0. But this is easy (using
the fact that we can differentiate term-by-term in the disk
Note that we of convergence):
can’t differentiate
term-by-term when ∞
!0
|z| ≥ 1: otherwise, d P∞ (reiθ )k X (reiθ )k P∞ (reiθ )k
this would be true (LHS) = (1 − reiθ )e k=1 k · + (−eiθ )e k=1 k

for all z. dr k
k=1
" ∞
!0 #
iθ iθ
X (reiθ )k P∞ (reiθ )k
= −e + (1 − re ) e k=1 k
k
k=1
6 DIRICHLET CHARACTERS AND REDUCTION OF THEOREM

Differentiating term-by-term, we see that


" ∞
!0 #
iθ k
X (re )
−eiθ + (1 − reiθ )
k
k=1

X
= −eiθ + (1 − reiθ )e iθ
(reiθ )k−1
k=1
1
= −eiθ + (1 − reiθ )eiθ = 0.
1 − reiθ
With the logarithm

in hand, we can now P
prove an analogous Proposition 3.3 (Condition forP convergence). Let an be
theorem for infinite
products of complex
a series of complex numbers. If an converges absolutely,
numbers. and an 6= 1 for all n, then
∞  
Y 1
converges.
n=1
1 − a n

Proof. As before, WLOG assume |an | < 1/2 for all n. Using
the properties of the log1 , and then the exponential function
(note that we use the complex exponential function from
The proof is com- chapter 1 as well)
pletely the same;
N   Y N
almost no point in 1
elog1 ( 1−an )
Y 1
showing it. =
n=1
1 − an n=1
PN
= e n=1 log1 ( 1−an ) .
1

P
Just like in the earlier proof, we note that since |an |
converges, and since
 

log1 1
≤ 2|an |,
1 − an
the sum in the exponent converges, and thus the limit ex-
ists. 
LECTURE 35: PRODUCT FORMULA FOR THE
L-FUNCTIONS

1. Proof of the product formula


Recall: we hope to obtain a product formula for L-functions
analogous to that for the zeta function, i.e.:

Theorem 1.1. For s > 1,



X χ(n) Y 1
L(s, χ) := = .
n=1
ns p∈℘ 1 −
χ(p)
ps
Proof of product for-
Proof. PFirst observe both sides converge: the left since |χ| ≤ mula
1 and ∞ 1
n=1 ns converges for s > 1, and the right side con-
verges by the previous proposition (on the convergence of
infinite products), since (letting pn denote the n-th prime)
for s > 1

X χ(pn )
converges absolutely,
n=1
psn
P∞ 1
again in comparison with n=1 ns .
Now, we want to show that

X χ(n) Y 1
− =0


ns χ(p)
p∈℘ 1 − s

n=1 p

i.e., that the difference is smaller than any  > 0. Well, by


Approximate both the triangle inequality, for any N, M ,
sides with finite 1
sums
2 PROOF OF DIRICHLET’S THEOREM

X χ(n) Y 1
− ≤


ns χ(p)

n=1 p∈℘ 1 − ps


X χ(n) X χ(n)
− +


ns n s
n=1 n≤N

M
 
X χ(n) Y χ(p) χ(p )

s
− 1 + s
+ · · · + M s
+
n≤N n p p

p≤N
!
χ(pM )
Y  
χ(p) Y 1
1 + s
+ · · · + M s
− χ(p)
+

p≤N p p 1 − ps

p≤N
!
Y
1 Y 1

χ(p)

χ(p)
p≤N 1 − ps p∈℘ 1 − ps

=: |L − SN | + |SN − ΠN,M | + |ΠN,M − ΠN | + |ΠN − Π|


= I + II + III + IV.
Fix  > 0. (I and IV:) By convergence, we can choose N
such that for all numbers greater, We can control the
first and fourth dif-
|SN − L| <  and |ΠN − Π| < . ferences: those are
just the finite ap-
proximations
Next we claim that we can choose M large enough that
II: |SN − ΠN,M | <  and
III: |ΠN,M − ΠN | < .
III is clear since, using the multiplicativity of the Dirichlet
characters, Third difference is
obvious. 
χ(pM ) [χ(p)]M
  
χ(p) χ(p)
lim 1 + s + · · · + M s = lim 1 + s + · · · +
M →∞ p p M →∞ p pM s

X χ(p)n 1
= ns
=
n=1
p 1 − χ(p)
ps
PROOF OF DIRICHLET’S THEOREM 3

The only non-trivial inequality is II.



M
X  
χ(n) Y χ(p) χ(p )
|SN − ΠN,M | := s
− 1 + s
+ · · · + Ms
;
n≤N n p p
p≤N
Of course, for n ≤ N , n’s prime factorization is composed of
primes p ≤ N ; further, there cannot be more than N primes
involved. So take M > N . Then, using multiplicativity,
we see that the second term contains all thePterms in the
first. The sum of the terms not contained in n≤N χ(n) ns are
contained in n>N χ(n)
P
ns , which has magnitude smaller than
 by our choice of N . 
Proposition 1.2 (Corollary of the the product formulae).
Recall the trivial Dirichlet character modulo q: Minor observation
 along the way:
1 if n and q are relatively prime L(s, χ0 ) is basically
χ0 (n) :=
0 otherwise. ζ(s)!

If q = pa11 · · · paNN is the prime factorization of q, then


L(s, χ0 ) = (1 − p−s −s −s
1 )(1 − p2 ) · (1 − pN )ζ(s).

Proof. By the product formula just proven, Proof: use the prod-
X χ0 (n) Y uct formulae.
1
=
n
ns p∈℘ 1 −
χ0 (p)
s p
and by Euler’s product formula for ζ, χ0 (p) = 0 when p|q.
Y 1
ζ(s) =
p∈℘
1 − p1s
The terms that are missing are those which correspond to
the primes that divide q; i.e., p1 , . . . , pN . 
2. Logarithms of L-functions
(Introduce “second reduction” as “corollary” here.) We’d like to be able
to take the log of
Proposition 2.1 (The technical proposition). Let χ be a both sides of the
above product for-
non-trivial Dirichlet character. Then mula. But we’ll need
to do a lot of work
first....
The technical propo-
sition: allowing us to
define the log of L.
Where do we need
0 < s < 1?
4 PROOF OF DIRICHLET’S THEOREM

i.

X χ(n)
L(s, χ) := converges for s > 0.
n=1
ns
Obtain controls on ii. L(s, χ) is continuously differentiable in s.
the growth of L and
d iii. For some constants c, c0 > 0, we have The L(·, χ) functions
ds L
L(s, χ) = 1 + O(e−cs ) as s → ∞ make sense for 0 <
s ≤ 1 - as long as χ
d 0 is non-trivial.
L(s, χ) = O(e−c s ) as s → ∞.
ds
Lemma 2.2 (The key lemma). If χ is a non-trivial Dirich-
Simple but cru- let character modulo q, then
cial observation
k

in demonstrating
X
χ(n) ≤ q for any k.

absolute and uni-
form convergence
n=1

in s > 0
Proof. First recall that since χ is non-trivial,
Xq
χ(n) = 0.
n=1
Q. Why do they Using the Euclidean algorithm, write k = aq + b with
prove this again? 0 ≤ b < q; then
X k aq
X aq+b
X aq+b
X
χ(n) = + χ(n) = χ(n).
n=1 n=1 n=aq+1 n=aq+1
By the triangle inequality (since |χ(n)| ∈ {0, 1}) the last
Characters map into term is less than or equal to q. 
S 1 ; Dirichlet charac-
ters can equal 0. 3. Proof of the technical proposition
Proof of the technical proposition. It is easy to see that the
series defining L(s, χ) converges uniformly and absolutely
d
for s > 1 as does the term-by-term derivative ds L(s, χ).
Proof of (i): To show convergence for s > 0, we rewrite the
series as follows. Let Sk denote the partial sum kn=1 χ(n)
P
We use some elemen- (and S0 := 0). Then
tary manipulation to
rewrite the series....
(Note that the book
is incorrect to say
that (p.263) one
uses summation by
parts.)
PROOF OF DIRICHLET’S THEOREM 5

N N
X χ(k) X Sk − Sk−1
=
ks ks
k=1 k=1
N N
X Sk X Sk−1
= − (since S0 = 0)
ks ks
k=1 k=2
N N −1
X Sk X Sk
= −
ks (k + 1)s
k=1 k=1
N −1  
X 1 1 SN
= Sk s − +
k (k + 1)s Ns
k=1
N −1
X SN
=: fk (s) +
Ns
k=1
P ...as an absolutely
Consider: the convergence of fk (s): by the key lemma, (and uniformly) con-
  verging series! Thus
1 1 the original series
|fk (s)| := Sk s − also converges abso-
k (k + 1)s lutely uniformly.
 
d
x−s = −sx−s−1

≤ q max
x∈[k,k+1] ds
1
= qs ;
k s+1
P
thus k fk (s) converges absolutely and uniformly for s > 0.
Differentiating the series term-by-term, we see by an ar-
gument similar to that above that the differentiated series
converges uniformly for s > 0, and thus converges to a
continuous function.
Proof of ii: Consider: for s > 1 +  (say), Control of the
∞ growth is easy
X χ(n)
|L(s, χ) − 1| :=

n s

n=2
Z ∞  
−s −s 2
≤ x dx = 2 ≤ 2−s O(1).
2 s−1
6 PROOF OF DIRICHLET’S THEOREM

Taking c = log 2, gives the desired result:


L(s, χ) = 1 + O(e−cs ) as s → ∞.
iii is proved similarly. 
4. What was the point of that?
To define the log of L(s, χ)
LECTURE 38: NON-VANISHING OF L(1, χ) FOR
NON-TRIVIAL REAL DIRICHLET CHARACTERS

1. Recall
Our goal is to prove the following.

Theorem 1.1 (Partial theorem, part II). For non-trivial,


real Dirichlet characters χ, L(1, χ) 6= 0.

We stated (without proof) the following propositions. Some simple results


on “p-series”. Of
course they diverge,
Proposition 1.2. There exists a number γ (the Euler- but we will need
Mascheroni constant) satisfying more precise esti-
mates on the partial
N sums.
X 1
− log N = γ + O(1/N ).
n=1
n

(See en.wikipedia.org/wiki/Euler-Mascheroni constant)

Proposition 1.3. For N ∈ N,


N
X 1
1/2
− 2N 1/2 = c + O(1/N 1/2 ).
n=1
n

2. Hyperbolic sums Trivial observations


Let F : N × N → C, N ∈ N, and AN := {(m, n) ∈ N × N : on particular finite
sums: that we can
mn ≤ N }. Let SN denote the finite sum sum them in differ-
X ent ways.
SN := F (m, n).
(m,n)∈AN

Now observe that we can express SN in three different


ways: Completely trivial,
1 but yet so useful....
2 PROOF OF DIRICHLET’S THEOREM

N
X X
SN = F (m, n) (vertically)
m=1 1≤n≤N/m
N
X X
= F (m, n) (horizontally).
n=1 1≤m≤N/n
N X
X
= F (m, n) (i.e., along hyperbolae indexed by k)
k=1 nm=k

3. Return to Dirichlet’s theorem


To finish the proof of Dirichlet’s theorem we will show
that L(1, χ) is well-approximated by ( √1N times) a partic-
ular hyperbolic sum, namely
X
SN := F (m, n)
(m,n)∈AN

where
χ(n)
F (m, n) := √ ;
nm
we’ll also show that the sum grows faster than the log N .
Q. Why is it obvi- Precisely,
ous that this hyper-
bolic sum√ approxi- Proposition 3.1. Let χ be a non-trivial real Dirichlet char-
mates 2 N L(1, χ)?
Recall L(1, χ) = acter. With the above definitions,
P χ(n)
n . i. ∃ c > 0 such that SN ≥ c log N .
ii. SN = 2N 1/2 L(1, χ) + O(1).
Remark. The above proposition finishes the theorem for, if
L(1, χ) = 0 then (ii) would imply that SN = O(1), while
Thus if we prove (i) states that SN ≥ c log N ; a contradiction.
the proposition, the
proof of Dirichlet’s To prove the two parts of the proposition we will need
theorem is over!
the following two lemmas.
Analysis of the sum Lemma 3.2. Let k ∈ N. Then
of χ over divisors of
k
PROOF OF DIRICHLET’S THEOREM 3


X 0 for all k
χ(n) ≥
1 if k = `2 for some ` ∈ Z
n|k

Proof of lemma.
Case I: k = pα for some p prime. Then Simple case first: k a
X prime power.
χ(n) = χ(1) + χ(p) + χ(p2 ) + · · · + χ(pα )
n|k

= χ(1) + χ(p) + χ(p)2 + · · · + χ(p)α .


Now, χ being a real Dirichlet character, χ(p) ∈ {−1, 0, 1};
so 

 α + 1 if χ(p) = 1
1 if χ(p) = −1 and α is even
X 
χ(n) =
 0 if χ(p) = −1 and α is odd
n|k 
1 if χ(p) = 0, i.e., p|q.

P
i.e., as desired, n|k χ(n) is greater than or equal to 0, and
only equals zero when α is odd (i.e., k is not a square).
Case II: General k. If k = pα1 1 · · · pαMM , then the divisors
of k consist of the set The general case
follows immediately
{pβ1 1 · · · pβMM : 0 ≤ βj ≤ αj ; j = 1, . . . , M }. from the first.
Thus every divisor has exactly M prime factors of powers (Bad notation in
text: using N for
0 through αj (and every such product is a divisor); so two purposes)
M
α 
X Y
χ(n) = χ(p0j ) + χ(p1j ) + χ(p2j ) + · · · + χ(pj j ) .
n|k j=1

As before, the only possibility of getting a 0 out of this is


if one of the αj is odd, in which case k is not a square. 
Lemma 3.3 (Second lemma). Let a, b ∈ N. If a < b then
i. bn=a χ(n)
√ = O(a−1/2 ) and
P
n
Pb χ(n)
ii. n=a n = O(a−1 ).
Remark. The proofs of these two facts are relatively straight-
forward; so we leave them to you.
4 PROOF OF DIRICHLET’S THEOREM

Now we can prove the proposition, which we restate again


for convenience.

Proposition 3.4. Let χ be a non-trivial real Dirichlet char-


acter. With the above definitions,
i. ∃ c > 0 such that SN ≥ c log N .
ii. SN = 2N 1/2 L(1, χ) + O(1).

Proof. (i) Summing along hyperbolae, we see

X χ(n)
SN := √
nm
(m,n)∈AN
N X
X χ(n)
= √
nm
k=1 nm=k
N
X 1
X
= √ χ(n).
k=1
k n|k

By the first lemma, then, we can obtain the desired lower


bound on SN as follows:

N
X 1 X
SN = √ χ(n)
k=1
k n|k
X 1 X 1
≥ √ = = log N 1/2 + O(1).
k=`2 ,`≤N 1/2
k `≤N 1/2 `

(ii) We want to get a precise estimate of

X χ(n)
SN := √ .
nm
(m,n)∈AN
PROOF OF DIRICHLET’S THEOREM 5

We separate SN into the sum SI + SII + SIII , where the


indices in the respective sums lie in the regions

√ √ N
I := {(n, m) ∈ N : 1 ≤ m <N, N < n ≤ }
√ √ m
II := {(n, m) ∈ N : 1 ≤ m ≤ N , 1 ≤ n ≤ N }
√ N √
III := {(n, m) ∈ N : N < m ≤ , 1 ≤ n < N }.
n

Consider SI : summing vertically, we get

X χ(n)
SI := √
√ √ N
nm
1≤m< N , N <n≤ m
 
X 1 X χ(n) 
= √  √
√ m √ N
n
m< N N <n≤ m

By the second lemma, the term in parentheses is of the or-


der O((N 1/2 )−1/2 ) = O(N −1/4 ); using the 21 -series estimate

 
X 1 1
= 2M 1/2 + c + O √
1≤n≤M
n1/2 M

we get
  
1
1/4
SI = 2N + c + O 1/4
O(N −1/4 ) = O(1)
N

as N → ∞.
6 PROOF OF DIRICHLET’S THEOREM

For the other two terms, we sum horizontally, again using


the 12 -series estimate:
 
X χ(n) X 1 
SII + SIII = 1/2

√ n m1/2
1≤n< N 1≤m≤ N
(  n   )
X χ(n) 1/2
N n 1/2
= 1/2
2 + c + O
√ n n N
1≤n< N
 
X χ(n) X χ(n)
 1
X
= 2N 1/2 +c + O 1 ,
1/2
n 1/2
n1/2 N 1/2
1≤n≤N 1≤n≤N 1≤n≤N 1/2
the last term from
X χ(n) n1/2 1 X
= χ(n).
1/2
n1/2 N 1/2 N 1/2
1≤n<N 1≤n≤N 1/2
Let’s call these three terms A, B, and C.
Now, since L(1, χ) = ∞ χ(n)
P
n=1 n , and since

1/2
X χ(n) 1/2

−1/2

N =N O N ,
1/2
n
n=N
we see that A = 2N 1/2 L(1/χ) + O(1). Further, part (i) of
the lemma implies that
X χ(n)
1/2
= O(1);
1/2
n
1≤n≤N
thus B = O(1). Finally, obviously C = O(1), and so
SN = 2N 1/2 L(1/χ) + O(1),
and the proposition is proved. 
LECTURE 37: PRODUCT FORMULA FOR THE
L-FUNCTIONS

We’d like to be able


1. Recall the technical proposition
to take the log of Proposition 1.1 (The technical proposition). Let χ be a
both sides of the
above product for- non-trivial Dirichlet character. Then
mula. But we’ll need
to do a lot of work
i.

first.... X χ(n)
The technical propo- L(s, χ) := s
converges for s > 0.
sition: allowing us to n=1
n
define the log of L.
Where do we need ii. L(s, χ) is continuously differentiable in s.
0 < s < 1? iii. For some constants c, c0 > 0, we have
The L(·, χ) functions
make sense for 0 < L(s, χ) = 1 + O(e−cs ) as s → ∞
s ≤ 1 - as long as χ
d 0
is non-trivial. L(s, χ) = O(e−c s ) as s → ∞.
Obtain controls on ds
the growth of L and
d 2. What was the point of that?
ds L
To define the log of L(s, χ)
Notice we don’t de-
fine log2 in general. Definition 2.1. Let s > 1, χ a non-trivial Dirichlet char-
acter. We define the “logarithm of L(s, χ)” by
Z ∞ 0
L (t, χ)
log2 L(s, χ) := − dt.
s L(t, χ)
The point is that this
integral turns out,
for a non-trivial Notice that the integral converges, since L(t, χ) can be made
Dirichlet charac-
ter to be the pri- arbitrarily close to 1, and L0 (t, χ) = O(e−ct ), which together
mary branch of the imply
log.
L0 (t, χ)
We need control of = O(e−ct ) as t → ∞.
the growth to justify L(t, χ)
this definition.
Proposition 2.2. Let s > 1. Then
i. elog2 L(s,χ) = L(s, χ) and (most importantly for us,)
1
2 PROOF OF DIRICHLET’S THEOREM
 
1
P
ii. log2 L(s, χ) = p∈℘ log1 Getting the loga-
1− χ(p)
ps rithmic version of
Dirichlet’s product
Proof. (i) We want to show that formula for the
L-functions
e− log2 L(s,χ) L(s, χ) = 1.
Well, the derivative of the LHS w.r.t s is The derivative trick
again.
L0 (s, χ)
 
− log2 L(s,χ) 0 − log2 L(s,χ)
e L (s, χ) + e − L(s, χ) = 0,
L(s, χ)
so the LHS is constant with respect to s. Now consider
R∞ L0 (t,χ)
lim e− log2 L(s,χ) L(s, χ) := lim e s L(t,χ) dt L(s, χ).
s→∞ s→∞
The integral, being convergent, vanishes as s → ∞, so the
first term tends to 1 as s → ∞; since L(s, χ) = 1 + O(e−cs ),
so does the second.
(ii) Now let’s show The crucial sec-
! ond step in the
X 1 second reduction
log2 L(s, χ) = log1 χ(p)
.
p∈℘ 1 − ps
We’ll show eA = eB ,
LHS
Well, consider the exponential of both sides. e = which almost implies
log2 L(s,χ) A = B...
e = L(s, χ), as we just saw. As for the RHS,
!
1
P
p log1 χ(p)
log2 L(s,χ) 1− s
e =e p
!
1
Y log1 χ(p)
1− s
= e p

p∈℘
!
Y 1
= χ(p)
= L(s, χ) also.
p∈℘ 1− ps
We’re not done! This only means that ...but not quite.
!
X 1
log2 L(s, χ) − log1 χ(p)
= 2πiM (s)
p∈℘ 1 − ps
PROOF OF DIRICHLET’S THEOREM 3

for some integer-valued function M . It is left as an exercise


to show that M is continuous and that lims→∞ M (s) = 0
which forces M ≡ 0 (since M (s) ∈ Z). 

What was the point


3. Second reduction of the problem
of all that? By the proposition just proved and subsequently the prop-
Following the same erties of log1 proven earlier,
lines of the argu- !
ment of Euler, but X 1
to show finiteness log2 L(s, χ) = log1
rather than infinite- p∈℘ 1 − χ(p)
ps
ness.   
X χ(p) 1
= s
+E
p∈℘
p ps
X χ(p)
= s
+ C.
p∈℘
p
Now, if we can show that L(1, χ) 6= 0 for χ non-trivial, then
Here’s why we since L(s, χ) is continuously differentiable for 0 < s < ∞,
needed the integral
representation of the
L0 (s, χ) is bounded near s = 1, so
Z ∞ 0
log L (t, χ)
lim+ log2 L(s, χ) := lim+ dt is bounded.
s→1 s→1 s L(t, χ)
In that case, the above equality implies lims↓1 p∈℘ χ(p)
P
ps is
also bounded; that is, the reduced problem would be solved.
Now comes the deep Thus it suffices to prove the following claim:
part....
My God, what a Theorem 3.1. L(1, χ) 6= 0 for non-trivial χ.
wonderful piece of
work. We will do this in two cases, that of complex Dirichlet char-
acters and that of real ones. The former is the easier.

4. Non-vanishing of the L-function


Case I: complex Dirichlet characters
Theorem 4.1 (Partial theorem). For non-trivial, complex
Dirichlet characters χ, L(1, χ) 6= 0.
4 PROOF OF DIRICHLET’S THEOREM

The proof will be by contradiction and rely on the following


two lemmas.
Q
Lemma 4.2. Let s > 1. Then χ L(s, χ) ≥ 1; in particu-
lar, the product is real-valued.
Product over all χ is
real-valued.
Proof. Using the product formula for the L-function and
the definition of log1 , we see
!!
Y XX 1
L(s, χ) = exp log1
χ χ p∈℘ 1 − χ(p)
ps
!!
XX 1
= exp log1
χ p∈℘ 1 − χ(p)
ps

!
XXX 1 χ(pk )
= exp
χ p∈℘ k=1
k pks

!
XX 1 1 X
= exp ks
χ(pk ) .
p∈℘
kp χ
k=1
Now, by Fourier series expansion
P of the characteristic func- (Another cool appli-
k 1 k cation of Fourier se-
tions of points, δ1 (p ) = ϕ(q) χ χ(p ), so the above equals ries.)

! Error in text: δ1 ,
XX 1 1
exp ks
ϕ(q)δ1 (pk ) ≥ 1, not δ0 . (Unless by 0
they mean 1.)
p∈℘
kp
k=1
since the term in the parentheses is non-negative. 
Lemma 4.3. On the behavior of
L(s, χ) as s ↓ 1: how
i. If L(1, χ) = 0, the L(1, χ) = 0 (obvious). it vanishes; how it
ii. If L(1, χ) = 0 and χ is non-trivial, then blows up.

|L(s, χ)| ≤ C|s − 1| for 1 ≤ s ≤ 2.


iii. In the case of χ = χ0 , we have
C
|L(s, χ0 )| ≤ for 1 < s ≤ 2.
|s − 1|
PROOF OF DIRICHLET’S THEOREM 5

Proof. i) L(1, χ) = L(1, χ).


ii) For χ non-trivial, L(s, χ) is continuously differentiable
for s > 0. Thus, by the Mean Value Theorem,
L(s, χ) − L(1, χ)
= L0 (s0 , χ)
s−1
for some s0 ∈ (1, s). Since L0 (s, χ) is continuous, it has a
bound C on [1, 2].
1 s
iii) Recall that ζ(s) ≤ 1 + s−1 = s−1 . For 1 < s ≤ 2,
then,
2
|ζ(s)| ≤ .
s−1
Since L(s, χ0 ) = Cζ(s), we are done. 
Theorem 4.4 (Partial theorem). For non-trivial, complex
Dirichlet characters χ, L(1, χ) 6= 0.
Proof of the partial theorem. By contradiction: suppose L(1, χ1 ) =
0; then by the previous lemma,
The point: for i. L(1, χ1 ) = 0 and
complex Dirichlet ii. For χ = χ1 and χ1 ,
characters, we’re
guaranteed that |L(s, χ)| ≤ C|s − 1| for 1 ≤ s < 2.
they occur in pairs; Q
so if L(1, χ) = 0, If we consider the product χ L(s, χ), then, we see that
then L(1, χ) = 0,
overpowering the
there are at least two Dirichlet characters (χ1 and χ1 ) that
blowing-up L(1, χ0 ). vanish like |s − 1| as s ↓ 1. L(s, χ) being continuously
differentiable on s > 0 for χ non-trivial, we see the only
term in this product that could (and does) go to infinity as
1
s ↓ 1 is L(s, χ0 ). But its growth is of order O( s−1 ), so
Y
lim L(s, χ) = 0, in contradiction with the earlier lemma.
s→1+
χ

LECTURE 37: NON-VANISHING OF L(1, χ) FOR
COMPLEX DIRICHLET CHARACTERS

We’d like to be able


1. Recall the Technical Proposition
to take the log of
both sides of the
Proposition 1.1 (The technical proposition). Let χ be a
above product for- non-trivial Dirichlet character. Then
mula. But we’ll need
to do a lot of work
i.

first.... X χ(n)
The technical propo- L(s, χ) := s
converges for s > 0.
sition: allowing us to n=1
n
define the log of L.
Where do we need
ii. L(s, χ) is continuously differentiable in s.
0 < s < 1? iii. For some constants c, c0 > 0, we have
The L(·, χ) functions
make sense for 0 < L(s, χ) = 1 + O(e−cs ) as s → ∞
s ≤ 1 - as long as χ d 0
is non-trivial. L(s, χ) = O(e−c s ) as s → ∞.
Obtain controls on ds
the growth of L and
d 2. What was the point of that?
ds L
To define the log of L(s, χ)
Notice we don’t de-
fine log2 in general. Definition 2.1. Let s > 1, χ a non-trivial Dirichlet char-
acter. We define the “logarithm of L(s, χ)” by
Z ∞ 0
L (t, χ)
log2 L(s, χ) := − dt.
s L(t, χ)
The point is that this
integral turns out,
for a non-trivial
Dirichlet charac- Remark. Notce that this integral converges since L(t, χ) can
ter to be the pri- be made arbitrarily close to 1, and L0 (t, χ) = O(e−ct ), which
mary branch of the
log. together imply
We need control of L0 (t, χ)
the growth to justify = O(e−ct ).
this definition. L(t, χ)
Proposition 2.2. Let s > 1. Then
i. elog2 L(s,χ) = L(s, χ) and, most importantly for us,
1
2 PROOF OF DIRICHLET’S THEOREM
 
1
P
ii. log2 L(s, χ) = p∈℘ log1 Getting the loga-
1− χ(p)
ps rithmic version of
Dirichlet’s product
Proof. i) We want to show that formula for the
L-functions
e− log2 L(s,χ) L(s, χ) = 1.
Well, the derivative of the LHS w.r.t s is
L0 (s, χ)
 
− log2 L(s,χ) 0 − log2 L(s,χ)
e L (s, χ) + e − L(s, χ) = 0,
L(s, χ)
so the LHS is constant with respect to s. Now consider
R∞ L0 (t,χ)
lim e− log2 L(s,χ) L(s, χ) := lim e s L(t,χ) dt L(s, χ).
s→∞ s→∞
The integral, being convergent, vanishes as s → ∞, so the
first term tends to 1 as s → ∞; since L(s, χ) = 1 + O(e−cs ),
so does the second.
ii) Now let’s show The crucial sec-
! ond step in the
X 1 second reduction
log2 L(s, χ) = log1 χ(p)
.
p∈℘ 1 − ps
We’ll show eA = eB ,
LHS
Well, consider the exponential of both sides. e = which almost implies
log2 L(s,χ) A = B...
e = L(s, χ), as we just saw. As for the RHS,
!
1
P
p log1 χ(p)
log2 L(s,χ) 1− s
e =e p
! !
1 1
P
p log1 χ(p) Y log1 χ(p)
1− s 1− s
=e p = e p

p∈℘
!
Y 1
= χ(p)
= L(s, χ) also.
p∈℘ 1− ps
We’re not done! This only means that ...but not quite.
!
X 1
log2 L(s, χ) − log1 χ(p)
= 2πiM (s)
p∈℘ 1 − ps
PROOF OF DIRICHLET’S THEOREM 3

for some integer-valued function M . It is left as an exercise


to show that M is continuous and that lims→∞ M (s) = 0
which forces M ≡ 0 (since M (s) ∈ Z). 

What was the point


3. Second reduction of the problem
of all that? By the proposition just proved and subsequently the prop-
Following the same erties of log1 proven earlier,
lines of the argu- !
ment of Euler, but X 1
to show finiteness log2 L(s, χ) = log1
rather than infinite- p∈℘ 1 − χ(p)
ps
ness.   
X χ(p) 1
= s
+E
p∈℘
p ps
X χ(p)
= s
+ C.
p∈℘
p
Now, if we can show that L(1, χ) 6= 0 for χ non-trivial, then
(Here’s why we since L(s, χ) is continuously differentiable for 0 < s < ∞,
needed the integral
representation of
L0 (s, χ) is bounded near s = 1, so
Z ∞ 0
log L.) L (t, χ)
lim log2 L(s, χ) := lim+ dt is bounded.
s↓1 s→1 s L(t, χ)
In that case, by the above equality, lims↓1 p∈℘ χ(p)
P
ps is also
bounded; that is, the reduced problem would be solved.
Now comes the deep Thus it suffices to show the following claim:
part....
My God, what a Theorem 3.1. L(1, χ) 6= 0 for non-trivial χ.
wonderful piece of
work. We will do this in two cases, that of complex Dirichleet
characters and that of real ones. The former is the easier.

4. Non-vanishing of the L-function


Case I: complex Dirichlet characters
Theorem 4.1 (Partial theorem). For non-trivial, complex
Dirichlet characters χ, L(1, χ) 6= 0.
4 PROOF OF DIRICHLET’S THEOREM

The proof will be by contradiction and rely on the following


two lemmas.
Q
Lemma 4.2. Let s > 1. Then χ L(s, χ) ≥ 1; in particu-
lar, the product is real-valued.
Proof. Using the product formula for the L-function and
the definition of log1 , we see
!!
Y XX 1
L(s, χ) = exp log1
χ χ p∈℘ 1 − χ(p)
ps
!!
XX 1
= exp log1
χ p∈℘ 1 − χ(p)
ps

!
XXX 1 χ(pk )
= exp
χ p∈℘ k=1
k pks

!
XX 1 1 X
= exp ks
χ(pk ) .
p∈℘
kp χ
k=1
Now, by Fourier series expansion
P of the characteristic func- (Another cool appli-
k 1 k cation of Fourier se-
tions of points, δ1 (p ) = ϕ(q) χ χ(p ), so the above equals ries. Recall δ` (m) =
1
P
∞ χ χ(`)χ(m).)
!
ϕ(q)
XX 1 1
exp ks
ϕ(q)δ1 (pk ) ≥ 1, Error in text: δ1 ,
p∈℘
kp not δ0 . (Unless by 0
k=1 they mean 1.)
since the term in the parentheses is non-negative. 
Lemma 4.3. On the behavior of
L(s, χ) as s ↓ 1: how
i. If L(1, χ) = 0, the L(1, χ) = 0 (obvious). it vanishes (if it van-
ii. If L(1, χ) = 0 and χ is non-trivial, then ishes); how it blows
up (if χ = χ0 ).
|L(s, χ)| ≤ C|s − 1| for 1 ≤ s ≤ 2.
iii. In the case of χ = χ0 , we have
C
|L(s, χ0 )| ≤ for 1 < s ≤ 2.
|s − 1|
PROOF OF DIRICHLET’S THEOREM 5

Proof. i) L(1, χ) = L(1, χ)


ii) For χ non-trivial, L(s, χ) is continuously differentiable
for s > 0. Thus, by the Mean Value Theorem,
L(s, χ) − L(1, χ)
(1) = L0 (s0 , χ)
s−1
for some s0 ∈ (1, s). Since L0 (s, χ) is continuous, it has a
bound C on [1, 2].
R∞ 1 s
1 dx
iii) Recall that ζ(s) ≤ 1 + s−1 = s−1 . For 1 < s ≤ 2, then,
P
ns≤ 1+ 1 xs =
1
1 + s−1 .
2
(2) |ζ(s)| ≤ .
s−1
Since L(s, χ0 ) = cζ(s), we are done. 
Theorem 4.4 (Partial theorem). For non-trivial, complex
Dirichlet characters χ, L(1, χ) 6= 0.
Proof of the partial theorem.
By contradiction: suppose L(1, χ1 ) 6= 0; then, by the
The point: for previous lemma, L(1, χ1 ) = 0 also, and for both χ = χ1 , χ1 ,
complex Dirichlet
characters, we’re
we have
guaranteed that (3) |L(s, χ)| ≤ C|s − 1| for 1 ≤ s < 2.
they occur in pairs; Q
so if L(1, χ) = 0, Now consider χ L(s, χ). For non-trivial χ, L(s, χ) is
then L(1, χ) = 0, continuously differentiable on s > 0 and thus finite at
overpowering the
blowing-up of L(1, χ) (i.e., when s = 1; among those Dirichlet charac-
L(1, χ0 ) as s ↓ 1. ters, there are at least two (viz., χ1 and χ1 ) that vanish
(like O(|s − 1|) as s ↓ 1). The only term that can (and
 to infinity as s ↓ 1 is L(s, χ0 ), which grows like
does) go
1
O s−1 . Thus
Y
(4) lim+ L(s, χ) = 0,
s→1
χ
Q
in contradiction with the fact that χ L(s, χ) ≥ 1.

LECTURE 38: NON-VANISHING OF L(1, χ) FOR
NON-TRIVIAL REAL DIRICHLET CHARACTERS

1. Recall the Goal; Behavior of p-series


Our goal is to prove the following.
Theorem 1.1 (Partial theorem, part II). For non-trivial,
real Dirichlet characters χ, L(1, χ) 6= 0.
We stated (without proof) the following propositions. Precise estimates on
the partial sums of
Proposition 1.2. There exists a number γ (the Euler- certain divergent p-
series.
Mascheroni constant) satisfying
N
X 1
− log N = γ + O(1/N ).
n=1
n
(See en.wikipedia.org/wiki/Euler-Mascheroni constant)
Proposition 1.3. For N ∈ N,
N
X 1
1/2
− 2N 1/2 = c + O(1/N 1/2 ).
n=1
n
2. Approximating L(1, χ) with hyperbolic sums
To finish the proof of Dirichlet’s theorem we will show
that L(1, χ) is well-approximated by a particular hyperbolic
sum, namely The trick: approxi-
X χ(n) mating L(1, χ) with
SN := √ a particular hyper-
nm bolic sum.
(m,n)∈AN
where AN := {(m, n) ∈ N × N : mn ≤ N }. We’ll see that

X χ(n) 1 X χ(n) 1
L(1, χ) := ≈ √ √ =: √ SN
n=1
n 2 N (m,n)∈A mn 2 N
N

1
2 PROOF OF DIRICHLET’S THEOREM

and, further, that the sum grows faster than log N . Pre-
Q. Why is it obvi- cisely:
ous that this hyper-
bolic sum√ approxi- Proposition 2.1. Let χ be a non-trivial real Dirichlet char-
mates 2 N L(1, χ)?
Recall L(1, χ) =
acter. With the above definitions,
P χ(n)
n .
i. ∃ c > 0 such that SN ≥ c log N .
ii. SN = 2N 1/2 L(1, χ) + O(1).
Remark. The above proposition finishes the theorem for, if
L(1, χ) = 0 then (ii) would imply that SN = O(1), while
Thus if we prove (i) states that SN ≥ c log N ; a contradiction.
the proposition, the
proof of Dirichlet’s To prove the two parts of the proposition we will need
theorem is over!
the following two lemmas.
Analysis of the sum ∈ N. Then
Lemma 2.2. Let k 
of χ over divisors of X 0 for all k
k χ(n) ≥
1 if k = `2 for some ` ∈ Z
n|k

Proof of lemma.
Simple case first: k a Case I: k = pα for some p prime. Then
prime power. X
χ(n) = χ(1) + χ(p) + χ(p2 ) + · · · + χ(pα )
n|k

= χ(1) + χ(p) + χ(p)2 + · · · + χ(p)α .


Now, χ being a real Dirichlet character, χ(p) ∈ {−1, 0, 1};
so 

 α + 1 if χ(p) = 1
1 if χ(p) = −1 and α is even
X 
χ(n) =
 0 if χ(p) = −1 and α is odd
n|k 
1 if χ(p) = 0, i.e., p|q.

P
i.e., as desired, n|k χ(n) is greater than or equal to 0, and
only equals zero when α is odd (i.e., k is not a square).
Case II: General k. If k = pα1 1 · · · pαMM , then the divisors
The general case of k consist of the set
follows immediately
from the first. {pβ1 1 · · · pβMM : 0 ≤ βj ≤ αj ; j = 1, . . . , M }.
(Bad notation in
text: using N for
two purposes)
PROOF OF DIRICHLET’S THEOREM 3

Thus every divisor has exactly M prime factors of powers


0 through αj (and every such product is a divisor); so
M
α 
X Y
χ(n) = χ(p0j ) + χ(p1j ) + χ(p2j ) + · · · + χ(pj j ) .
n|k j=1

As before, the only possibility of getting a 0 out of this is The product can
if one of the αj is odd, in which case k is not a square.  equal zero only
when one of the
sums is 0; by the
Lemma 2.3 (Second lemma). Let a, b ∈ N. If a < b then previous case, αj
must be odd
i. bn=a χ(n)
√ = O(a−1/2 ) and
P
n
Pb χ(n)
ii. n=a n = O(a−1 ).

Remark. The proofs of these two facts are relatively straight-


forwardl; we leave them to you.

Now we can prove the proposition, which we restate again


for convenience.

Proposition 2.4. Let χ be a non-trivial real Dirichlet char-


acter. With the above definitions,
i. ∃ c > 0 such that SN ≥ c log N .
ii. SN = 2N 1/2 L(1, χ) + O(1).

Proof. (i) Summing along hyperbolae, we see Summing along


hyperbolae gives
X χ(n) the lower bound
SN := √ result.
nm
(m,n)∈AN
N X
X χ(n)
= √
nm
k=1 nm=k
 
N
X 1 X
= √  χ(n) .
k=1
k n|k

By the first lemma, then, since the term in parentheses is (The pair (m, n)
such that nm = k is
determined by the
first entry.)
4 PROOF OF DIRICHLET’S THEOREM

≥ 1 when k is square,
X 1
SN ≥ √
k=`2 ,`≤N 1/2
k
X 1
= = log N 1/2 + O(1),
1/2
`
`≤N

which is the desired lower bound on SN (the last equality


follows from the estimate of the partial sum of the harmonic
series).
Summing ver- (ii) Consider:
tically and
horizontally gives X χ(n)
L(1, χ). SN := √ .
nm
(m,n)∈AN

Break the sum We separate SN into the sum SN = SI + SII + SIII , where
into various regions.
(Draw the hyperbola
the indices for the respective sums lie in the regions
correctly!) √ √ N
I := {(n, m) ∈ N : 1 ≤ m < N , N < n ≤ }
√ √ m
II := {(n, m) ∈ N : 1 ≤ m ≤ N , 1 ≤ n ≤ N }
√ N √
III := {(n, m) ∈ N : N < m ≤ , 1 ≤ n < N }.
n
Consider SI : summing vertically, we get
X χ(n)
SI := √
√ √ N
nm
1≤m< N , N <n≤ m
 
X 1 X χ(n)
= √  √ 
√ m √ N
n
m< N N <n≤ m

Use the bound on


P χ(n) By the second lemma, the term in parentheses is of the or-

n der O((N 1/2 )−1/2 ) = O(N −1/4 ); using the 21 -series estimate
and the bound on the we get
p = 12 -series
PROOF OF DIRICHLET’S THEOREM 5

  
1
1/4
SI = 2N + c + O 1/4
O(N −1/4 ) = O(1)
N
as N → ∞.
For the other two terms, we sum horizontally. Using the
1
2 -series estimate, SII + SIII : use the
1
2 -series bound again
 
X χ(n) X 1 
SII + SIII = 1/2

√ n N
m1/2
1≤n< N 1≤m≤
(  n   )
X χ(n) 1/2
N n 1/2
= 1/2
2 + c + O
√ n n N
1≤n< N
 
X χ(n) X χ(n)
 1
X
= 2N 1/2 +c 1/2
+ O 1/2
1 ,
1/2
n 1/2
n N 1/2
1≤n≤N 1≤n≤N 1≤n≤N

the last term from


X χ(n) n1/2 1 X
= χ(n).
1/2
n1/2 N 1/2 N 1/2
1≤n<N 1≤n≤N 1/2

Let’s call these three terms A, B, and C.


We’ll deal with them in reverse order. Obviously C = C and B are O(1)
O(1). Further, part (i) of the lemma implies that
X χ(n)
1/2
= O(1);
1/2
n
1≤n≤N

thus B = O(1). A: we basically have


Now, since L(1, χ) = ∞ χ(n) L(1, χ)
P
n=1 n , and since


X χ(n) 
−1/2

=O N ,
1/2
n
n=N
6 PROOF OF DIRICHLET’S THEOREM

we see that
X χ(n)
A := 2N 1/2
n
1≤n≤N 1/2
h  i
1/2 −1/2
= 2N L(1, χ) − O N
= 2N 1/2 L(1, χ) + O(1).
giving the desired estimate for SN . 

You might also like